Você está na página 1de 160

AN AUSTRALlAN MAT11EMAT1CS TRUST PUBLlCATlON

Published by
A U S T R A L I AN MATHEMATICS T R U S T

Australian M athematics Trust


University of Canberra ACT 2601
AUSTRALIA

Copyright

1998 Australian Mathematics Trust

Telephone: +61 2 6201 5137

AMTOS Pty Ltd ACN 058 370 559


National Library of Australia Card Number and lSSN
Australian Mathematics Trust Enrichment Series lSSN 1326-0170
Polish and Austrian Mathematical Olympiads 1981-1995
ISBN 1 876420 02 2

THE A USTRALlAN MATHE MATICS T R U S T

ENRlCHMENT

SERlES

EDlTORlAL COMMlTTEE

Chairman

GRAHAM H POLLARD, Canberra AUSTRALIA

Editor

PETER J TAYLOR, Canberra AUSTRALIA

WARREN J ATKINS, Canberra AUSTRALIA


ED J BARBEAU, Toronto CANADA
GEORGE BERZSENYl, Terra Haute USA
RoN DUNKLEY, Waterloo CANADA
WALTER E MIENTKA, Lincoln USA
NIKOLAY KONSTANTINOV, Moscow RUSSIA
ANDY L!U, Edmonton CANADA
JORDAN B TABOV, Sofia BULGARIA
JOHN WEBB, Cape Town SouTH AFRICA
The books in this series are selected for the motivating, interesting
and stimulating sets of quality problems, with a lucid expository style
in their solutions. Ty pically, the problems have occurred in either
national or international contests at the secondary school level.
They are intended to be sufficiently detailed at an elementary level
for the mathematically inclined or interested to understand but, at
the same time, be interesting and sometimes challenging to the
undergraduate and the more advanced mathematician. lt is believed
that

these

mathematics

competition

problems

are

influence on the learning and enrichment of mathematics.

positive

THE AUSTRALIAN MATHE MATI CS TRUST

ENRlCHMENT
BOOKS
1

SERlES

THE SERlES

lN

ALL THE BEST FROM THE AUSTRALIAN MATHEMATICS COMPETITION

JD Edwards, DJ King

Et

PJ O'Halloran

MATHEMATICAL TOOLCHEST

AW Plank

Et NH

Williams

TOURNAMENT OF TOWNS QUESTIONS AND SOLUTIONS 1984-1989

AUSTRALIAN MATHEMATICS COMPE11110N BOOK 2 1985-1991

PJ Taylor

PJ O'Halloran,

Pollard

Et

PJ Taylor

PROBLEM SoLVING VIA THE AMC

TOURNAMENT OF TOWNS QUESTIONS AND SOLUTIONS 1980-1984

7
8
9
1

10

11

12

W Atkins
PJ Taylor

TOURNAMENT OF TOWNS QUESTIONS AND SOLUTIONS 1989-1993

PJ Taylor

THE AsiAN PACIFIC MATHEMATICS OLYMPIAD

H Lausch
METHODS OF PROBLEM SOLVING BOOK 1

JB Tabov

Et

PJ Taylor

CHALLENGE! 1991-1995

JB Henry, J Dowsey, AR Edwards, U M ottershead,


A N akos Et G Vardaro
USSR MATHEMATICAL OLYMPlADS 1989-1992

AM Slinko

AUSTRALIAN MATHEMATICAL OLYMPIADS 1979-1995

H Lausch

Et

PJ Taylor

13

CHINESE MATHEMATICS COMPETITIONS AND 0LYMPlADS 1981-1993

14 PoLISH AND AusTRIAN MATHEMATICAL OLYMPIADS 1981-1995

A Liu

ME Kuczma

Et E

Windischbacher

FOREWORD

The traditions of National Mathematical Olympiads in many European


countries dates back to about 1950 (in some cases further back). These
Olympiads are used, inter alia, to select national teams to participate in
the International Mathematical Olympiad.
The traditions of the Polish and Austrian Mathematical Olympiads are
particularly strong, and to a certain extent they are linked, since together
they developed the Austrian-Polish Mathematical Olympiad, one of the
world's strongest regional events.
As the reader will determine, the problems in this book are quite exquis
ite, having been hand-picked from the problems of many years. They are
also noted for having multiple independent solutions, making the math
ematics so much richer. There can be little more satisfying than finding
a different, independent solution to a known one. Being mathematics, of
course, the result is always the same after having taken a quite different
route.
The authors of this book have many decades of experience at this level.
Of the two, I have only had the pleasure of personally knowing Dr
Kuczma. Dr Kuczma has one of the world's highest reputations in prob
lem creation. Indeed, he has had no less than four of his problems posed
in International Mathematical Olympiads. Further, he has had many
more reach the final preselection stage. He is also equally renowned as
a problem solver. From mutual acquaintances and examination of the
work in this book, Erich Windischbacher is held in no less regard.
The Australian Mathematics Trust aims to set a high standard of mate
rial and exposition in this Enrichment Series. The contents of this series
involve pedagogical material in problem solving and instructive problems
which have not appeared before in English. We are confident that this
book achieves the high standards to which we have aimed.
Peter Taylor
Executive Director
Australian Mathematics Trust
Canberra
4 August 1998

PREFACE
Mathematics Olympiads have a long tradition in Poland as well as in
Austria, and they have many features in common in both these countries.
Academic supervision comes from the Mathematical Societies and from
university centres. Financial support is provided, in the greatest part, by
the Ministries of Education (the exact official name of that institution,
.
in each country, has changed several times during the past decades).
The effective running of the competitions relies on people (high school
teachers and university teachers) whose enthusiasm and devotedness is
practically the sole motive for their activities.
The organizational format is much the same in the two countries. Con
testants are high school students, most of them attending the last or
the last but one grade. College students are not allowed to participate.
The final round of the Austrian MO and of the Polish MO is a two day
written exam, with three problems to be solved each day -just like the
IMO. As regards earlier stages, there are some differences; but, anyhow,
ea<::h elimination round consists of problem solving. All the problems
posed at our olympiads are essay type; all steps of the reasoning have
to be explained and justified by the solver - short answer questions or
multiple choice questions are not used.
In the late seventies, a bilateral agreement on cultural exchange was
concluded between the Polish and the Austrian Ministers of Education.
This resulted, in particular, in frequent visits of scientists and teachers,
from one country to the other, and has led to exchange of experience
for instance, in the orgnization of math olympiads (remember that, in
those years, Austria and Poland pertained to distinct political zon of
Europe). It is also in that time that the Austrian-Polish Mathematics
Competition was launched.1
The authors of the present book are just two of those "enthusiasts of the
Olympic idea in mathematics", for many years involved in the running of
the national mathematics olympiads in Poland and in Austria. It is quite
a time ago that we first met. Soon the idea occurred to us to present, in
book form, a selection of our countries' olympiad problems.
As a guideline for the selection, we have decided to take the diversity of
methods of solution. Accordingly, each problem in this book is presented
1A

compilation of all the problems posed at the first sixteen rounds of that compe

tition, with complete solutions, has appeared in book form:

ME Kuczma,

144 problems of the Austrian-Polish Mathematics Competition 1978-93,

Problems.

published

1994 by: The Academic Distribution Center, 1216 Walker Rd., Freeland, Maryland
21053, USA.

viii

Preface

with at least two solutions, and sometimes more than two; this feature of
the book we consider important enough to be reflected in the sub-title.
It is obvious that various ways of approach to any problem provide a
better understanding of its nature, reveal several aspects of the relevant
topics and teach various techniques.
Now, it can always be questioned whether a different solution is a really
different one. In some rare cases, it can be justly considered as such.
In many cases, it cannot - and this is evident at first glance. And in
most other cases- also not; the "second" method can use other sym
bols, language, terminology, it may look quite unlike the "first" one, and
still be, in fact, the same. For instance: is the Law of Cosines any
thing else than operating with vectors and their inner products? Is the
examination of divisibility of polynomials via manipulation in real do
main anything essentially different from complex roots and factorization
technique? Combinatorial arguments, when disguised in the language
of polynomials (in fact, the generating functions of the quantities un
der consideration) , do they really differ from the analogous arguments
presented in pure form, without disguise?
This list can be continued, of course. Viewed from a certain level of
professionalism, all or almost all approaches to a particular olympiad
style problem are just like dressing the same idea in a robe of one or
another colour. What can be, however, immediately recognized by a
mathematician, need by no means be evident to a young student who
just makes the first steps in off-curricular areas of mathematics.
Indeed, we think that - besides getting acquainted with various tools
and tricks supplied by various methods - the reader's own discovery
of the intrinsic uniformity hidden behind apparently distinct ways of ap
proach is the true profit she or he can have from studying those solutions,
and is the best we can offer her or him.
Most of our solutions have been elaborated in detail. The intention was
to make them accessible to a rather wide audience; some readers will
find them unnecessarily lengthy, perhaps. We are sure that the readers'
invention will often go further; no doubt, they will find yet other ways
of resolving this or that problem, possibly more elegant or more general
than the presented ones. So much the better! Satisfaction from a good
job done is the solver's true reward.
( Another kind of satisfaction comes from detecting the authors' errors
and mistakes; these are also very instructive! )
There is one more thing we must mention here. There should be no
surprise if a problem turns out to be identical or very closely related to a
question that had appeared at some other competition or in the problem
section of some journal. It is no secret that problems "circulate" and

Preface

ix

are being "borrowed" from one competition to another. There is also


nothing paradoxical in the fact that very similar ideas occur to people
who independently devise olympiad stuff, in distant parts of the globe.
Although we have tried to avoid the use of problems of which we knew
to have been used elsewhere, we can by no means be sure. . .
We are presenting 64 problems (a beautifully round number) from the
two national olympiads, half from the Austrian, half from the Polish. 2
They are arranged more or less thematically; the rough rules are (ilasy to
spot. Such rules can never be quite univalent; a problem may be difficult
to classify; it can pertain to more than one thematic area, sometimes
depending on the solution method.
The arrangement has nothing to do with the level of difficulty; quite
challenging problems often follow or are followed by trivially simple ones.
The reader should not know "what to expect next".
We will be happy to receive any feedback from the readers: comments,
communication about mistakes, any suggestions. We wish all the readers
joy, fun and pleasure in tackling the problems.
Marcin E Kuczma

Erich Windischbacher

Institute of Mathematics
University of Warsaw
ul. Banacha 2
PL-02-097 Warsaw
Poland

Bundesrealgymnasium
KeplerstraBe 1
A-8020 Graz
Austria

2Problems from the Austrian MO: 1, 4, 5, 7, 9-12; 16-19, 23, 28-30, 32-35, 37,

38, 43, 52, 53, 55-61.

Problems from the Polish MO: 2, 3, 6, 8, 13-15, 20-22, 24-27, 31, 36, 39-42, 44-51,

54, 62-64.

ACKNOWLEDGEMENTS

We are happy to see our book appearing as an AMT publication.


Our sincere thanks go to Professor Peter Taylor, the executive director
of the AMT, for his invitation to publish this book in the AMT Enrich
ment Series and for his help in typesetting/ formatting; and to Dr Andrei
Storozhev for producing the diagrams.
MEK&EW
May, 1998

CONTENTS

FOREWORD

PREFACE

ACKNOWLED G EMENTS

PROBLEMS
Arithmetic and Combinatorics
Algebra
G eometry

vii
xi

11

SOLUTlONS
Arithmetic and Combinatorics
Algebra
G eometry

15

57

117

a-l + 1

th.

>0

for

==

1,

, n,

Problems: Arithmetic and Combinatorics


1. Show that 2<3n) + 1 is not divisible by

17, for any integer

0.

>

2.

Let a, b, c be positive integers with the properties: a3 is divisible


by b, b3 is divisible by c, c3 is divisible by a. Show that (a+b+c)13
is divisible by abc.

3.

For every integer n


divisible by 3.
(The symbol

4.

>

2 show that the number

L ( - 1) k (3k ) is

ln/3J

k =O

Lx J denotes the Greatest Integer Function. )

Calculate the sum of all divisors of the form 2x 3 Y (with x, y


of the number N 19 88 - 1 .

>

0)

5 . Show that there do not exist four successive integers whose product

is

1993 less than a perfect square.

6.

Show that there are infinitely many positive integers n such that
each one of the three numbers n - 1 , n , n + 1 can be represented
as the sum of two perfect squares.

7.

Show that the following system of simultaneous equations has no


solution in integers:

x 2 - 3xy + 3y 2 - z 2
-x 2 + 6yz + 2 z 2
x 2 + xy + 8z 2
8.

Solve the following equation in integers x,

9.

If x,

31
44

100.
y:

y, z are integers, at least one of which is 1990, show that


x 2 + y4 + z 6 > xy 2 + y 2 z 3 + xz3

10.

Consider the sequence

xo

0,

x1

1,

for n = 1, 2, 3 , . . . . Define Yn = x ; + 2n+2 . Show that Yn is the


square of an odd integer, for every non-negative integer n .
11.

The sequence (an} is defined recursively by


ao

1,

a1

2,

an

a ;_ _1 +
an-2

1 for n

2, 3, 4,

.. . .

Show that each an is an integer.


12.

Findall functions f mapping non-negative integers into non-negat


ive integers and such that f (! (n )) + f (n) = 2n + 6 for every integer
n

2:

0.

13.

Show: that ln.J3J is a power of 2 for infinitely many natural num


bers n.
(The symbol lx J denotes the Greatest Integer Function. )

14.

Four numbers are randomly chosen from the set { 1, 2, , 3n} (n


is a fixed integer greater than 1). Compute the probability that
the sum of those four numbers is divisible by 3.

15.

For what natural numbers n is it possible to tile the n x n-chess


board with 2 x 2 and 3 x 3-squares?

16.

A triangular prism is a pentahedron whose two parallel faces ( "top


base" and "bottom base" ) are congruent triangles and the remain
ing three faces are parallelograms. We are given four non-coplanar
points in space. How many distinct triangular prisms having the
four given points as vertices are there?

1 7.

Consider the infinite chessboard with squares coloured white and


black, in the usual manner. SupposeS is a set of 1976 squares such
that every two squares in S can be connected by a path consisting
of consecutively adjacent squares. (Two squares are adjacent if
they have a common edge. ) Show that there are at least 494 white
squares inS. Moreover, show that 494 is the exact bound.

Arithmetic a nd Combinatorics
18.

Consider an alphabet consisting of three symbols a, b, c. How many


n-character words with the following properties (1) and (2) can be
composed?

(1) the word should begin and end with an a;


(2) neighbouring positions must be occupied by different symbols.
19.

Nine trucks follow one another, in a line, on a highway. At the


end of a day's ride it turned out that each driver disliked the style
of the driving of the one in front of him. They wish to rearrange
themselves so that, next day, no truck would follow the same truck
that it followed on the first day. How many such rearrangements
are possible?

20.

We are considering paths (Po, Pt. . .. , Pn) of length n over lattice


points in the plane (i.e., points (x, y) with integer coordinates); for
each i, the points Pi-l and Pi are assumed to be adjacent on the
lattice grid. Let F(n) be the number of those paths that begin in
Po = (0, 0) and end in a point Pn lying on the line y = 0. Prove
that F(n) = e:).

Problems: Algebra

21. Determine all real polynomials P (x) of degree not exceeding 5, such
that P(x) + 1 is divisible by (x - 1) 3 and P(x)- 1 is divisible by
(x + 1) 3.
22. Prove that the polynomial xn + 4 factors into the product of two
polynomials of lower degrees with inte_ger coefficients if and only if
n is divisible by 4.
23. Find all natural numbers n for which the polynomial
Pn (x) = x2 n + (x + 1)2 n + 1
is divisible by the trinomial T(x) = x2 + x + 1.
24.

For every positive integer

k show that the polynomial

is divisible by the binomial x 5 + 1 .


25. Find all pairs of real numbers a, b such that the polynomials

have two distinct common real roots.


26. Let

a, x, y, z be real numbers such that


cos x + cos y + cosz
cos(x + y + z)

x + sin y + sinz
= a.
= sinsin(x
+ y +z )

Prove the equality: cos(y + z ) + cos(z + x) + cos(x + y)

= a.

27. If a, b, e are pairwise distinct real numbers, show that the value of
the expression
a-b b-e e-a
-- + -- + -1 + ab 1 + be 1 + ea
is never equal to zero.

Algebra

28. Solve the system of equations:

x + y + xy

19,

y + z + yz 11,

z + x + zx 14.
=

29. Solve the system of equations

X l (X l - 1)
X 2 (X 2 - 1)

X2 - 1
X3 - 1

in real numbers x 1 . . . . , X n
30.

Solve the system of equations

2xy
X 2 + Y 2 + -x + y = 1,
in real numbers x,
31.

y.

Solve the system of equations

x 2 + Y 2 + z 2 2,
in real numbers x, y, z.
=

x + y + z = 2 + xyz

32. Let n 3 be a fixed integer and let a, b, e be fixed real numbers


with a + b + e = 0. Find all n-tuples (x1. . . . , x n ) of real numbers
satisfying the system of simultaneous inequalities

ax i- l + bxi + ex i + l 0 for
where by definition xo = X n , X n+l = x 1 .

1, . . . , n,

33.

Let a,

b, e be the sides of a triangle. Show that


b
e
a
-- + -- + -- < 2.
b+e e+a a+b

34.

Let a,

b, e, d be positive real numbers with abed = 1. Show that


a 2 + b2 + e2 + d 2 + ab + ae + ad + be + bd + ed 10.

Problems

35.

Let a, b be non-negative real numbers with a2 + b2 = 4. Show that

ab -<vIn
-2 1
a+b +2 - and determine when equality holds.
36.

The real numbers ai, bi, ci, di are such that 0 ci ai bi di


and ai + bi = Ci + di for i = 1,2, . . . , n. Prove the inequality
n
n
n
n
+
+
bi
i

ai
C
II
II
II i
II di.
i=l
i=l
i=l
=l

37.

Prove the following inequality for all integers n > 1:


1 + (n + 1)n+l n-1 > 1 + nn n
n +1
n +2

38.

39.

n
n,. ) v' +l
Let n ;::: 9 be an integer. Which one of the numbers (vr.::
and ( v'n + 1) ,;n is greater?
Prove the inequality

C:)
40.

ffn

< 4n for n 1,2,3,. .. .


=

Prove that the inequality

holds for any real numbers a17 a2, ..., ar. Find conditions for
equality.
41.

For a fixed integer n ;::: 1 find the least value of the sum
x2 x3
xn
Xl + _1. + __1_ + + __!!:
n '
3
2
given that XI, .. . , Xn are positive numbers satisfying
1
1
1 + ++-=n.
Xn
Xl X 2

Algebra

42.

On a given segment AD, find points B and C so as to maximize


the product of the lengths of the six segments AB, AC, AD, BC,
BD, CD.

43.

Find all functions j: JR. JR. satisfying the equation


x2 f(x) + f( 1-x) = 2x - x4 for xER

44.

Let A and B be real numbers different from zero. Prove that the
function f ( x)= A sin x + B sin ( J2 x) is not periodic.

45.

Find all monotonic functions j: JR. JR. satisfying the equation


f(4x) - f( 3 x) = 2x for xE JR..

46.

A sequence ao, a1. a 2 , .. . of real numbers different from zero is gen


erated according to the rule: a n+l = (a - 1)/ (2a n ) Show that
it contains infinitely many positive terms and infinitely many neg
ative terms.

47.

Four sequences of real numbers

satisfy the simultaneous recursions

for n =

48.

0, 1, 2, . . . . Suppose there exist integers k, r ;.::: 1 such that

The sequences xo, x1. x2 , . . . and yo, Y l . Y2 , . . . are defined by:


Xo = YO= 1,
Xn+l =

Xn + 2
Xn + 1'

y + 2
Yn+l = --- for n =
2Yn

Show that Yn = X2 " - l for every integer n ;.:::

0.

0, 1, 2 , . . .

10

Problems

49.

Two sequences of integers all a2, a3, . . n and b1, b2, b3, . . are de
fined uniquely by the equality (2 + v'3 ) = an + bn v'3. Compute
(
)
nlim
-+oo an/bn

50.

The sequence (xn) is defined by

1
X1=
2'

2n- 3
Xn= Xn-1 for n=2,3,4, ... .

Prove the inequality

x1 + x2 +
5 1.

+ Xn < 1 for n 1, 2, 3, ... .


=

A sequence of real numbers ao, a1, a2, satisfies the recurrence


! ani = an-1 + an+1 for n = 1 , 2, 3, . . . . Show that an+9=an for
all n.
.

Problems: Geometry

52.

Construct a right triangle ABC with a given hypotenuse


that two of its medians are perpendicular.

53.

Let ABC be a triangle, AC =/:. BC. Assume that the internal bi


sector of angle AC B bisects also t_he angle formed by the altitude
and the median emanating from vertex C. Show that ABC is a
right triangle.

54.

If ABCDEF is a convex hexagon with AB = BC, CD = DE,


EF = FA, prove that the altitudes ( produced ) of triangles BCD,
DEF, FAB , emanating from vertices C, E, A , concur.

55.

Let ABCDEF be a regular hexagon with M and N points on


diagonals CA and CE (respectively) such that AM = CN. If M ,
N and B are collinear, prove that AM = AB.

56.

Let ABC be an acute triangle with altitudes BD and CE. Points


F and G are the feet of perpendiculars BF and CG to line DE.
Prove that EF = DG.

57.

Consider the right triangle ABC with LC = 90 . Let A 1 and B 1


be two points on line AB (produced beyond A and B) such that
AA 1 = AB = BB 1 and let N be the foot of the perpendicular from
A 1 to line B 1 C. Show that the rectangle with sides B 1 C and CN
has area twice as large as the square with side AB.

58.

Let ABC DE be a convex pentagon inscribed in a circle. The dis


tances from A to lines BC, CD, DE, and BE are a, b, c, and d,
respectively. Express d in terms of a , b, c .

59.

Let ABC be an isosceles triangle with base AB. Let U be its


circumcentre and M be the centre of the excircle tangent to side
AB and to sides C A and C B produced. Show that

2 CU < CM < 4CU.

such

12

60.

Problems

The diagonals AC and BD of a convex quadrilateral ABCD inter


sect in E. Let F1 , F2 and F be the areas of triangles ABE, CD E
and quadrilateral ABCD , respectively. Show that

When does equality hold?


61.

Let P1 P2 be a fixed chord (not a diameter) of a circle k. The


tangents to k at P1 and P2 intersect at Ao. Let P be a variable
point on the minor arc P1 P2 . The tangent to k at P intersects lines
AoP1 and AoP2 at A 1 and A 2 , respectively. Determine the position
of P for which the area of triangle A 0 A 1 A 2 is a maximum.

62.

P be a point inside a parallelepiped whose edges have lengths


a, b and c. Show that there is a vertex whose distance from P does
not exceed v'a 2 + b 2 + c 2 .

63.

Do there exist two cubes such that each face of one of them meets
each face of the other one (possibly at an edge or a corner)?

64.

Let A 1 , A 2 , Aa, A 4 be points on the sphere circumscribed about


the regular tetrahedron with edge 1 such that A i Aj < 1 fori =1- j.
Prove that these four points lie on one side of a certain great circle
of the sphere.

Let

Q(

- - cot
2
2

tan

j3
2

7)
2

+ tan -

Solutions: Arithmetic and Combinatorics


Problem 1

Show that 2W> +1 is not divisible by 17, for any integer n

>

0.

Problem 1, Solut ion 1

Assume, to the contrary, that 23" + 1 is divisible by 17 for a certain


1 ( we write just abe for aW>) . Let r be the remainder left by 23"-1
in division by 17. Thus

by our assumption. This implies

r3 +1= (r + 1)(r2- r +1)

( mod 17).

Direct examination of all possible remainders shows that the second fac
tor (r2- r+1) is never 0 ( mod 17); and since 17 is a prime, the .first
factor must be 0 (mod 17), i.e., we haver= -1 ( mod 17). So we have
shown that
23"

-1

17)

(mod

23"

forces

-1

-1

(mod 17).

Descending, we conclude inductively that


23"

-1

( mod 17)

for k = n , n - 1, n - 2, ... , 1, 0. This, however, is a contradiction because


230 = 2

=fi -1

( mod 17).

Problem 1, Solution 2

Fix an n 1. The exponent 3 n is a number of the form 4 k + 1 or 4 k + 3.


If 3 n = 4 k +1, then
23"

and if 3 n

= 24k+l

16k. 2

2(-1)k

( mod 17),

16k

8 (-1)k

( mod 17).

4 k + 3, then
23" = 24k+3

Note, however, that


2(-1)k

+ 1=

-1

for k odd,
3 for k even,

{-7
8(-1)k+1=
9

for k odd,
for k even.

16

Solutions

n
So 23 + 1 is never congruent to 0 (mod 17).
2
Let a, b, c be positive integers with the properties: a3 is divisible by
b, b3 is divisible by c, c3 is divisible by a. Show that (a+ b + c)13 is
divisible by abc.
Problem

2, S o lution 1
The 13-fold product (a+ b + c)13, when multiplied out, splits into 313
summands of the form
Problem

k, m, n

0 integers, k + m + n = 13 .

(1)

It will be enough to show that each of them is divisible by abc. This is


evident when the exponents k, m, n are all positive. So it remains to
consider the case where one of them is zero. Let e.g. n = 0. The product
(1) then becomes
k, m

0 integers, k + m= 13.

(2)

The conditions of the problem imply that a9 is divisible by c and b9 is


divisible by a. Any number of the form (2) can now be represented as the
product of four factors (separated by multiplication dots in the listing
below):
if
k= 13 ' m=O
if 10$k $ 12 , 1$ m$3
if 1$k$9 , 4$m$12 :
if
k=O , m= 13

akbm = a a3 a9 1;
akbm = a b a9(ak-10bm-1);
akbm = a b b3 (ak-1 bm-4);
akbm = b9b b31;

in each case the first factor is divisible by a, the second by b, the third
by c (and the fourth is an integer), and so the product abc is a factor of
akbm. That does the job.
2, S o lution 2
Let p be any prime divisor of the product abc. Write

Problem

a = p01u,

(3)

where a, (3, 7 0 and u, v, w 1 are integers non-divisible by p.


Since a3 is divisible by b, the exponents a and (3 satisfy 3 a (3. Likewise,
3 (3 7 and 3 7 a; and hence 9a 7, 9 (3 a, 9 7 (3.
Let r = min( a, f3, 7 ). We obtain

Arithmetic and Combinatorics

17

The numbers a, b, c are divisible by pr. Thus ( a+ b + c ) 13 is divisible


by p13r, hence by p01+/3+-r, in view of inequality (4). On the other hand,
according to
o: + (3+ 'Y is the exact power in which p enters the prime
factorization of abc. Since p was an arbitrarily chosen prime factor of abc,
we conclude that ( a+ b+ c ) 13 is divisible by abc.
Problem 3
Ln/3J
For every integer > 2 show that the number L (
is divisible by
( The symbol Lx denotes the Greatest Integer Function. )

(3),

3.

nk )
-1
)k
(
k=O 3

Pr oblem 3, So lut ion 1

For any fixed non-negative integer

is valid for

(; )

n, the fundamental Binomial Identity


(1)

eve ry integer j, if we agree that


= 0 for j < 0 and for j

>

n.

(2)

Consider the following three sums:

An =
Bn
Cn =

(- 1)kC:).
(- 1)k(3k':_ 1).
(- 1)k(3k':_2).

(3)
k

Summation limits have not been indicated; we may assume that ranges
from oo to oo That will cause no ambiguity because there are only
finitely many non-zero terms in each of these sums. E.g., in An summa
tion actually spreads from k = 0 to k =
Thus An is exactly the
number defined in the problem statement. We will show that
-

Ln/3J.
(4)
An Bn Cn 0 ( mod 3) for n 3.
( It is only required to show that An= 0 ( mod 3); however, it proves
practical to handle the assertion in this more general version. )
=

2:':

Using equation (1) we derive the following recursion formulas:

18

Solutions

(-1)kC:)+ (-1)k(3k1)
An+ Bn,

Bn+l

)-1)k(n3k+-11)
k
(-1)kck1)+ (-1)k(3k2)
Bn + Cn,

(5)

(6)

and

)-1)k(n3k+-21)
k
(-1)k(3:-2) + (-1)kck3)
2)-1)k(3kn-2)+I)-1)1+1(3ln)
k
(-1)k(3k2)- (- 1)1(;)
(7)
Cn - An .
And since A3 1- 1 = 0, B3 -3, G3 -3, obvious induction justi
fies the claimed relations ( 4).
Cn+l

Remark

It is not hard to derive from


second order for the Ans:

(5), (6), (7) the recurrence equation of the

(8)
An+2 3(An+l An) for n 1
(with the initial data A1 A2 1); we invite the reader to do that.
Readers familiar with linear recurrences may like to work out an explicit
formula (on the basis of the system (5), (6), (7) or of the single equation
(8); compare Problem 10, Solution 3). We now show how to find that
formula by a different method.
=

Problem 3, Solution 2

(3) together with the convention (2) is preserved from Solution


1.Notation
For any complex number we have by the Binomial Theorem
z

Arithmetic and Com binatorics

where

fn(z)

Yn( z)

and hn( z)

19

( () + (6j:3)z3)z6i,
L ((
. 6j-1)+ (6j+2)z3 ) z6j-l '
L ( ( 2)+ (
. 6j- 6j+1)z3) z6j-2 .
J

-1

In particular, if w is any cubic root of


(i.e., a complex number satis
fying w3
then we have for any integer
=

-1),

w6i

1,

j:

w -1 -w2,
w6i-l
j
w6 -2 = w-2- -w
=

'

hence (compare

(3)),

and likewise

Equality (9) thus implies


(1 +w)n =An- Bnw2- Cnw for w3 =-1 .

{1 0)

Setting w = -1 we hence obtain

Bn

An + Cn for n =

1, 2, 3, ....

(11)

Now consider the complex number

!+ !v'3i =cos(7r/3) i sin(7r/3),

1.

also satisfying a -1 , and moreover, a2 a


For w a equalities
(1 0) and
yield
(1 +a)n=An- Bn(a- 1 )- Cna =An- ( !An+ Cn)v'3i.

(11)

{12)

On the complex plane, the numbers 0, and a represent the vertices of


an equilateral triangle, which is completed to a parallelogram (rhombus)

Solutions

20

by the vertex 1 + a. Thus 1 + a =J3 (cos( 71"/6) + i sin( 71"/6)) , and by de


Moivre's Theorem

(1 + a t=3n l2 (cos(

n1r

(6) + i sin(n7r/6)) .

(13)

Comparing the real parts of (12) and (13),

An= 2 3n/2-1 cos(n1r /6);

this is the explicit formula we have promised. If ln/2J =q, we can


rewrite it as
where

for n=2q,
2cos(n7r/6)
2VJcos(n71"
. /6) for n= 2q+ 1.

For each n, Kn is an integer; in particular, K3 =0. Hence A3 = 0; and


for n ;::: 4 we have q;::: 2, so An=3q-l Kn is divisible by 3.
Problem 4

Calculate the sum of all divisors of the form 2z 3Y (with x, y


number N =1988 - 1.

>

0) of the

Problem 4 , Solution 1

The only trouble is to determine the highest powers of 2 and 3 that divide

N . This can be done using the Binomial Theorem:


1 988

( 20- 1)88

c:) - c18) 20 + (8:) 202 - c:6 ) 203 +. . . + (::) 2088

1- 88 20 + (terms divisible by 2 )

(we used the fact that ( 8:)

! 88 87= 22 3 1 1 29); and

(18 + 1 ) 88
88 88
88 3 . . .
88
88 2
88
18
18 + +
18 +
18 +
=
+
88
0
1
3
2
1 + 88 18 + (terms divisible by 3 4 )

() () ()

()
.

()

Since 88 20= 25 5 1 1 and 88 18 = 32 2 4 11, these representations


show that N =1988 - 1 is divisible by 25, but not by 2 6 , and is divisible
by 32, but not by 33.

A rithmetic a nd Combinatorics

21

Consequently, the sum we are about to evaluate equals


s

(x,y): x,y>O
2"' 3Y dividing N

2"' 3Y

xE{l,2,3,4,5}
yE{l,2}
5
2
L::2"'L::3y
x=l y=l
(2 + 4 + 8 + 16 + 32)(3 + 9)
744.

2
Let us inspect the powers of

Problem 4, Solut ion

1 92 = 361 -23,
=

and

19 modulo 26 and modulo 33:


194 (-23)2 = 52 9 17 (mod 64 ),
=

198 172 = 289 33

(1)

(mod 64);

while

(mod 27).
(2)
1 92 = 361 10
The well-known theorem of Euler (sometimes referred to as ge n eralized
Fermat's Theorem) asserts that if a, n are relatively prime natural num
bers, then a<l>(n) 1 (mod n ) , where
=

18. Thus
(mod 64) and 19 1 8 1

In particular, 4>(64) = 32 and 4>(27) =

(mod 27).

Raising the first of these relations to third power and the second one to
fifth power, we get
(mod 64) and

19 90 1

(mod 27);

or - which is exactly the same (mod 64) and

192 19 88

(mod 27).

22

Solutions

Consequently, in view of

(1) and ( 2),

(3)
1988 " 1 (mod 27)
(if 19 88 were 1 (mod 64), the product 198 1988 would be 33 rather than
1 (mod 64); and the second relation of (3) is justified similarly).
On the other hand, equation (1) shows that 19 8 1 (mod 3 2). Besides,
19 1 (mod 9) . If we raise the first relation to power 11 and the second
(mod 64) and

to power 88, we obtain

(mod 3 2) and

1988

(mod

9).

(4)

Statements (3) and (4), combined, show that N is divisible by 3 2 and by


9, but not by 64 or 27. The concluding calculation is done as in Solution
1.
P roblem 4, S o lution 3

The argument of Solution 2 can be carried out without resorting to Eu


ler's Theorem and Euler's Function, in a fashion less sophisticated and
more straightforward. Namely, upon arriving at formulas (1) and ( 2), we
continue a:s follows. Since

332 = (3 2 + 1) 2

3 22 + 2 . 3 2 + 1

(mod 64),

(1)
19 88 = (198 ) 11 3311 = (332)5. 33 33

we obtain from

(mod 64).

And since by ( 2)

193 = 19 2 . 19

10 . 19 = 190 1
=

(mod 27),

we conclude that
(mod 27).
Claims (3) hence result. The remaining portion of the preceding solution
has to be repeated without any changes, yielding the outcome: S = 744.
P roblem 5

Show that there do not exist four successive integers whose product is
1993 less than a perfect square.
P roblem 5, S o lut ion 1

Assume that the equation

x(x + 1) (x + 2)(x + 3) + 1993 = y2

(1)

Arithmetic a nd Combinatoric&

23

is fulfilled for some integers x and y. Examine equation (1) modulo 5.


Either the product x(x + 1)(x + 2)(x + 3) is divisible by 5 or its four
factors leave remainders 1, 2, 3, and 4, in which case the product equals
4 (mod 5).
Anyhow, the expression on the left of (1) is either 3 or 2 (mod 5), and
this is obviously a contradiction because a perfect square y 2 can only be
0, 1, or 4 (mod 5).
P roblem 5, Solution 2
Assume equation (1) and transform the product under examination as
follows:

x(x + 3) (x + 1) (x + 2) = (x 2 + 3x) (x 2 + 3x + 2) = (z - 1) (z + 1) = z 2 - 1,
where we have denoted by z the expression x 2 + 3x + 1; this quadratic
trinomial has the minimum value (over the reals) equal to - 5 /4, and
hence z;::: -1. Equation (1) now takes the form z 2 + 1992 = y 2 , i.e.,
(2)
(y - z)(y + z) 1992.
We see from (2) that z cannot be - 1 ; hence z ;::: 0. We may also assume
(see (1)) that y ;::: 0. So the second factor in equation (2) is non-negative;
=

consequently, both factors must be positive, the second one greater than
the first. Both factors are integers of the same parity; their product
is even, so they both are even. In view of the prime decomposition
1992 = 2 3 3 83, the prime factor 83 must enter y + z and we conclude
that the pair (y - z, y + z) must be one of the following:

(2, 996 ), (4, 498 ), (6, 332), (12, 166 ).


Accordingly, z equals 497, 247, 163, or 77, which means that the product
(x + 1)(x + 2) equals 498, 248, 164, or 78. However, it is easily verified

that no one of these four numbers is equal to the product of two successive
integers. Contradiction ends the proof.
Problem 6

Show that there are infinitely many positive integers n such that each
one of the three numbers n- 1, n , n + 1 can be represented as the sum
of two perfect squares.
Problem 6, Solution 1

Define nk = (2k 2 + 1) 2 for k = 0, 1, 2, . . . .


Then the sequence n1, n2 , n 3 , . .. is strictly increasing and each of its
terms is equal to the sum of two squares:
nk-

1 = (2k2 ) 2 + (2k) 2 ,
nk = (2k 2 + 1) 2 + 0 2 ,
nk + 1 = (2k 2 + 1) 2 + 1 2 .

24

Solutions

Problem 6, Solut ion

Now let n k

= 2m% + 1,

2
where m k

= k (k + 1). It is enough to notice that


n k - 1 = m% + m %,
n k = (k 2 + 2k ) 2 + (k 2 - 1) 2 ,
n k + 1 = (m k + 1) 2 + (m k - 1) 2 .

6, Solut ion 3
Define the sequences a 1 . a 2, a 3 , . .. and

Problem

b 1 . b2 , b3, . . . recursively by

ao = 4, bo = 3,
and notice the equality a% + 2 = 2b% ( easy proof by induction ) . Hence,
if we set n k = a% + 1, we are done because

7
Show that the following system of simultaneous equations has no solution
in integers:
Problem

x 2 - 3xy + 3y2 - z2 = 31
44
-x 2 + 6yz + 2z 2
2
2
100.
x + xy + 8z
7, S olut ion 1
Since the terms x 2 and z2 appear in all the three equations, it is tempting
to apply the method of elimination so as to get rid of them. If we multiply
the first equation by a, the second by b, and the third by c, and add the
resulting equations, we obtain an equation in which the coefficients of
x 2 and z 2 are a - b + c and -a + 2b + 8c, respectively. Setting these
expressions to be zero, we find that e.g. a = 10, b = 9 and c = -1 do the
job, producing the equation
Problem

10

i.e.,

( -3xy + 3y 2 ) + 9

6yz - xy

= 10 3 1 + 9 44 - 100,

y ( -31x + 30y + 54z)

606.

This yields the possible values of I YI: 1, 2, 3, 6, 101, 202, 303, 606.
In a similar manner we can eliminate the terms x 2 and xy, multiplying
the first, the second and the third equation of the system by suitable
factors a, b, c; now we need that a - b + c and - 3a + c ( the coefficients

xy in the resulting equation ) should be zero. When we take


1, b = 4, c = 3, we obtain
(3y 2 - z 2 ) + 4(6yz + 2z 2 ) + 3 8z 2 = 31 + 4 44 + 3 100,

A rithmetic a nd Combinatorics

25

of x2 and
a=

i.e.,

31z 2 + 24yz + (3y 2 - 507) = 0.

Viewing this as a quadratic equation with the unknown


its discriminant:

z, we compute

D = (24y) 2 - 4 31 (3y 2 - 507) = 4(51y 2 + 15717);


then the roots z 1, z2 are: (- 12y ..fDJ4)/3l. Thus 51y 2 + 15717 ought
to be a square number in order that
z 2 be integers. Yet, for the
previously found values of I YI this expression takes values 15768, 15921,
16176, 17553, 535968, 2096721, 4697976, 18744753, no one of which is a
ZI,

perfect square. So the system has no integer solutions.


P roblem 7, Solut ion 2
An astonishingly simple proof results from examination of the two outer
equations modulo 5 ( the middle equation is not needed! ) . Multiplying
the first equation by 8 and adding the third equation we get

9x 2 - 23xy + 24y 2 = 348,

which is

-x 2 + 2xy - y 2 3, i.e., (x - y) 2 2 (mod 5) .


Yet the square of an integer can only be 0, 1 or 4 ( mod 5); the claim
=

follows.

Problem 8

x, y:
x 2 (y - 1) + y 2 (x - 1) = 1.

Solve the following equation in integers

P roblem 8, Solut ion 1

Set x =

u + 1, y = v + 1; the equation becomes


(u + 1) 2 v + (v + 1) 2 u = 1;

equivalently:

u 2 v + 2uv + v + uv 2 + 2uv + u
uv(u + v) + 4uv + (u + v)
uv(u + v + 4) + (u + v + 4)
(u + v + 4) (uv + 1)

1
1 ,
5 ,
5.

26

Solutions

One of the factors must be equal to 5 or -5 and the other to 1 or -1


(respectively). This means that the sum u + v and the product uv have
to satisfy one of the four equation systems:

u +v
uv

1
0

u+v
uv

u+v
uv

-3

u+v
uv

= -5
= -6

-9

-2

Accordingly, the numbers u and v have to be the roots of one of the four
quadratic trinomials:

t 2 + 9t - 2 j

t2 + 5t

-6

The two trinomials in the middle (the second and the third) have no
integer roots. The first one has roots 0, 1, and the last one has roots -6,
1. Thus ( u, v) must be one of these two pairs, up to permutation. Hence
the final outcome: (x, y) = (u + 1, v + 1) must be one of the pairs: (1, 2),
( -5, 2), (2, 1), (2, -5).

2
The symmetric shape of the equation suggests introducing the funda
mental symmetric forms s =x + y and q= xy. The equation, rewritten
as xy (x + y) =x 2 + y 2 + 1, takes the form
Problem 8, S olut ion

sq =s 2 - 2q + 1;

(1)

i.e., (s + 2)q =s 2 + 1. The factor (s + 2) cannot be zero, and division


is admissible:

5
s2 + 1
=s - 2+--.
(2)
s +2
s +2
If this has to be an integer, the denominator s + 2 must be a divisor of
5, which means that s must be one of the numbers -7, -3, -1, 3. For
each of these values of s , the corresponding value of q is computed from
(2) and we arrive at the four possible systems of equations for s x + y,
q = xy:
q=

--

+y
xy

x+y
xy

-7

-10

x+y
xy

-1
2

x +y
xy

-3]

10

(3)

(they correspond, in a certain order, to the four systems obtained in


Solution 1). The numbers x and y must be the roots of the respective

Arithmetic and Combinatorics

27

quadratic trinomial

t 2 + 7t - 10 j

t 2 + 3t - 10 j

t 2 - 3t + 2 .

Of these, only the second and the fourth have integer roots; these are,
respectively,. -5, 2 and 1, 2. So (x, y) is one of the pairs (-5, 2), (2, -5),

(1, 2), (2, 1).

Problem 8, Solut ion 3

Use the symmetric forms s = x + y, q = xy. The resulting relation (1)


can be viewed as a quadratic equation with the unknown s and parameter

q:

s 2 - qs + (1 - 2q) = 0.
Its discriminant equals D = q 2 + 4(2q - 1) = (q + 4) 2 - 20 and produces

the roots

(4)
8 2 = (q - v'D).
One of these roots has to be equal to x + y, an integer. Therefore D
must be the square of an integer: D = d 2 ; d 0. Then
20 = (q + 4) 2 - D = (q + 4 + d) (q + 4 - d),
with both factors of same parity, the first factor greater than the second.
There are only two factorizations of 20 that suit the need: 20 = 10 2
and 20 = (-2) (- 10), giving rise to the equation systems

q + 4 + d = -2
q + 4 + d = 10
and
q + 4 - d = -10,
q+4-d=2
with solutions q = 2, d = 4 in the first .case and q = - 10, d = 4 in the
second. Recall that d = v'D. Thus, in view of (4), the possible values of
s are: 3, -1 ( if q 2) and -3, -7 (if q = - 10) . So we have obtained the
systems of equations (3) from Solution 2. Repeating its final passage we
determine the four integer pairs ( x, y) that make up the solution of the
=

given equation.

Problem 8, So lut ion 4

The technique of inspecting the discriminant can be employed in a yet


more straightforward manner, without introducing the forms s and q.
Suppose a pair (x, y) is a solution. At least one of the integers x, y is
greater than 1; otherwise the left-side expression would be nonpositive.
In view of symmetry we may assume x > 1. Let us look at the given
equation as a quadratic one with respect to variable y,

(5)

28

Solutions

with discriminant
(6)
D = x 4 + 4(x - 1)(x 2 + 1) = x 4 + 4x 3 - 4x 2 + 4x - 4,
which must be a perfect square in order that equation (5) has an integer
root y.
Suppose x > 2. Then the following inequalities hold:
D - (x2 + 2x - 4) 2
2 0(x - 1) > 0,
2
2
D - (x + 2x - 2)
-4(x - 1 ) (x - 2) < 0,
showing that D is strictly comprised between the squares of two skip
consecutive integers x 2 + 2x - 4 and x 2 + 2x - 2. Therefore D has to be
the square of x 2 + 2x - 3. This, however, cannot be the case, since this
last number is of different parity than D (see (6)).
The only possibility that remains is that x 2. Equation (5) then be
comes y 2 + 4y - 5 0; equivalently; (y - 1)(y + 5) 0, and we get y = 1
or y = -5. So (2, 1) and (2, -5) are all pairs of integers ( x ,. y ) with
x > 1, satisfying the equation. Symmetry yields two other pairs (1, 2)
and ( -5, 2); and there are no more- as the argument shows.
=

P roblem 8, Solut ion 5

Assume that the integers x, y satisfy the equation. Its left side is the
sum of two addends, one of which must be 2:: 1 and the other one 0.
Let e.g. y 2 ( x - 1) 2:: 1, x 2 (y - 1) 0. Then x 2::2, y # 0, y 1.
If y = 1, then of course x = 2 (just look at the equation).
Assume y < 0 for the sequel (remember that y 0 has been excluded).
Again let x + y = 8 and rewrite the equation in the form
=

8x 2 - x 3 - x 2 + x 3 - 28x 2 + 8 2 x - x 2 + 28x - 8 2 1;
x(8 + 2) (8 - x) = 8 2 + 1.
The factor 8 - x y is negative; x is positive. Hence 8 + 2 must be
negative, and so 8 -3, whence 8 2 2:: 9.
Rewrite the last equation as f(x) = 0, where by definition
f(x) = [ - (8 + 2) ] x 2 + [ 8(8 + 2) ]x - [8 2 + 1 ] .

Expanding and regrouping,

Notice that the coefficients (in square brackets) are positive. Thus, in
view of x 2:: 2, we get

f(x) 2::! (2)

-4(8 + 2) + 28 (8 + 2) - (8 2 + 1) 8 2 - 9 2::0.
=

(7)

Arithmetic and Combin atorics

29

Equality f (x) = 0 implies that both inequalities in (7) must turn into
equalities. Now, f(x) = ! (2) means that x = 2, while s2 = 9 means that
s = -3. Hence y = s- x = -5. Recalling the case of y = 1 (mentioned
at the beginning), we obtain the two solving pairs (x, y) with y 1:
(2, 1) and (2, -5). Interchanging the roles of x and y we get the other
two pairs: (1, 2) and ( -5, 2) ; and these four pairs constitute the complete
solution.
P roblem 9

If x, y, z are integers, at least one of which is 1990, show that


x2 +y4 +z6 > xy 2 +y2z3 +xz3.

Problem 9, So lut ion

This is in fact the Cauchy-Schwarz inequality for the triple of numbers


x, y2, z3; it can be settled (in the weak form) as follows, using the
arithmetic mean-geometric mean inequality for pairs of numbers:
x2+y4 x2 +z6 y4 +z6
+
+
x2 +y4 +z6

>
=

>

JxY+v'x2z6+
lxly 2 +lxllzl3+ Y21zl3
xy2 +y2z3 +xz3

(because lxl ;:::: x and lz l ;:::: z ).


Equality would require that x2 = y4 = z6 and either y 0, xz ;:::: 0, or
y =/= 0, x, z ;:::: 0. In the first case we get x = y = z 0, in contradiction
to the "1990" condition.
Regarding the second case, we now have z3 = y2 = x > 0. Since x, y,
z have to be integers, z3 = y2 forces that z is itself a perfect square:
z = u2, with u being a positive integer. Thus y = u3, x u6. By
assumption, one of the numbers x = u6, y u3, z = u2 has to be
1990. And since 1990 is neither a square or cube or sixth power, equality
cannot occur and the given inequality holds (in the strict form).
=

P roblem 9, Solut ion 2

The proof can be also derived from the following transformations:


(x2 +y4 +z6) (xy 2 +y2z3 +xz3)
_

z6 (y2 +x)z3 + (y4 xy 2 + x2)


x2 (z3 +y 2 )x+ (z6 y2z3+ y4)
_

(z3- !(Y2 +x)) + i(y2- x)2


(x- !(z3+y2)) + i(z3- y2)2 .

(1)

30

Solutions

These expressions are non-negative. Now, x, y, z are integers, one of


them being equal to 1990. If x = 1990, then y 2 =I x. If y = 1990 or
z = 1990, then z 3 =I y 2. In each case one of the terms (y 2 - x) 2 and
(z 3 - y 2) 2 is strictly positive, and so is the difference expressed by for
mulas (1).
P roblem 10
Consider the sequence xo = 0, x1 = 1,

X n+2 = 3x n+l - 2x n for n = 1, 2, 3, . . . .


Define Yn = x + 2 n+2 . Show that Y n is the square of an odd integer, for
every non-negative integer n .
P roblem 10, S o lut ion 1
The initial X n S are 0, 1, 3, 7, 15,

31, . . . , so it is natural to guess that


(1)
We prove this by induction. For n = 0 and n = 1, (1) holds. Assume
that (1) holds for some two successive integers n and n + 1. Then
3x n+l - 2x n = 3(2 n+l - 1) - 2(2 n - 1)
X n+2
= 3 . 2 n+l - 2 n+ l - 1 = 2 n+2 - 1,
proving (1) for n + 2. Hence, formula ( 1) is true for all integers n 0.
From (1) we get
(2 n - 1) 2 + 2 n+2
Yn = x ; + 2n+2
2 2n - 2 2 n + 1 + 4 2 n
2 2n + 2 2n + 1 = (2 n + 1) 2 ,
showing that Y n is the square of an odd integer, as asserted.
=

10, Solut ion 2


The recursion formula for X n+2 can be rewritten as
P roblem

X n+2 - X n+ l = 2x n+ l - 2x n for n 0, 1 , 2, . . .
Thus, setting X n+l - X n = t n we have t n+l = 2t n for n = 0, 1, 2, . . . ; and
since to = 1, we infer t k = 2 k , i.e.,
(2)
for k = 0, 1, 2, . . . Fix an integer n 1. Summing the equalities (2) over
k = 0, 1, 2, . . , n 1 we obtain
=

Arithmetic a nd Combinatorics

31

or, which is the same (in view of xo

0),

So we have formula (1) of Solution 1 (without guessing), and it remains


just to repeat the last paragraph of that solution.
Problem

10,

Solution 3

We are dealing with the homogeneous linear recursive equation of the


second order

X n +2 - 3x n+l + 2x n = 0 for

n = 0,

1, 2, . . . .

The well-known method of handling such recursions is to solve the char


acteristic equation, which in this case is

q2 - 3q + 2 0,
(3)
and to postulate X n = Aa n + B (3 n , where a and (3 are the roots of that
equation (provided they are distinct). Now, equation (3) has roots a = 2
and (3 = 1, yielding X n = A 2 n + B. From the initial data xo 0, x 1 = 1
=

we get

A + B = 0, 2A + B = 1.
Thus A = 1 and B = -1, i.e., X n 2 n - 1. As in the first solution, we
hence obtain Yn (2 n + 1) 2 .
=

Problem

10,

Solution 4

If one prefers (unwisely enough) to work out a recursive formula for the

Yn S, that is also possible. Squaring the equation that defines X n+2 we

obtain

x;+2 = 9x; + l - 12x n X n+ l + 4x ;,


whence by setting x Yn - 2 n+2 and denoting X n X n+l by Zn :
=

This simplifies to

12zn = -Yn+2 + 9Yn+l + 4 yn - 18 2 n+2 .


Consider

Zn+l :
Zn+l = X n+1 X n+ 2
= X n+l (3x n+l - 2x n )
3x+ l - 2x n X n+l
3(Yn+l - 2 n +3) - 2zn .
=

(4)

Solutions

32

Multiply this by 12 and insert expression


sion for 12z n+l ) :

(4) ( and the analogous expres

-Y n+ 3 + 9y n +2 + 4Yn+l - 18 2 n+3
36(Yn+l - 2 n+3) - 2( -Y n +2 + 9Y n+l + 4yn - 18 2 n+2 ).
The powers of 2 cancel out and we are left with

Apply the method described in Solution


is

3.

The characteristic equation

q 3 - 7q 2 + 14q - 8 = 0 .

Its coefficients sum up to 0 , hence one of the roots is 1 and the equa
tion factors into (q - 1) (q 2 - 6q + 8) = 0. The roots of the quadratic
factor are found e.g. from the Viete's Formulas; they are 2 and 4. So we
postulate
(5)
Y n = A 4n + B 2 n + C.

The initial terms xo = 0 , x 1 = 1 , x 2 = 3 yield the initial terms of the


sequence (Y n }: Yo = 4, Y l = 9, Y 2 = 25 . Setting these in (5) we obtain
the system of linear equations for the constants A, B, C :

A +B+C

= 4,

4A + 2B + C 9,
=

with the unique solution A

Problem

= 1, B = 2, C = 1.

= 25,

Therefore

11

The sequence

(a n } i s defined recursively by

ao = 1 ,

for

Show that each


Problem

16A + 4B + C

11,

n = 2, 3, 4, . . . .

a n is an integer.

Solution

We proceed by induction. The first t hree terms ao = 1 , a1 = 2 and a 2 = 5


are integers. Fix n 3 and assume that the aks are integers for all k :::; n ;
we will show that a n+l i s a n integer also. According t o the defining
formula, a n - 1 (a;_ 2 + 1)fa n -3 i thus
=

Arithm etic a nd Combinatorics

33

The numbers an- 1 . an- 2 , an- 3 are integers, by the inductive assumption.
The last formula shows that an-1 and an- 2 are coprime. Now,

a! + 1

=
and hence

All the aks occurring in this equality are whole numbers. So the prod
uct (a + 1)a_ 2 is divisible by an-1 And since an- 2 and an-1 are
coprime numbers, an-1 has to be a divisor of a + 1. Consequently,
an+l = (a + 1)/an_1 is an integer. This completes the inductive step.
P roblem

1 1 , S o lut ion 2
According to the definition,

Replacing

by

+ 1 we obtain

Subtracting the first equation from the second one,

This shows that the sequence ((an + l + an-1) /an) is constant . It begins
with (a2 + ao)/a1 = (5 + 1)/2 = 3, and hence (an+l + an-d ian = 3 for
all n ; equivalently,

an +1

3 an - an-1 for

n =

1, 2, 3, . . . .

Since ao = 1 and a1 = 2 , this forces that all the ans are integrs.

34

Solutions

Problem

11,

S o lut ion 3

few initial terms of the given sequence are: ao = 1, a1 = 2, a 2 = 5 ,


a 3 = 13 , a 4 = 34, a s = 89; the even-indexed Fibonacci numbers are
immediately recognized. The Fibonacci sequence, defined by the recur
sion

Fo =

1,

F1

1,

Fn = Fn- 1 + Fn-2 for

2 , 3 , 4, .

.. ,

(1)

begins with

( Fo , F1 , F2 , F3 , F4 , Fs , Fs , F7, Fa , . . . )

= (1, 1, 2 , 3, 5 , 8, 13, 2 1 , 34, . . . ) ,

so it is natural to guess that

= 0, 1, 2 , 3, . . . .
1, it will be enough

a n = F2n for

(2)

Since (2) holds for n = 0 and n =


to prove that
the sequence ( F2n ) fulfills the same recursion formula that defines the
sequence (a n ) :

F2n

Fi( n - 1 ) + 1
F2 (n-2 )

for

2,

3, 4, . . .

equivalently,

F2nF2n- 4 - Fin - 2

for

2,

3, 4, . . . .

(3)

The Fibonacci numbers are expressed by the well-known equality


where

1-1+v'5
v'5 , f3 = -= -2
2 .

( Readers not familiar with this expression may like to derive it from
the recursion (1) , employing the techniques described in the solution to
P roblem 10, this book. ) Notice that a + f3 = 1, a - f3 = v'5 , a /3 = -1.
Thus

a4n -2

( a f3) 2n-3 ( a 4 + /3 4 )

+ f3 4n-2

a 4n -2

2 ( a f3) 2n - 1
5

f3 4n-2

Arithmetic a nd Combinatorics

35

4
) 2 + ,8 4
= a - 2(a,B
5
=

(a

- ,8) 2 = 1
'
5

equality (3 ) results, proving our claim (2). It j ust remains to use the fact
that the Fibonacci numbers are integers.
Problem

12

Find all functions f mapping non-negative integers into non-negative


integers and such that f (f ( n)) + f ( n) = 2n + 6 for every integer n 0 .
P roblem

12, Solut ion 1


Suppose f satisfies the given equation

J (f(n)) + f(n) = 2n + 6

n = 0, 1, 2, . . . .
(1)
n, m 0 we get f(f(n)) = j(f(m)),
for

Assuming f(n) = f(m) for some


whence by (1) n = m. Thus f i s injective. Denote:

f(O) = a, f ( a ) = b, f ( b ) = c, f(c ) = d, f ( d ) = e.
Setting in (1) n = 0 , a, b, c we obtain, respectively,

(2 )

b + a = 6, c + b = 2a + 6, d + c = 2b + 6, e + d = 2c + 6. (3)
If a were zero, all the numbers in ( 2 ) would be zero, in contradiction
to b + a = 6. So a =I= 0 , and by injectivity f(a) =I= f(O), i.e. , b =I= a. Since
a + b = 6, we see that a =I= 3.
Subtract the first equation of ( 3 ) from the second, the second from the
third, and the third from the fourth:

c - a = 2a, d - b = 2b - 2a, e - c = 2c - 2b.


By the first equation of

c = 3a,

(3), b = 6 - a.

d = 3b - 2a = 18 - 5a,

(4)

Relations (4) hence imply:

e = 3c - 2b = lla - 12.

(5)

All the values taken by f are non-negative integers; in particular, d 0


and e 0. This in view of (5) shows that :$ a :$ 1f . Since 3 has been
excluded as a possible value of a, we infer a = 2.

Solutions

36

b = 4, and from equations (4)


e = 10. The obvious guess is

Thus

( or

(5))

we compute:

= 6, d = 8,

(6)
f(2k) = 2k + 2 for k = 0, 1, 2, . . . .
This holds for small values of k. Assuming (6) holds for a certain k, we
get from equation (1)
f(2k + 2) = j(f(2k)) = 2(2k) + 6 - f(2k) = 2(k + 1) + 2,
showing that (6) holds with k + 1 i n place of k. S o , equality (6) i s settled
by induction.
Now, let ! (1)

= q.

By equation

(1),

(7)
f(q) + q = 8 .
So q 8. The numbers 2, 4, 6, 8 are values of f at 0, 2, 4, 6, respectively
(see (6)). Injectivity forces that q !(1) must be one of the numbers:
0, 1, 3, 5, 7. We will show t hat 0, 1, 5, and 7 can be easily eliminated.
If q = ! (1) 0 then, by (7) and ( 6 ) , f (O) = f(q) = 8 = ! (6), violating
=

injectivity.
If q = f(1) = 1, contradiction with equation (7) is evident.
If q = !(1) = 5 then, by (7), ! (5) = 3. Setting in equation
n = 5, and then n = 3, we obtain

(1),

first,

! (3) + f (5) = 16 and !(!(3)) + !(3) = 12;


hence f (3) = 16 - ! (5) = 13 and f(f(3)) = 12 - f (3) = - 1, a contradic

7 then, by (7), ! (7) = 1. Equation (1) with n = 7 yields


!(!(7)) + ! (7) = 20,
i.e. , f(f(7)) = 19, in contradiction to !(!(7)) = ! (1) = 7.
We are left with the only possible value f(1) = 3. Induction very similar
to the proof of formula ( 6 ) shows that
(8)
f(2 k + 1) = 2k + 3 for k = 0, 1 , 2, . . . .
Equalities ( 6 ) and (8) jointly result in
f (n) = n + 2 for n = 0, 1 , 2, . . . .
tion again.
If q = f(1) =

and it is readily verified that this function indeed satisfies t he given


equation (1).

Arithmetic and Combinatorics

37

P roblem 12, Solution 2

Choose and fix an integer


non-negative integers:

n 0.

Consider the following sequence of

ao = n, a 1 = f(n), a 2 = J (f(n)), . . .

'

superscript denoting iteration. In equation (1) set


the result is
a k +2 + a k +l = 2a k + 6.
Subtracting

. . . , (9)
f k (n) in place of n;
(10)

2a k+1 from both sides we get

that is,

r k+l + 2r k - 6 = 0,
where r k = a k +l - a k . Write r k = X k + 2; the equation becomes
k = 0, 1, 2, . . . . The last equation obviously
X k = ( -2 ) k xo. Consequently
r k = 2 + (-2) k xo for k = 0, 1 , 2, . . .
By telescoping, we obtain for every integer m 1:
m -1
am
ao + L ( a k +l - a k )
k=O
m- 1
=
ao + L r k
k=O
m- 1
ao + 2m + L (-2) k xo
k=O
m
=
ao + 2m + 1 - (-2)
(11)
xo.
3
Recall that all a m s are supposed t o b e non-negative. The exponential
growth of I ( - 2) m xol can be in no way matched by the linear term 2m, un
less xo 0. ( To be more precise: if xo > 0 then the expression obtained
in (11) is negative for large even m ; and if xo < 0 then it is negative for
large odd m.)
Therefore xo must be 0, whence ro = 2; i.e. , a 1 - ao = 2. This i n view
of definition (9) means that f(n) - n = 2.

All t hese relations hold for


implies the explicit formula

Solutions

38

We have begun by choosing an integer n 0 arbitrarily. The conclusion


is that
f (n) = n + 2 for n = 0, 1, 2, . . . .
Problem 1 2 , S o lution 3

This is j ust a variation of Solution 2. Introduce the sequence of iterates


(9) and write equation (10) . Note that (10) is an inhomogeneous linear
recursion of the second order, with a constant free term.
The method of solving such equations is algorithmic. One postulates a
solution of the form aA, = Ck (ignoring the initial data) . In the case of
equation (10) this yields C = 2; thus the sequence (2k) is a particular
solution of (10).
If ( ak ) is the sequence (9) we are looking for, then the difference

satisfies the homogeneous equation corresponding to

(10):
(12)

This is solved by the standard method (see Problem 10, for example) :
the characteristic equation .>. 2 + >. - 2 0 has roots 1 and - 2, and so
C k = A ( -2) k + B is the general solution of (12). This implies
=

(13)
a k = A(-2) k + B + 2k,
with unknown constants A and B; the explicit evaluation of those con
stants has been carried out in the previous solution, formula (11), in
terms of the data ao and xo ro - 2 a1 - ao - 2. But we do not need
to know their values! Just note that if A # 0 then the term 2k is negligi
ble alongside with A( -2) k , and so ak is negative for k sufficiently large,
even or odd according as A < 0 or A > 0. And since it is required that
ak
f k (n) 0 for all k, we conclude that A = 0. So a k = 2k + B for
k 0, 1, 2, . . . . Hence by definition (9)
f(n) - n = a 1 - a o = (2k + 2 + B) - (2k + B) = 2,
=

and we arrive at the same result as in the two former solutions.


Problem 1 2 , Solut ion 4

The ideas of the Solution 1 and Solutions 2/3 can be neatly combined
to produce a fourth one. Consider the sequence of iterates (9) and their
recursion equation (10):

Arithmetic a nd Co mbinatorics

39

Using this recursion we compute:

a3
a4
a5
as
a7

3a l - 2ao,
6ao - 5a l + 18,
lla 1 - 10ao - 12,
22ao - 21a l + 54,
43a l - 42ao - 72,

(15)

In the Solutions 2 and 3, the sequence (a k } was generated by an arbitrary


initial term ao = n. Now let us take n = 0 and n = 1, and denote the
resulting sequences (a k } by (P k } and (q k }:

P k = f k (O), qk = f k (1) for k = 0, 1, 2 , .


( thus P o = 0, qo = 1 ) . Equalities (15) yield, in particular,
P 5 = llp l - 12,
54 - 21p l ,
P6
qs = 76 - 21%
43q l - 1 14.
q7

These numbers have t o be non-negative. So w e get the two-sided estimates:

12 < p < 54
1 1 - l - 21 '

1 14 < q < 76
43 - l - 21 '

Each one of these intervals contains only one integer, and hence p 1 = 2,
q 1 = 3. Formulas (15) applied to (a k } = (P k } and (a k } = (q k } now pro
duce
PO = 0, Pl = 2, P 2 = 4, P3 = 6 ( and so on )
and

1, q 1 = 3, q2 = 5, q3 = 7 ( and so on ) .
The general rules P k = 2k and q k = 2k + 1 are easily guessed and equally
easily proved by induction, based on the recursion formula ( 14 ) . Restate
qo =

them more explicitly as:

f k (O) = 2k,

/ k (1) = 2k + 1 for k = 0, 1, 2, . . .
This means that the function f acts as follows:
In other words,

f is the function: f (n) = n + 2.

Solutions

40

P roblem 13

Show that Ln v'3J is a power of 2 for infinitely many natural numbers

n.

(The symbol Lx J denotes t he Greatest Integer Function.)


Introductory Remark

There is nothing peculiar about t he number v'3. In fact, it can be re


placed by any other number a with 1 < a < 2. We present three solu
tions to the problem involving the sequence L na J , with an arbitrarily
fixed a E (1, 2), plus a fourth solution in which a is additionally assumed
to be irrational.
So, let us fix an a with 1 < a < 2. Call an integer n nice if Lna J is a
power of 2. We wish to show that there are infinitely many nice ns.
P roblem 13, Solut ion 1

Assume this is not the case. Take an integer k with


nice n. Let q be the (unique) integer such that qa < 2 k
r = 2 k - qa; thus 0 < r ::5 a.
There i s a (unique) integer j 0 for which

Since (by assumption)


obtain

<

2,

the number

a/2

exceeds

2 k > na for all


:::; (q + 1)a. Set

a 1,
-

and we

equivalently,

(2i q + 1 ) a 1 < 2H k :::; (2 i q + 1 ) a.


Denoting 2iq + 1 by m we thus have Lmaj = 2i +k , and hence m i s nice.
Note, however, that m satisfies the inequality
-

which is impossible, according to the definition of k. Contradiction ends


the proof.
P roblem 1 3 , Solut ion 2

Clearly, 1 is nice (so the set of nice numbers is non-empty ) . Choose


any nice number n. The product na represents as na = 2 k + r, k 0 an
integer, r E [0, 1). Consider three cases.
Case 1. 0 :::; r < 1/2 . Then L2na J = 2 k+ 1 , hence 2n is nice.
Case 2. a/2 :::; r < 1. (This case cannot occur for n = 1 ; indeed, if
n = 1, then k = 0 and r = a - 1 < a/2.) Now we have

(2n - 1)a 2 k+1 + (2r - a),


=

41

Arithmetic a nd Combinatoric&

[ 0, 1). Hence, L(2n - 1)aJ = 2 k+ l and so 2n - 1 is nice.


- 1 > n.)
Case 3. 1/2 ::; r < aj2. Define
. '
xJ. 2 + a -2 1 ( 1 -
2J ) or J 0 1 2 )
this is an increasing sequence, starting from xo 1/2 and converging to
aj2. So there exists a ( unique) integer j 1 such that x; - 1 r < x;.
Since r = na - 2 k , we obtain the inequalities

with 2 r - a E
( Note that 2n

!I

a - 1 ( 1 - -.1 - ) < na - 2 k < -1 + -a - 1 ( 1 - -:-1 )


2
2
2J '
2
2J -l =

1
2

- + -equivalent to

2 k+i +l + 2 - a (2i + 1 n - 2j + 1)a < 2 k+i +l + 1.


Denote the number i n parentheses by m . We see that l111 a J
and hence m is nice. Evidently, m > n because l na J
2k .

2 k+i +l ,

In each of the three cases we have found a nice number m greater than
n. It follows that there are infinitely many nice numbers.
Problem

13,

S olut ion

Consider the binary representation of

(O.c1c 2 c3 . . . ) 2 with

1/a :

C k E {0, 1}

k = 1, 2, 3, . . . .

for

(1)

This representation is not unique if 1/a is a dyadic fraction ( e.g. , 3/ 4


can be written either as (0. 11) 2 or as (0. 1011 1 1 . . . ) 2 ) . In such a case,
choose the infinite expansion. Thus, in the sequel we are only considering
representations (1) with infinitely many cks equal to 1.
Choose an index

Recalling that

k for which C k+ l = 1.

According t o

m a n integer,

T k E ( ! , 1] .

k
1 < a < 2, we get

with
Hence

2 k a1

(1),

T k .+ -a1 E (1, 2).

1
m k + 1 < (2 k + 1) a ,
-

42

Solutions

showing that L(mk

+ 1)aJ

2k .

So mk

+ 1 is a nice number.

To distinct ks with ck+l = 1 there correspond distinct m k s (because


mk = ( c 1 c 2 . . . ck ) 2 ). And since ck+l equals 1 for infinitely many ks,
this proves that there are infinitely many nice numbers.
P roblem

13,

S o lut ion 4

Here we assume that a E (1, 2) is an irrational number.


Let b E (2, oo ) be the number determined from the equation

1 1
- + - = 1.
a

(2)

The reasoning will be based on the well-known theorem which says that
if a, b are any positive irrational numbers satisfying equation (2) then
the sets
A=

{ lna J : n E N }

and

B = { l nb J n E N }
:

constitute a partition of the set N of positive integers ; this means that


they are disjoint and their union exhausts all of N. (Reference: D. J. New
man, A Problem Seminar, New York-Heidelberg-Berlin, 1 982; Problem
46, p. 8; Solution, p. 68. )
We will prove that

2k E A

for infinitely many ks;

this is equivalent to the assertion of the problem. And since N = A


is a partition, it suffices to show that

2 k E B then 2 k+i E A for a certain j >


Thus assume 2 k E B. So 2 k = LnbJ for some n E N:
nb = 2 k + r,
r E (0, 1 ) irrational.
if

0.

There exists a (unique) exponent j ;:::: 1 such that 2ir E ( 1 , 2). We claim
that 2 k+i E A . Suppose not ; then 2 k+i E B, i.e. , 2 k+i = Lm bJ for some
m E N:
s E (0, 1 ) irrational.
m b = 2 k+i + s ,
Hence

(2i n - m ) b = (2i +k + 2i r) - (2 k+i + s ) = 2i r - s .


The number on the right side belongs to the interval (0, 2) ; that on the
left is a multiple of b > 2. This is obviously a contradiction. Thus,
indeed, 2 k+i E A ; the proof is complete.

Arithmetic a nd Combinatorics

43

Remark

There exists numbers a > 2, arbitrarily close to 2, such that the set of
integers "nice with respect to a" is finite.
Take for instance a number whose reciprocal has the binary representa
tion

1
-;; = (0.0 1 1 1 . . . . . . ) 2
m
where m 1 and m i > m for each
Consider the product u 2 k (1/a), where k is an integer greater than
-

m. The first binary digit of u after the point is either a zero or a one

followed by a block of mi zeros (for some i). In either case, the "fractional
part" of u satisfies the estimate

Note also that

1 .
(0.0 01) 2 = 21 - 2 m1+ l + 2 m+
3
m
+ (1/a) < 1, and consequently lu + (1/a)J =

-;; <

Hence u - LuJ
luJ . As u
is not an integer, this equality shows that there are no integers in the
interval [u , u + (1/a)]. In other words, there is no integer n satisfying
the inequalities 2 k ::; na ::; 2 k + 1.
This shows that n o power 2 k , with any exponent k > m , i s equal t o the
integer part of any product na. Clearly, a is close to 2 if m is large
enough. The block lengths m 1 , m 2 , m 3 , . . . may form a periodic sequence
or not ; accordingly, a can be made rational or irrational, as we please.
Problem

14
Four numbers are randomly chosen from the set {1, 2, . . . , 3n} (n is a
fixed integer greater than 1). Compute the probability that the sum of
those four numbers is divisible by 3.
Problem

14, Solut ion 1


The sum of four integers is divisible by 3 if and only. if their remainders
modulo 3 constitute one of the following patterns:

(0, 0, 0, 0) ,

(0, 1, 1, 1) ,

(0, 2, 2, 2),

(0, 0, 1, 2) ,

(1, 1, 2, 2)

- up to permutation (in each quadruple) . Enumerate these patterns 1


through 5, in the order as they are listed above. Suppose there are Ni

Solutions

44

four-element subsets of
( for i = 1, 2, 3, 4, 5).

{ 1, 2, . . . , 3n} corresponding to the i-t h pattern

Each residue class (0, 1 o r


{1, 2, . . . , 3n}. Therefore

( mod

3))

i s represented by

numbers in

The probability p we are about to evaluate is equal to the fraction

NI D with numerator N = N 1 + N2 + N3 + N4 + Ns and denomina


tor D = e:) , the number of all four-element subsets in the set under

consideration. It is a matter of a simple calculation to get the outcome


p = N/D = 1/3.
P rob lem 1 4 , Solut ion 2

Let :F be the family of all four-element subsets of {1, 2, . . . , 3n }. For each


C E :F denote by r( C) the remainder the sum of elements of C leaves in
division by 3. Clearly, :F = :Fo U :F1 U :F2 , where
:Fi = {C E :F I r (C) = i} for i = 0, 1, 2.
The probability sought equals

I :F_::.o l..;
P = -:-l.ri=-o-=1 +--:F-:21 '
I :Fl i-+-:I-the symbol i :Fi l denoting the cardinality of family :Fi , i.e. , the number of
sets in that family.
Define the operation c --+ c', acting in {1, 2, . . . , 3n}, by

c' = { c1 + 1

- the cyclic shift

Since

c < 3n,
c = 3n
( mod 3n). To each set C E :F assign the set
c ' = {c' I c E C}.
if
if

C consists of four numbers, i t follows that


r (C) = 0 if and only if r(C ') = 1,
r (C) = 1 if and only if r(C') = 2,
r ( C) = 2 if and only if r(C') = 0.
the assignment C
C' maps :Fo onto :F1 , :F1 onto :F2 ,

Thus
--+
and :F2
onto :Fo ; hence the three families are equipotent ( consist of equally many
members ) : I :Fo l = IF1 I = IF2 I , and so p = 1/3.

A rithmetic a nd Combinatorics

45

Remark

Suppose we choose a k-element set


{ 1 , 2, . . . , 3n }

C from

(k fixed, 1 :::; k :::; 3n) .

Are all values of r(C) equally probable, as in the case of k

= 4?

The method of the second solution yields an affirmative answer to this


question, provided that k is not divisible by 3; indeed, r (C') = r (C) + k
(mod 3) ; the assignment C 1-4 C' maps :Fo onto :F1 or onto :F2 (etc . ) ,
according as k = 1 o r 2 (mod 3) . The argument, however, breaks down
when k is divisible by 3. For instance, the probability that the sum of 3
numbers randomly drawn from { 1 , 2, . . . , 9} is divisible by 3, equals 5/ 14
rather than 1/3.
P roblem 1 5

For what natural numbers n is it possible to tile the


with 2 x 2 and 3 x 3-squares?

n x

n-chessboard

P roblem 1 5 , Solution 1

If n is even, the tiling is trivially possible. Thus let n be odd and suppose
the chessboard has been tiled as described. In each 3 x 3-tile, colour blue
the three cells (unit squares) adj acent to its left edge, colour red the three
cells adjacent to its right edge, and colour green the three cells in the
middle; the 2 x 2-tiles remain uncoloured.
Enumerate the columns (vertical lines) of the board 1 through n. S up
pose there are b i blue cells, 9i green cells, ri red cells and u i uncoloured
cells in the i-th column. Clearly, Ui is even, and the sum b i + 9i + ri + Ui
is equal to n 2 , an odd number. Therefore

bi + 9i + ri = 1

(mod 2)

for

1, . . . , n.

(1)

The right neighbour of a blue cell is a green cell; the right neighbour of
a green cell is a red one. Thus b i = 9 i + l ri + 2 and we restate relations
( 1 ) as

ri +2 + ri +l + ri = 1

(mod 2 )

for

or - which is the same -

ri +3 + ri+2 + ri+l = 1
Subtract (2) from (3) to obtain

(mod 2)

for

. - 2;

1, . . , n

= 0, . . . , n - 3.

(2)

(3)

46

Solutions

In the two leftmost columns of the board there are no red cells ; so
= 0, and relations (4) imply

r1 = r 2

Ti 0
=

( mod 2 ) for i non-divisible by 3.

(5)

In the rightmost column there are no blue or green cells; so b n = 9 n = 0,


whence by (1): T n = 1 ( mod 2 ) . This in view of conditions (5) shows
that n must be divisible by 3.
In conclusion, if the board can be tiled as required, then n is divisible by
2 or 3. The converse implication is obvious.
Problem 1 5 , S o lution 2

A colouring argument can be used in a yet smarter manner. As in So


lution 1 , assume n is odd. Colour all the columns of the board black
and white alternately, in a "zebra" fashion. For n odd, the two outer
columns are coloured alike - say, black; so there are n black cells more
than white ones.
Suppose the tiling is possible. Each 2 x 2-tile covers two white cells and
two black cells. Each 3 x 3-tile covers three white cells and six black
cells, or conversely. The difference between the number of black cells
and the number of white cells covered by a single tile equals 3, -3 or
0. The total difference between the numbers of black and white cells
( in the whole board ) equals n. Thus n is the sum of some threes, some
minus-threes, and some zeros - hence, it is a number divisible by 3.
Conclusion as in Solution 1: a tiling in question is possible if and only if
n is divisible by 2 or by 3.
Remark

The easy "if" part results from "uniform" tilings, using tiles of only one
of the two kinds. It is however worth noticing that if n > 6 is divisible
by 3 or 2, then one can tile the n x n-board actually using at least one
tile of either kind ( the reader may try to show that ) .
Problem 1 6

A triangular prism is a pentahedron whose two parallel faces ( "top base"


and "bottom base" ) are congruent triangles and the remaining three faces
are parallelograms . We are given four non-coplanar points in space. How
many distinct triangular prisms having the four given points as vertices
are there?
Problem 1 6 , Solut ion 1

The four points can be distributed between the two bases of a prism in
two fashions: 3 + 1 or 2 + 2.
First case ( 3 + 1): Choose three points out of four ( this can be done in 4
ways ) ; they span a triangle, which we take for the base of a prism . Link

A rithmetic a nd Combinatorics

47

the fourth point with one of the vertices of that base (3 possibilities) ;
the connecting segment will be a side edge of the prism , which is thereby
fully determined.
Second case (2 + 2): S plit the given set of four points into two pairs
(there are 3 ways to do that) ; label the points in one pair A, B and
those in the other C, D . The points A, B are supposed to lie in one
base of the prism under construction, and C, D in the other. Take one
of the segments AC, AD, BC, CD to be a side edge of the prism (4
possibilities ) ; again, the prism is determined.
Thus we can construct 4 3 12 prisms of the first type and 3 4 = 12
prisms of the second type, and this gives 24 as the final outcome.

Problem

16,

Solution 2

Imagine an arbitrary triangular prism. Any quadruple of its vertices nec


essarily contains at least one pair of points belonging one to the bottom
base, the other one to the top base, and connected by a side edge of the
prism. Hence, if the four given points are to be vertices of a triangular
prism, two of them (call them P, Q) must be the endpoints of a side
edge. The other two points (R and S) cannot be joined by an edge, since
they are not coplanar with P and Q .
There are () = 6 ways t o choose the pair P, Q . This done, we can attach
each of R, S to either P or Q . The resulting pairs of segments will be
edges of the prism:
PR , PS

or

Q R , QS

or

PR, QS

or

PS, Q R.

This defines four possible cases. We claim that in each case the prism is
uniquely determined.
In each one of the first two cases, we have already one base triangle (and
the side edge P Q ) ; translate that triangle by the corresponding vector
(PQ or QP) to get the other base.
Consider the third case, with P Q , PR, QS being edges of the prism.
Complete the p&allelograms P Q R'R and PQ SS' (note that R' =f. S and
S' =f. R) ; the points R' and S' are the remaining two vertices of the prism.
The fourth case is analogous.
We see that, on the total, there exist 6 4 24 triangular prisms with
four vertices in the given points.

Problem

17

Consider the infinite chessboard with squares coloured white and black,
in the usual manner. S uppose S is a set of 1976 squares such that every
two squares in S can be connected by a path consisting of consecutively
adjacent squares. (Two squares are adj acent if they have a common

48

Solutions

edge. ) Show that there are at least 494 white squares in S. Moreover,
show that 494 is the exact bound.
Problem 17, S o lution 1

If every two squares in a certain set can be linked (within that set) by
a path consisting of consecutively adjacent squares, we will say that the
set is connected.
We are going to prove a fact slightly more general than requested:
For any positive integer n, the number of white squares in every con
nected set of n squares is not smaller than (n - 1 )/4.
This is trivially true for n = 1, 2. Fix an integer n > 2 and assume
inductively that the claim holds for all positive integers smaller than n.
Take any connected set S composed o f n squares.
Define the distance between two squares as the minimum number of
edges one has to cross while going from one square to the other, along an
admissible path (within S) . Choose and fix a pair of squares A, B E S
whose distance is a maximum; denote their distance by m . ( Since n > 2,
m > 1.) Thus there exists a path CoC 1 . . . Cm - 1 C m , composed of squares
Ci E S, with Co = A, Cm = B. Remove from S square Cm - 1 together
with those squares adj acent to C m - 1 whose distance from A is exactly
m. Denote by S' the set that remains.

Note that square Cm - 2 has not been removed (its distance from A is
m - 2 and not m ). So we have removed at most four squares, and hence
I S' I = n' n - 4.

One of the squares C m - 1 and C m is white, and these two squares have
been moved from S ; so there is at least one white square in the set S \ S'.
We now show that S' is connected. Choose a square D E S'. There exists
a path EoE 1 . . . E k- 1 E k in the set S, with Eo = A, E k = D ; of course,
k :::; m (by the maximality of m ) .

Squares Eo, E 1 . . . . , E k- 2 obviously belong to S' (the distance from A to


each of them is less than m - 1 , so they have not been removed from
S in the formation of S') ; the question is whether E k- 1 also belongs
to S'. S uppose not. The only removed square distant from A by less
than m is C m - 1 Hence Ek- 1 = C m - 1 and k = m. But then the square
D = Ek , adjacent to Ek- 1 , i.e. , to Cm - 1 , has distance m from A ; and
this means that this square should have been removed - contrary to its
choice (D E S') .
S o every square D E S' can be linked with A within S'. The connect
edness of S' follows and the induction hypothesis applies: there are at
least ( n' - 1 ) / 4 white squares in S'. And since S \ S' contains at least
one white square, we conclude that the number of white squares in S is

A rithmetic a nd Combinatorics

not less than (n' - 1 ) /4 + 1


precisely the induction claim.

49

(n - 4 - 1 ) /4 + 1

(n - 1) /4.

This is

The theorem formulated at the beginning is now proved. For n = 1976


it implies the required bound 494. To see that 494 is optimal, consider a
horizontal 3 x 988 rectangle, with white squares removed from the upper
row and from the lower row (but not from the middle one) . This is a
connected set of 1976 squares, out of which exactly 494 are white.
Problem 17, Solution 2

We will apply another inductive reasoning to prove the fact stated at


the beginning of Solution 1 : if a connected set of n squares has w white
squares, then w (n - 1 ) /4.
Assume this is true for all positive integers smaller than a fixed integer
1. Let S be a connected set of squares.
Choose a white square W E S. From any other square in S a path
(contained in S) leads to W , and this path necessarily passes through
one of the four squares adjacent to W . Consequently, every square in
S \ { W } is accessible from one of those four squares via a route omittipg
W ; i.e., a route contained in S \ { W } . Thus the set S \ { W } is the union
of at most four connected sets. Label these sets St , . . . , S1 (1 $ l$4) .
Suppose
consists of n i squares,
of them being white. According
to the inductive assumption, wi
1 ) / 4 for i = 1, . . . , l . Therefore,
denoting by w the number of white squares in S, we obtain

n>

si

Wi
(ni I

i
L
i=l W

1+

w
>

-1 L: n i
4

i= l

4 (n - 1 ) ,

completing the induction step.


The claim i s proved. For an argument that for
1976 the bound
w
1) /41 = 494 is sharp, see the last paragraph of Solution 1 .

r(n -

Solution s

50

Problem

18

Consider a n alphabet consisting o f three symbols a, b, c. How many


n-character words with the following properties ( 1) and (2) can be com
posed?

(1)

the word should begin and end with an

a;

(2) neighbouring positions must be occupied by different symbols.


Problem

18, Solution 1
Consider words of length n that begin with an a and satisfy condition (2) .
Denote by a n , b n , e n the numbers of such words ending in a, b , c , respec
tively. Attaching an a to a word of length n whose last character is b or
c we obtain an admissible word of length n + 1. Hence a n+l = bn + en .
Likewise, bn + l = a n + en and c n+l = a n + b n . Since the roles of symbols
b and c are symmetric, we infer b n = en , and so

Consequently
i.e.
(3)
The initial ans are:
using (3) :

a 1 = 1, a 2 = 0 ; a few subsequent terms are computed

( a n ) :
( . . . , a 4, a 5 , a 6, a 7, a s, a g, . . . ) = ( . . . , 3, 9, 15, 33, 63, 129, . . . ).
The pattern becomes plain: a n = 2 n -2 + (- 1) n - \ i.e. ,

A "roughly geometric" sequence? Consider

Once guessed, this equality is easily proved by induction. For n = 1


and n = 2 formula (4) gives the correct values, and the inductive step
( (n, n+1) -+ (n+2) ) follows immediately from equality (3) :

an +2 = H2 n-1 + (- 1) n ) + 2 . H2 n- 2 + (-1) n - 1 ) = H2 n + ( -1) n+l ) .


Note that a n is the number we had to calculate. Its value is given by
formula (4) .

A rithmetic a nd Com binatorics

51

Remark

The explicit formula (4) could be derived from (3) without guessing,
by the usual method of solving linear recursions (compare Problem 10,
Solution 3, for instance) .
P roblem 1 8 , Solut ion 2

Assume n 2:: 3. Imagine a row of n empty cells. They have to be filled-in


with symbols a, b, c, observing conditions ( 1 ) and (2). In the two outer
cells, as must be placed; this is prescribed. Assume that character a oc
curs k times inside the row. (Consider k to be fixed, for the while. ) There
remain n - 2 - k cells to be filled with other symbols. The occurrences
of a split those cells into k + 1 blocks of positive lengths {3 1 , . . . , f3 k+ l
(positive, because the as never occur on neighbouring positions) .

There are ( n - -k ) ways to represent the number n - 2 - k as a sum


of k + 1 positive integers n 2 - k = {31 + + f3k+ l (because the sums
f3 t , f3 t +{3 2 , f3 t + fJ2 +f33 , . . . , f3 t + +f3k can constitute an arbitrary
subset of { 1 , . , n-3-k} ) . Every such representation determines the
positions of a. Each of the k + 1 blocks must be filled with bs an d cs
alternately, and that can be done in two ways. Summarizing, there are
a n words satisfying conditions ( 1 ) and (2), where
-

(5)
applying the usual convention that ('; ) = 0 whenever
(cf. the solution to Problem 3}, we may assume that
formula (5) ranges over the set of all integers.
To bring this sum to a closed form, note that

=
=

=
=

(n : k ) 2 k+l

(n - :

) 21+2

2 n - : - l 2 1+1
- l

2a n-1

(for

n 2:: 4) ,

k < 0 or k > m
k in the sum in

Solutions

52

and we arrive at the recurrence formula (3) of Solution 1. For n = 3, 4


the expression (5) yields aa = a4 = 2 . The explicit formula (4) is deduced
in a standard way; see Solution 1 .
Problem 1 9

Nine trucks follow one another, i n a line, o n a highway. A t the end of a


day's ride it turned out that each driver disliked the style of the driving
of the one in front of him. They wish to rearrange themselves so that ,
next day, no truck would follow the same truck that it followed on t he
first day. How many such rearrangements are possible?
P roblem 1 9 , Solution 1

Translated into mathematical terms, the problem is to calculate the num


ber of permutations of the set { 1 , 2, . . . , 9} in which no one of the suc
cessions 12, 23, 34, 45 , 56, 67, 78, 89 appears. Let F (n) be the num
ber of permutations of { 1 , 2, . . . , n } with the analogous property (call
them feasible) . We are going to derive a recurrence formula. Clearly,
F ( 1 ) = F (2) = 1 .
A permutation of
{ 1 , 2, . . . , n, n+ 1 }
arises from a permutation

1r

of
{ 1 , 2 , . , . , n}

by inserting the element "n + 1" to one of n + 1 positions (n - 1 sockets be


tween successive entries, plus two outer positions, one at the beginning
and one at the end) . The arising permutation will be feasible if either
1r is a feasible permutation and the element "n+ 1" is placed on any po
sition except the one immediately after the "n" or 1r is a non-feasible
permutation, feasibility violated by a single forbidden pair k, k + 1 in di
rect succession, and the element "n+ 1" is placed j ust so as to disconnect
that pair.
In the first case there are n possibilities of inserting the element "n+1"
into one of F (n ) feasible permutations 1r of { 1 , 2 , . . . , n } . This yields the
first summand in the recurrence formula ( 1 ) (below) . In the second case
the address of "n+1" is determined (between "k" and "k+ 1" ) , while k
can be any number out of 1 , 2 , . . . , n - 1 ; the permutation 1r with the only
non-separated pair k, k + 1 can be identified with a feasible permutation
of an (n - 1 )-element set, since we can regard the "brick" (k, k + 1) as a
single entity. This yields the second summand in the formula we arrive
at:
(1)
F (n + 1 ) = nF(n) + (n - 1 ) F (n - 1 ) .
Using this formula and knowing the initial data F ( 1 ) = F (2) = 1 we
easily compute F (9)
148329.
=

Arithmetic a nd Com binatorics

Problem

19,

53

Solut ion 2

i = 1, 2, . . . , n , let Pi be the set of those permutations of the set


{1, 2, . . . , n } in which the element "i" appears immediately after "i- 1" .
Note that I Pi l = (n - 1)! ( the number of arrangements of the elements
1, 2, . . . , i-l, i + 1, . . . , n ; the position of "i" is determined ) .
If K is a subset of {2, . . , n } with IKI = k, then
For

I n Pi I = (
iEK

n - k)!

(2)

( such is the number of permutations of {1, 2, . . . , n } \ K; any such per


mutation uniquely generates a permutation that belongs to n pi -

iEK

the elements of K have to be inserted, from smallest to greatest, to the


appropriate places, fully determined ) .

n
= n ! - IU Pi I
i=2

A permutation is feasible if and only if it does not belong to any


Thus
F (n )

By the Inclusion-Exclusion Principle, and in view of equality

Pi

(2),

n
n
= L I Pi l - L I Pi n P; l + . . . + (-1) n-1 1 n Pi I
i= 2
i= 2
i <i
n-1
L:c- 1) k +l L: 1 n Pi I
KC{2, .. ,,n} iEK
k =1
I K I =k

n-1
(n - 1)! L (- 1) k +l n k. k .
k=1
Consequently,
F (n)

n-1
n-1
n-k
n-k
= (n - 1)! L( - 1) k -- .
= n! - (n - 1)! L (-1) k + 1 1
k.
k.1
k =1
k =O

For n = 9 this quantity evaluates to 148329. This is the number sought.

54

Solutions

Problem

20

We are considering paths ( Po, P1 , . . . , Pn ) of length n over lattice points


in the plane (i.e. , points (x, y) with integer coordinates) ; for each i, the
points Pi-1 and Pi are assumed to be adjacent on the lattice grid. Let
F (n ) be the number of those paths that begin in Po = (0, 0) and end in
a point Pn lying on the line y = 0. Prove that F (n) = e:) .
Problem

20,

Solut ion 1

For any integer k , denote by f ( n , k ) the number of paths of length n


(short, n-paths) beginning in (0 , 0) and ending in a point on line y = k .
Thus F (n) = f (n , 0 ) . Every ( n - 1)-path ending either o n line y = k - 1
or on line y = k + 1 can be uniquely extended to an n-path ending on
line y = k . An (n - 1 )-path ending on line y = k admits exactly two such
extensions. Hence follows the recursion formula

1, k - 1) + 2f (n - 1 , k) + f (n - 1 , k + 1 ) .

f (n , k) = f (n -

(1)

There exists only one feasible path of length 0 (both its endpoints coin
ciding with the origin) . So
f (O , k ) =

for k = 0,
for k =f. 0.

(2)

Formulas (2) and (1) generate the table of values of f (n , k):


k=
n=O
1
2
3

-4 - 3 - 2 - 1

0
0
0
0

:
:
:
:
.

0
0
0
1
.

0
1
4
15

0
0
1

1
2

0
1
4
15

0
0
1

0
0
0

0
0
0
0

20
.

. .

Even-numbered rows of Pascal's triangle are readily recognized. Hence


we guess that f (n , k ) = g (n, k ) , where
g (n, k ) =

( )
2n
n+k

for

n, lkl = 0, 1, 2, . . .

(3)

To prove this guess, it will be enough to show that the numbers g (n , k)


obey the same recursion relation as the f (n , k ) s.

With the convention that () = 0 when r is smaller than 0 or greater


than q, the fundamental Binomial Identity (;) = (=D + (q 1 ) holds,
without any restriction, for every natural q and every integer r (co mp are

A rithmetic a nd Combinatorics

55

Problem 3, Solution 1 ) . Using this identity, we calculate

( ) (
) ( )
( ; ) ( ; ) C ;:)
2n
n+k

2n - 1
n+k-1

2n - 1
n+k
n
+
+2
n
2
n
1
n
g (n - 1 , k - 1 ) + 2 g (n - 1 , k) + g (n - 1 , k + 1 ) ;

g (n, k)

and moreover,
g (O, k ) =

for k = 0 ,
for k #- 0 .

(4)
(5)

The recursion formulas ( 1) and ( 4) for .f and g are the same, and so are
the initial values (2) and (5) . So the guess (3) was correct. Hence, in
particular,
F (n) = .f (n, 0) = g (n, 0)

c:)

for

n = 0, 1 , 2, . . . .

20, Solution 2
An n-path (Po , P1 , . . . , Pn ) beginning in Po = (0, 0) can be encoded by
a (2n)-string of zeros and ones ( q , c 2 , . . . , c 2n- 1 , C 2n ) , according to the

Problem

following rules:

if = [1 , 0] ,
if = [0, 1] ,
if Pj-lPj = [- 1 , OJ ,
if Pj - l Pj = [0, - 1] ,

then
then
then
then

c 2j - l = 1 , c 2 j = 0 ;
C 2j - 1 = 1 , c 2j = 1 ;
C 2j-1 = 0, c 2j = 1 ;
c 2j-l = 0, c 2 j 0 .
=

To put this i n words: each couple o f successive symbols (c 2j-l , c 2j ) rep


resents one step on the path; a step east is rendered by the coupling
(10) ; a step north - by ( 1 1) ; a step west - by (01 ) ; a step south - by
(00 ) . And conversely, every such (2n )-string represents a feasible path.
To distinct paths there correspond distinct codes; the coding is bijective.
Suppose a path consists of u steps north, v steps south, and (jointly)
n - u - v steps east or west. Then the endpoint Pn lies on line y = k
if and only if u - v = k ( "horizontal" steps are irrelevant) . The number
of "ones" in the code of such a path equals 2u + (n - u - v ) , i.e. , n + k.
Consequently, the paths ending on line y = 0 are encoded by binary (2n)
strings with exactly n "ones" . As there are exactly e:) such steps, the
result follows.

Solutions

56

P roblem

20,

Solut ion 3

As in Solution 2, we regard a path as consisting of steps north, south,


east and west. A path ends on the line y = 0 if and only if there are
equally as many steps north and south.
Now, look at the polynomial (x 2 + 2 x + 1) n , written in the form

(x 2 + X + X + 1) (x 2 + X + X + 1) (x 2 + X + X + 1)

(6)

n factors ) . Multiplying out, we obtain the sum of terms


with exponents r = 0, 1,
, 2n. Each of these terms results by an
independent choice of one of the four entries from each factor ( the x 2 ,
the "first" x, the "second" x, the 1).
Every such selection induces a feasible path. Namely, let u s agree that :
( product of

xr

if the term selected


from the j-th factor is:
first
second

x2
x
x
1

Paths ending on the line

then the j-th step


is performed:
north,
west ,
east,
south.

y =

0 correspond to those selections in which


The product of the selected terms equals
case it is different from x n . Hence, the
number of paths ending on line y = 0 is equal to the coefficient of x n in
the polynomial ( 6 ) . It remains j ust to notice that

x 2 is used as many times as 1.


x n in that case; in any other

Accordingly, the coefficient of x n equals

e:) .

Thus

F(n) = e:) .

Solutions: Algebra
Problem

21

Determine all real polynomials P ( x) o f degree not exceeding 5, such that


P(x) + 1 is divisible by (x - 1) 3 and P(x) - 1 is divisible by (x + 1) 3 .
Problem

Let

P(x)

2 1 , S olution 1
be as required; then

P(x) + 1 = (x - 1) 3 Q (x), P(x) - 1 = (x + 1) 3 R(x),


with Q ( x) and R ( x) real polynomials of degree 2 ( at most ) . In each of
them the leading coefficient is the same as in P(x) (zero not excluded ) ,
and thus Q(x) = ax 2 + bx + c, R(x) = ax 2 + px + q. The postulated
identities result in P(x) + 1 = (x 3 - 3x 2 + 3x - 1) (ax 2 + bx + c), i.e. ,
P (x)+ 1 = ax 5 + (b-3a)x 4 +(c- 3b+3a)x 3 + ( -3c+3b-a)x 2 + (3c- b);z:- c
and, analogously,

Hence, comparing the coefficients,

b - 3a
c - 3b + 3a
- 3c + 3b - a
3c - b
-c

=
=
=
=
=

p + 3a,
q + 3p + 3a,
3q + 3p + a,
3q + p,
q + 2.

Luckily enough, this system of five equations with five unknowns is quite
easy to solve and yields

a = _ 28 ' b - _ .!!8 ' c = - 1 ,


Thus

P (x)

P - .!!
8>

q = - 1.

is given by any one of the following two equalities:

P(x) = (x - 1) 3 (- i x 2 - x - 1 ) - 1,
P(x) = (x + 1) 3 (- ix 2 + x - 1) + 1.
Each of them produces the final formula P(x) = - ix 5 +
defining the unique polynomial with properties a s needed.

x 3 - 1;x,

58

Solutions

Problem

21,

Solut ion

The reasoning will be based on the well-known fact of algebra: a poly


nomial F(x) is divisible by (x - x 0 ) k if and only if its derivative F'(x) is
divisible by (x - xo) k- 1 , and moreover F(xo) = 0.
Let P(x) be any polynomial of degree at most 5. Then each one of the
statements 1-4 ( below ) is equivalent to the subsequent one ( we write
G (x) IH(x) when G(x) divides H (x)) :
Statement 1 .
and (x + 1) 3 I P(x) This is j ust the condition of the problem.
Statement 2.

(x - 1) 3 1 P(x) + 1

1.

(x - 1) 2 1 P'(x); (x + 1) 2 I P'(x); P(1) + 1 = 0 = P ( -1) - 1.


Statements 1 and 2 are equivalent i n view o f the theorem formulated at
the beginning, applied to F1 (x) = P(x) + 1 and F2 (x) = P (x) - 1.
Statement

3.

(x 2 - 1) 2 I P '(x) ;
Statements 2 and

P(1) = - 1 ;

P(- 1) = 1.

3 are equivalent because (x - 1) 2 and (x + 1) 2 are


coprime polynomials, and hence P' ( x) is divisible by both of them if and
only if it is divisible by their product.
Statement 4 .

P'(x) = Ax 4 - 2Ax 2 + A;
( The degree of P(X) does
Statement

P(1) = - 1 ;

P(- 1) = 1.

not exceed 5 . )

5.

P(x) = !Ax 5 - Ax 3 + Ax + C;

P (1) = - 1 ;

P(-1) = 1.

The equivalence between statements 4 and 5 follows from the deriva


tive/ antiderivative algorithm.
Now, if P (x) satisfies the conditions of Statement 5, then by setting x = 1
and x = - 1 we obtain C + 18 A = -1, and C - 18 A = 1, whence C = 0 ,
5
5
1 5 , and so
A- 8
-

P(x) =

ix 5 + x 3 - 1:x.

Conversely, if P(x) is given by this formula, then the conditions of state


ment 5 are satisfied. And since statement 5 is equivalent to statement 1,
we infer that the polynomial P(x) = - ix 5 + 1x 3 - ';tx i s the unique
solution to the problem.
Problem

22
Prove that the polynomial x n + 4 factors into the product o f two polyno
mials of lower degrees with integer coefficients if and only if n is divisible
by 4.

A lgebra

59

Problem 2 2 , Solut ion 1

Assume

x n + 4 = F(x)G(x),

ai , b;

integers;

0 < k < n,

0<

m < n,

+ m = n.

Then aobo = 4, a k b m = 1, and hence a k = b m = 1. It is convenient to


set a i = 0 for i > k and b; = 0 for j > m. Let a be the least index for
which a a is odd and let {3 be the least index for which bp is odd. Since
a k bm = 1, we have a 5 k and {3 5 m. In the product F (x)G(x), the
coefficient of x a +/3 equals
=

In this expression all summands except a a bp are even numbers ( because


ais are even for i < a and b;s are even for j < {3). So the coefficient ( 1 )
o f x a +/3 i n F(x)G(x) = x n + 4 is odd, which means that a + {3 = n ,
and consequently a = k, {3 = m . Thus, according t o the definition o f a
and {3,
(2)
ao, a , . . . , a k - 1 , bo, b 1 , . . . , b m - 1 are even numbers.
Since aobo = 4, we get ao = bo = 2.
We claim that k = m. Assume the contrary: let k < m, say. Write the
product of F(x) and G(x) in the form
(3)
F(x)G(x) = P(x) + Q (x) + x n ,
where

P(x) = (ao + a 1 x + + a k- 1 X k - 1 ) (bo + b 1 x + + bm - 1X m - 1 ) ,


Q (x)
(a k box k + a k b 1 x k+ 1 + + a k bm - 1X n -1 )
+ (bmaox m + bma 1 x m+1 + + bm a k- 1X n - 1 ) .
In view of property (2) , all coefficients of P(x) are divisible by 4, while
the coefficient of x k in Q (x) equals 2 ( as a k = 1, bo = 2) . Thus the
coefficient of x k in the polynomial (3) is an even integer, non-divisible by
4; and this is a plain contradiction with the equation F (x)G (x) = x n + 4.
This settles the claimed equality k = m. Consequently n = 2k, and there
fore
(4)
F(x)G(x) = x 2 k + 4 > 0.

It follows that F ( x) and G ( x) are real polynomials without real roots,


hence of an even degree. So k is an even number, i.e. , n is divisible by 4.

60

Solutions

And conversely, if n

= 4l, l E N, then

is the desired factorization.


P roblem

22,

Solution

The equation x n = -4 has n distinct complex roots z 1 , . . . , z n , each of


an absolute value of 4 1 / n . If x n + 4 factors into a product F(x)G (x),
then each o f zt, . . . , Z n must b e a root of either F(x) o r G (x). Assum
z 1 , . . , Z k are roots of F(x) and z 1 , . . . , Zn -k are roots of G (x ) ; then
.

F(x) = A (x - z 1 ) (x - z k ), G(x) = B (x - Z k+ t ) (x - Zn ) ,
with non-zero constants A and B, satisfying A B = 1. The factors F ( x)
and G ( x) are assumed to have positive degrees and integer coefficients.
So 0 < k < "n and A = B = 1. The free terms of F (x) and G (x), equal
respectively to (-1) k Az 1 z k and (-1) n -k Az k+ l z n , should also be
whole numbers. Their absolute values 4 k / n and 4 ( n -k ) j n are comprised
strictly between 1 and 4 (as 0 < k < n) , and their product is equal to 4.
Hence, each of them equals 2, which means that k n/2, and we arrive
at the inequality ( 4) of Solution 1. Conclusion as before.
=

Problem

23
Find all natural numbers n for which the polynomial

Pn (x) = x 2n + (x + 1) 2n + 1
is divisible by the trinomial T (x) = x 2 + x + 1 .
Problem

23,

Solution

Note that T (x - 1) x 2 - x + 1 , and so


x 6 - 1 = (x - 1)(x 2 + x + 1) (x + 1) (x 2 - x + 1) = (x 2 - 1) T (x) T( x - 1).
=

Replacing

x by x + 1,
(x + 1) 6 - 1 = (x 2 + 2x) T (x + 1) T (x).

Therefore the difference

x 2n+6 + (x + 1) 2n+6 x 2n (x + 1) 2 n
x 2n (x 6 - 1) + (x + 1) 2n ( (x + 1) 6 - 1)
_

T( x ) , for every integer n 2: 0. Since


Po(x) 3, P1 (x ) = 2T (x) , P2 (x ) = 2T(x ) 2 ,

is divisible by

Algebra

61

obvious induction shows that


is non-divisible by 3.

Pn (x) is divisible by T(x)

if and only if n

Problem 2 3 , Solut ion 2

A polynomial
roots of T ( ) ,

P(x)

is divisible by

a = - ! + ! v'3 i

T(x)

if and only if the two complex

{3 = - ! - ! v'3i,

and

are also roots of P(x). These numbers satisfy the equalities:

Consequently, each of the two numbers

and

Pn (f3) = (f3 2 f + ((/3 + 1) 2 f + 1


an + {3 n + 1.

equals j ust
V'lriting a natural number n in the form n
2, we obtain

=
=
=

Conclusion:

= 3 k + r , with r being 0 , 1 or

Pn (f3)
a 3k +r + {3 3k+r + 1
a r + {3 r + 1

{3
0

if r
if r

0,

or r

= 2.

T(x) divides Pn (x) if and only if r . 0

(mod 3) .

P roblem 24

For every positive integer k show that the polynomial

is divisible by the binomial


Problem 24, Solut ion

x 5 + 1.

Write for brevity

p(x) = x 3 - x 2 + x - 1,

q(x) = x 4 - p(x)

and notice that

x 4 - 1 = (x + 1)p(x),

x 5 + 1 = ( x + 1)q(x),

(1)

62

Solutions

and thus

The claim will be proved by induction. For k =

1,

{x 4 - 1) {x 3 - x 2 + x - 1 ) + (x + 1)x 3
x 7 - x 6 + x 5 + x 2 - x + 1 = (x 2 - x + 1) { x 5 + 1) .
It is evident from (1) and (2) that the divisibility of Pk (x) by x 5 + 1
P1 (x)

is

equivalent to the divisibility of the polynomial

by

q(x ) .

And since, by the definition of q(x ) ,

p(x) k+2 + x 4k +3
p(x) k+2 + (p(x) + q(x))x 4 k - 1
p(x)Q k (x) + q(x)x 4 k - I ,
Q k +l (x) is divisible by q(x) whenever Q k (x) is.
Q k + l (x)

we see that
pletes induction.
P roblem

Let

(1),

p(x)

24, Solut ion 2


and q(x) have the same meaning as in Solution

This com

1.

In view of

(x 4 - 1 ) q(x) = (x + 1)p(x)q(x) = (x 5 + 1)p(x).


The k t h power o f p(x) = x 4 - q(x) equals ( by the Binomial Theorem )
x4 k plus the sum of terms divisible by q(x). Thus
where R k (x) is a polynomial. Using these equalities together with (2)
we obtain

Pk (x)

=
=

showing that

{x 4 - 1) [q(x)R k (x) + x 4k ] + (x + 1 ) x 4 k - l
(x 4 - 1)q(x)R k (x) + x 4k - l (x 5 - x) + (x + 1 ) x 4k-l
(x 5 + 1 ) p(x )R k (x) + x 4k - l (x 5 + 1 ) ,

Pk (x) is divisible by x 5 + 1.

Problem

2 4 , Solut ion 3
It will be enough to show that each complex root of x 5
of Pk (x) . Obviously, xo = - 1 is a root of Pk (x) .

+ 1 is also a root

Algebra

63

Let now .A be any non-real root of x5 + 1 Then by the second equality


of ( 1 ) , q(.A) = 0, and hence p(.A) = .A\ by the definition of q(x) . This
inserted into equation (2) results in
.

j ust as needed.
Problem

25
Find all pairs o f real numbers

a, b such that the polynomials

P (x) = x 4 + 2ax 2 + 4bx + a 2 and Q (x) = x 3 + ax + b


have two distinct common real roots.
Problem

25, S o lut ion 1


Suppose x 1 and x 2 are real roots of P(x) and
also the roots of

T(x)

P(x) - xQ (x)

Q (x) (x 1 =/= x 2 ) . They are

ax 2 + 3bx + a 2

Thus a =/= 0 and the discriminant D of the t rinomial


tive: D 9b 2 - 4a3 > 0. By Viete's Formulas,

T (x) must be posi

X l + X 2 = -3bja,
Since, by assumption,
0

x 1 =/= x 2 and Q (x l )

Q (x 2 ) = 0, we obtain

Xl - X2
x - x + ax1 - ax 2
=
X l - X2
x 21 + x 1 x 2 + x 22 + a
(x l + X 2 ) 2 - X l X 2 + a
(-3bfa) 2 .
=

So b = 0 and 0 < D
And conversely, if b

= -4a3, i.e. , a < 0.


0 > a, then the polynomials

have the common roots and


those of the form (a, 0) with a < 0.

- Thus the pairs sought are

64

Solutions

Problem 2 5 , S o lut ion 2

The derivative of P (x) equals 4Q (x) . Thus if x 1 and x 2 are real roots
of both P(x) and Q (x ) , then they are double roots of P(x ) , and conse
quently
Therefore 2x 1 + 2x 2 = 0, xix = a 2 , and we conclude that one of the
numbers xb x 2 must be equal to vial and the other to - vial (the
condition x 1 =I= x 2 implies a =I= 0) . Now,
Comparing this with the definition of P (x) we see that b = 0 and
a = - la l < 0,
and so the pair (a, b) must be of the form (a, 0) with a < 0. Conversely,
every such pair satisfies the demands of the problem (see Solution 1 ) .
Problem 26

Let a, x, y, z be real numbers such that


+ y+
cos(x + y + z)

COS X

COS

COS Z

sin x + sin y + sin z


= a.
sin(x + y + z)

Prove the equality: cos(y + z) + cos(z + x) + cos(x + y) = a.


Problem 2 6 , Solut ion 1

In what follows, all sums are cyclic over the triple (x, y , z) (thus, e.g. ,
the symbol I: sin x denotes the sum sin x + sin y + sin z, etc.) . Write
w = x + y + z. Then, by assumption,

2: cos x = a cos w ,

L sin x = a sin w .

Hence

2: cos(y + z)

L cos(w - x)
L (cos w cos x + sin w sin x)
(cos w) 2: cos x + (sin w ) 2:: sin x
a cos 2 w + a sin 2 w = a.

A lgebra

65

26, S olut io n 2
Using the Euler Formulas

P roblem

cos x =

eix _ e-ix

sin x = ----

2i

we restate the assumptions in the form


Adding and subtracting these two equalities, we obtain two new ones:

"'"' e -i:z: = ae -iw .


L...,

Consequently,

2a

1 - e -ix
-.e1-w L e ix + -.
e-w L
2: e i(x-w> + 2: e i(w-x)
L (ei(w-x) + e-i(w-x} )
2 I: cos (w - x ) ,

and the claim results.


Problem 27
If a, b, c are pairwise distinct real numbers, show that the value of the
expression

b - e + -a - b + -c-a
1 + ab 1 + be 1 + ca

--

is never equal to zero.


P roblem 27, Solution 1
Multiply the given expression by the product of the three denominators
and denote the resulting expression by F(a, b, c):

F(a, b, c)

(a - b )(1 + bc) (1 + ca) +


+(b - c) (1 + ca)(1 + ab) + (c - a) (1 + ab) (1 + be). (1)
We have to show that if a =/= b =/= c =/= a, then F(a, b, c) =/= 0; and this fol
=

lows directly from the transformation:

F(a, b, c) = (a - b) + (a 2 - b 2 )c + (ca - bc)abc


+(b - c) + (b2 - c 2 )a + (ab - ca)abc
+(c - a) + (c 2 - a2 )b + (be - ab)abc
ca 2 - b 2 c + ab 2 - c 2 a + bc 2 - a 2 b
=
(a - b) (b - c) (c - a).
=

Solutions

66

Problem 27, S olution 2

Consider a and b to be fixed and replace c in (1) by a variable x :


F (a, b, x ) = (a - b) ( 1 + bx) (1 + ax) + (b - x ) ( 1 + ax) ( 1 + ab)
+(x - a) (1 + ab) (1 + bx) . (2)
For a, b fixed, (2) is a quadratic polynomial in x. The coefficient of x 2
in (2) equals
(a - b)ba - a(1 + ab) + (1 + ab)b;
this simplifies to b - a =f:. 0, showing that the polynomial ( 2 ) is not equal
identically to zero. Setting in (2) x = a and x = b we get value 0 (easy
verification). A non-zero polynomial of degree 2 cannot have a third
root. Since a, b, c are three distinct numbers, we infer that the value of
(2) for x = c is different from zero; i.e., F (a, b, c) =f:. 0, as needed.
Problem 27, S o lut ion 3

Let a = tan a , b = tan ,B, c tan -y with a , ,B, -y E (-7r/2 , 7r/2) . Since a,
b, c are pairwise distinct, so are a , ,8 , 'Y From the equality
=

tan(a - ,8 ) =

tan a - tan ,B
a-b
=
1 + tan a tan ,B 1 + ab

we see that the expression defined in the problem statement is equal to


(3)

tan(a - ,B) + tan(,B - -y ) + tan('Y - a ) .

In the identity tan u + tan v (1 tan u tan v) tan(u + v) set for u and
v the differences ,B - 'Y and 'Y - a; the sum (3) is seen to be equal to
=

tan(a - ,B) + (1 - tan(,B - -y ) tan( 'Y - a) ) tan(,B - a) ,


simplifying to

(4)
tan(,B - -y ) tan('Y - a) tan( a - ,B) .
The (distinct) numbers a, ,B, 'Y lie in ( /2, /2) ; so the numbers ,B - 'Y ,
'Y - a, a - ,B lie there, too, and are different from zero. It follows that
the factors of the product (4) are different from zero, and this is j ust
what we need to conclude the proof.
-1r

1r

Problem 2 8

Solve the system of equations:


x + y + xy

19,

y + z + yz

1 1,

z + x + zx

14.

67

Algebra

28, So lut ion 1


From the first and the second equation,
Problem

x(y + 1) = 19 - y,

z(y + 1) = 11 - y.

Evidently, y cannot be -1, so we may divide by y + 1:

19 - y ,
x = -y + 1

11 - y .
z = -y+1

Substitution into the third equation of the system yields

19 - y + -11 - y + (19 - y)(11 - y) - 14 ,


-2
y+1

y+1

(y + 1)

which is equivalent to

14(y + 1) 2 - (30 - 2y) (y + 1) - (19 - y)(11 - y) = 0,


simplifying to y 2 + 2y - 15 = 0. The solutions of this quadratic are
y 1 = 3 and Y = -5; the corresponding values of x and z are computed
2

from the previous formulas. So there are exactly two solution triples
(x, y, z): (4, 3, 2) and ( 6 5 4 )
Problem 28, Solution 2
The system is equivalent to
-

(x + 1)(y + 1) = 20,

(y + 1)(z + 1) = 12,

(z + 1)(x + 1) = 15.
The sums x + 1, y + 1, z + 1 must be different from 0. Dividing the
first equation by the second we obtain (x + 1)/(z + 1) = 5/3, i.e.,
x + 1 = i (z + 1). Inserting this into the third equation, (z + 1) 2 = 9.
Thus z = 2 or z = -4; accordingly, x = i (z + 1) - 1 equals 4 or -6,
and y is computed from any one of the first two equations of the system.
Outcomes as in Solution 1.
28, Solution 3
Recast the equation system into the form as in Solution 2. Multiply these
equations to obtain
Problem

so the product P = (x + 1)(y + 1) (z + 1) is equal to either 60 or 6 0


Now,
p -1
p
x = (x + 1) - 1 =
-1=-

(y + 1) ( z + 1)

12

68

Solutions

and analogously

p - 1,
p
= 15
z=20 - 1.
Setting P = 60 and P = -60 yields the two triples (x, y, z )
( -6, - 5 , -4) .
y

( 4, 3, 2) and

29
Solve the system of equations

P roblem

Xt(Xt - 1)
X2(X2 - 1)

=
=

X2 - 1
X3 - 1

xt, . . . , Xn .

in real numbers
P roblem 29, S o lut ion 1
Let
be a solution. Note that
1 =?
Assume
= +
::; 0 for a certain io. Then
and all sub
sequent ( cyclically )
are in particular,
a contradiction.
Thus all
must be positive. Now the system implies that all the
differences
are simultaneously positive, negative or zero. In the
first two cases we multiply all the equations and cancel the non-zero
( which appears on both sides ) , with the result that
product
n =
This however contradicts the fact that all
are greater than
or they are all smaller than
The only possibility that remains is that
for all i. Clearly, this is
a solution.
Problem 29, Solution 2
Adding all the equations leads to

Xio

(x1, . . . , x n )

XiS

XiS
Xi - 1
TI(xi - 1)
Xi 1.

Xi
Xi+t 1.
Xio+l 1 Xi0(xi0 - 1) 1,
1;
Xio 1,
XiS

1.

Xi = 1

n
n
x - xi) = L (xi - 1);
)
i=l
i=l
equivalently,
n
(x - 2xi + 1) 0,
L
i=l
i.e.,
n
(x - 1) 2 = 0.
L
i=l
Thus xt = = X n 1.
=

69

Algebra

30
Solve the system of equations

P roblem

2xy = 1,
2 + y 2 + -x+y
in real numbers x, y.
X

30, S o lut ion 1


If x, y are a solution, then clearly x + y
get from the first equation
Problem

2xy
= X 2 + y 2 + -x+y

>

>

0. Assuming x + y > 1, we

x 2 + y 2 + -2xy = (x + y) 2
x+y x+y
x+y

= X

+ y > 1,

a contradiction. A similar contradiction is yielded by assuming that

x + y < 1 ( the inequalities have to be reversed ) . Thus x + y must be 1,


and the second equation of the system becomes 1 = x 2 - (1 - x ) , with
roots x = 1 and x = -2 . So the system has two solutions ( x, y); these
are: (1, 0) and ( - 2 , 3).
30, S o lut ion 2
Multiply the first equation by (x + y ):
Problem

(x 2 + y 2 ) (x + y) + 2xy = x + y.
Adding x 2 + y 2 to both sides of this equation, we are driven by standard
manipulations to a nice factorization:

(x 2 + y 2 ) (x + y) + (x + y) 2 = (x 2 + y 2 ) + (x + y);
(x 2 + y 2 ) (x + y - 1) + (x + y)(x + y - 1) = 0;
(x 2 + y 2 + x + y) (x + y - 1) = 0.
The first factor cannot be zero because the sum x + y is positive ( this is
obviously implied by the system ) . So x + y - 1 must be zero. Inserting
y = 1 - x into the second equation of the system we find the two solutions
(x, y) = (1, 0) and (x, y) = ( 2 , 3)
-

31
Solve the system of equations

Problem

x 2 + Y2 + z 2 = 2 ,
in real numbers x, y, z.

x + y + z = 2 + xyz

70

Solutions

3 1 , S o lution 1
Two equations and three unknowns? This is a clear indication that there
must be some inequality hidden behind the problem statement.
Suppose x, y, z satisfy the system. Write
Problem

= yz,

zx,

s = x + y + z, q = xyz.
From the first equation of the system we get x 2 + (y - z) 2 = 2 - 2 u .
u

v=

= xy,

The left-side expression is a non-negative number. Hence u 1; analo


gously, v, w 1 , and therefore

( 1 - u )( 1 - v ) ( 1 - w ) 0.

(1)

By the definition of s, u , v , w ,

s 2 = (x + y + z) 2 = 2 + 2(u + v + w ) ,
and so u + v + w = !s 2 - 1. Moreover, vw + wu + uv = sq and
uvw = q 2 Thus we can transform the left side of ( 1 ) as follows:
( 1 - u ) ( 1 - v ) ( 1 - w ) 1 - ( u + v + w ) + ( vw + wu + uv ) - uvw
= 1 - ( !s 2 - 1 ) + sq - q 2 = 2 - !(s - q) 2 - h 2 .
=

According to ( 1 ) , this is a non-negative number. Hence follows the in


equality

(2)

Therefore Is - ql 2, while the second equation of the system says that


- q = 2. This means that equality must hold in (2).
Now, the right inequality in (2) arose from q 2 0; so it turns into equal
ity only if one of the numbers x, y, z equals 0. Let e.g. z = 0; then
u = v = 0 and x + y = = + q = 2. The left inequality in (2) comes
from condition ( 1 ) ; equality in ( 1 ) , combined with z = 0, implies w = 1 ,
i.e., xy = 1. The unique solution o f the system x + y = 2, xy = 1 is
8

x = y = l.

This yields the Solution 1 triple (x, y, z) = ( 1, 1, 0 ) ( the only one with
z = 0 ) . By symmetry, ( 1, 0, 1 ) and ( 0, 1 , 1 ) are two other solution triples;
and there are no others.
31, Solution 2
Readers familiar with multivariable calculus can regard this as a maxi
mization problem: inspect the extrema of
Problem

f(x, y, z) = x + y + z - xyz,

Algebra

71

given that x 2 + y 2 + z 2 2. The last equation describes a sphere, hence


a compact set. So a maximum must be attained at some point(s) of this
sphere. By the Lagrange Muliplier Theorem, any extremum point is a
critical point of the Lagrange function
=

g(x, y, z) = f(x, y, z) - A(x 2 + y 2 + z 2 - 2) ;


i.e. , a point such that agjax = agjay = agjaz 0. Thus, at an ex
=

tremum point:

1 - yz = 2Ax,

1 - zx = 2Ay,

1 - xy = 2Az.

(3)

Multiplying the first of these equations by x, the second by y and the


third by z, we get

xyz = x - 2Ax 2 = y - 2Ay 2 = z - 2Az 2


(4)
For any fixed value of A, the function 1/; (t) = t - 2At 2 can take the value
xyz at two distinct points, at most; so the system (4) forces that two of
x, y, z must be equal. Let e.g. x = y. Then the system (3), accompanied

by the equation of the sphere, becomes

1 - x 2 = 2Az, 2 - 2x 2 = z 2 .
(5)
The second and the third equation of (5) result in z 2 = 4Az. So we
have either z 0 (yielding x = y 1) or A = !z, which inserted into
the first equation gives 3xz 2. This together with the last equation
of (5) is solved in a routine way. Outcomes: x = y = !z = i v'3 and
x = y = z = i v'B.
Now we have the complete list of points (x, y, z) at which f(x, y, z) might
1 - xz = 2Ax,
=

be a maximum (up to permutation of :variables) :

0) ,
with = 1. Comparing the values of f at these points we find out that
2 is the maximum value of f on the sphere in question, attained only
at (1, 1, 0), (1, 0, 1) and (0, 1, 1 ) . Hence, these three triples are the only
solutions (x, y, z) of the system under consideration.
, ,

Prob lem 32

Let n 2: 3 be a fixed integer and let a, b, c be fixed real numbers with


a + b + c = 0. Find all n-tuples (xt , . . . , x n ) of real - numbers satisfying

the system of simultaneous inequalities

axi - l + bxi + cxi+ l 2: 0 for i = 1, . . . , n,


where by definition xo X n 7 X n+l = x1.
=

72

Solutions

Problem 32, Solut ion 1

Suppose (x1, . . . , x n ) is a solution. Let s = x 1 + + X n The left side


expressions of all the given inequalities are non-negative numbers and
their sum equals as + bs + cs = 0. Thus all those numbers are equal to
zero and the inequalities of the system are in fact equations:

axi - l + bxi + cxi+ l = 0 for i = 1, . . . , n .


If a = b = c = 0 , then every n-tuple (xt , . . . , x n ) i s a solution.

(1)

Rejecting this trivial case from further considerations, assume that

a 2 + b 2 + c 2 > 0.
Then at least two of the numbers a, b, c must be different from zero.
Hence a 2 + c 2 > 0.
Evidently, every constant n-tuple x 1 = = X n satisfies the system (1) .

Let us look for non-constant solutions.


Since c = - (a + b) , equation (1) rewrites as

(2)
which is further transformed into successively equivalent forms (each
equation is valid for all i):

a(Xi+ l - Xi + Xi - Xi- l ) + b(xi+ l - Xi) = 0;


(a + b) (xi + l - Xi) + a(xi - Xi - l ) = O
a(x i - Xi - d = c(xi + l - xi) (3)
Set Yi = Xi - Xi-l Clearly, Yl + + Yn = 0. Since not all the x i s are
equal, there exists a Yi a =I 0. Equation (3) says that
(4)
This has to hold for all i. Since a2 + c 2 > 0, equation ( 4) can be solved
either for Yi +l or for Yi ( Yi+ l = (a/c)yi or Yi = (cja) Yi+ l ) If any one of
the numbers (y 1 , . . . , Yn ) were zero, we could infer (inducting forward or
backward) that all the YiS are zero, in contradiction to Yia =I 0. Conse
quently the product p = Yl Yn is different from zero. Multiplying the
n equations (4) we obtain a np = c np, and hence a n = e n .
If the numbers a and c were equal, equations ( 4) would force that all the
being zero and product
YiS are equal; and this is impossible, their sum
non-zero. Therefore a =I c. The equality a n = e n then implies that n
is an even number and c = -a ( =I 0) . Hence b = - (a + c) = 0. We go
'

back to the system (1) , which becomes simply

Xi-l - Xi + l = 0 for i = 1, . . . , n .

(5)

Algebra

73

This means that the even-indexed x i s must be equal and the odd-indexed
x i s must be equal; if they are, system (5) is satisfied.
So we can formulate the answer:
If a = b = c 0, the XiS can be arbitrary real numbers.
If b 0, a = -c =f:. 0 and is even, the general solution of the system (1)
is ( x t . . . . , x n ) = (u , v, u , v, . . . , u , v) with u , v arbitrary real numbers.
In all the other cases, the general solution of the system (1) is

( x t . . . . , X n ) = ( t, . . . , t),
with t an arbitrary real number.
Problem 32, Solut ion 2

Begin as in Solution 1 and reduce the given system of inequalities to the


system of equations (1).
Assume ab =f:. 0. Rewrite the ith equation of (1) in the form (2):
Squaring yields

a 2 x i2_1 + 2 a bXi -1Xi + b 2 x i2 = a 2 x i2+ 1 + 2 abx i2+ 1 + b 2 x i+2 1 .


This holds for i = 1,
Adding these equations we obtain
n
n
n
(a 2 + b 2 ) I.> + 2ab L X i-1Xi (a 2 + b 2 + 2ab) L: x ,
. . . , n.

i= 1

i= 1

i= 1

whence ( in view of the assumption ab =f:. 0)

n
n
2 L X i-1Xi 2 L: x .
=

i= 1

Rewrite this as

n
2 L Xi - 1Xi
i= 1

i.e.,

L:
i= 1

(6)

i= 1

x +

x i:. 1 ,
L:
i
=1

L
( Xi - Xi - 1 ) 2
i 1

0;
=
the equality x 1
= X n follows. (Another argument consists in notic
ing that (6) is the instance of equality in the Cauchy-Schwarz Inequality
1
::::U i vi $ (L:::U D 11 2 (L:::V l ) 1 2 , applied to Ui X i 1 , Vi = Xii and this
also implies x 1 = x n . )
=

74

Solutions

Now assume be =f 0; then (2) should be rewritten in the form

bx i + cx i+ l = (b + c)xi -1
Repeating the reasoning of the previous case, we again conclude that

Xl =

= X n

So, we are done with the cases where ab =f 0 or be =f 0. It remains to


consider ab = be = 0. Then necessarily b = 0. (Indeed: assuming b =f 0,
we would obtain a = c = 0, contrary to the condition that a + b + c = 0 )
The equality b = 0 implies c -a. Substitution into equations (1) then
yields
(7)
a(x i -1 - Xi+I ) = 0 for i 1, . . . , n.
If a = 0 then we have a b c 0 and all the equations of the system
are satisfied trivially, for every choice of x 1 , . . , X n And if a =f 0 then
equations (7) reduce to Xi -1 = Xi + l for all i (compare equations (5) of
Solution l ) , implying that n is even and the x i s assume some two values
alternately (these values may be distinct or equal) .
Summing up, we have the general form of n-tuples (xi . . . , x n ) satisfying
the system; it is presented in detail in the final section of Solution 1.
Problem 33
Let a, b, c be the sides of a triangle. Show that

= =

c <2
a + -b + --b+c c+a a+b .
33, S o lution 1
By the triangle inequality a < b + c we have

Problem

a
b+c
Likewise,

2a
2a
(b + c) + (b + c) < ---a + b + c---

2b
b
---c+a < a+b+c'

c
2c
---a + b < ---a + b + c---

Adding the three inequalities we obtain the required one.


Problem 33, S olut ion 2
Let a + b + c = u, be + ca + ab v . The proposed inequality is equiva
lent to
=

i.e. , to L < R where


L

b + -c < 2,
a + --u-a u-b u-c

a(u - b) (u - c) + b(u - c) (u - a) + c(u - a)(u - b),


2(u - a) (u - b) (u - c) .

Algebra

75

Simple manipulations bring these expressions to the following forms:


L

(a + b + c)u 2 - [a(b + c) + b(c + a) + c(a + b)]u + 3abe


u 3 - 2uv + 3abe,
2[u 3 - u 2 (a + b + c) + u(bc + ca + ab) - abc] = 2uv - 2abe,

and the problem reduces to showing that

(1)
-u 3 + 4uv - 5abe > 0.
Since a, b, c are the sides of a triangle, the sum u = a + b + e exceeds
each of 2a, 2b, 2e. This yields
R-

0 < (u - 2a) (u - 2b)(u - 2e)


=

u 3 - u 2 (2a + 2b + 2c) + u(4be + 4ca + 4ab) - Babe


u 3 - u 2 2u + u 4v - Babe,

i.e. ,

-u 3 + 4uv - Babe > 0.


The claimed inequality (1) is immediately implied by (2).

Problem

(2)

34

Let a, b, e, d be positive real numbers with abed = 1. Show that

a 2 + b 2 + e 2 + d 2 + ab + ae + ad + be + bd + cd ;::: 10.
34, Solution 1
In view of the well-known inequality
cause of abed 1,
Problem

+ (1/x) ;::: 2 for x > 0 and be

1 + ae + -1 + ad + 1 > 6 .
ab + ed + ae + bd + ad + be = ab + ad ae
ab
-

This combined with the AM-GM Inequality

a 2 + b 2 + e 2 + d 2 ;::: 4

a 2 b 2 c 2 d2

=4

immediately results in the asserted inequality.


Problem 34, Solut ion 2
Since
and since

a + b ;::: 2Vab,

e + d ;::: 2,

76

Solutions

we have

a 2 + b 2 + c2 + d 2 + ab + ac + ad + be + bd + cd
= a2 + b2 + c2 + d 2 + (a + b) ( c + d) + ab + cd
;:::: 2ab + 2cd + 2-.rah 2 + ab + cd
3ab + 3cd + 4 3 ( ab + :b ) + 4 ;:::: 10 .
=

34, S olut ion 3


We have the chain of inequalities:
Problem

a 2 + b2 + c 2 + d 2 -- a + b + c + d v abed
---J 4 - 4 >

>

4r;-;

(1)

(the root mean square, the arithmetic mean, and the geometric mean).
Denote the sum a + b + e + d by 8. Then the left inequality of ( 1 ) implies
a 2 + b 2 + e2 + d 2 ;:::: 8 2 f 4, and the right one says that 8 ;:::: 4. Therefore

a 2 + b2 + e 2 + d 2 + ab + ae + ad + be + bd + ed
a 2 + b2 + c 2 + d 2 + (a + b + e + d) 2
2
2
2 82
8
;:::: 8 + 2
58 2 5 16
= - > --

8 -

10 .

34, S o lut ion 4


The means inequality(-ies) can be used in various ways here, of which
the following seems to be the fastest shortcut:

Problem

>
=

a 2 + b2 + e 2 + d2 + ab + ae + ad + be + bd + cd
1
1 0 . {Ya 2 b 2 e 2 d 2 ab ae ad be bd ed
l
1 0 {Ya5b5e5d5
l
1 0 . {YiS = 1 0 .

Problem 35
Let a, b be non-negative real numbers with a 2 + b 2

ab < y'2 _ 1
a+b+2 and determine when equality holds.

4.

Show that

A lgebra

77

P roblem 3 5 , Solut ion 1

Applying the AM-GM Inequality to the pairs of numbers a, b and a2, b 2


we have
(1)
a + b ;::: 2v;;h and 4 = a2 + b2 ;::: 2ab.
The second inequality yields

ab :5 2, i.e.,

v;;b :5 vf'i.

(2)

Instead of examining the ratio from the problem statement, we consider


its inverse, applying the first inequality of (1) and both inequalities of

(2):

(-1- _!_) (

1.

a + b + 2 2 -rab + 2
;::: 2 2.. + = vf2 +
=2
+
;:::
ab
ab
-lab ab
vf2 2
(This requires ab =f. 0; clearly, if a = 0 or b = 0, the proposed inequality
holds.) Inverting, we obtain
1
ab
_ = vf'i - 1 .
a + b + 2 <- _
v'2 + 1
The means inequality turns into an equality only when the averaged
numbers are equal. Thus a = b = vf2 is the condition for equality.
Problem 3 5 , Solution 2

If, for some reason, one prefers not to invert, one can start from inequal
ities (1) and (2), and continue like this:

ab < ab
---a + b + 2 - 2 -rab + 2

2
= -2y + 2

'

where y stands for -lab; according to (2) , 0 :5 y :5 vf'i. It now suffices to


show
y2
(3)
<
- vl'i - 1
or, which is the same,

-2y + 2

y2 - 2 (v'2 - 1)y - 2 ( v'2 - 1) :5 0.

(4)

The roots of this quadratic trinomiai are y 1 = vf2 and Y 2 = vf2 - 2, and
hence inequality (4) reduces to
This holds because 0 :5 y :5 vf'i, so the second factor is positive, while the
first one is negative or zero. Equality occurs for y = vf2 only; i.e., when
(1) and (2) become equalities; and this is the case only for a = b = v'2.

78

Solutions

(Another option might be to examine the left-side expression of either


(3) or (4) by calculus over y E [O , J2 ] .)
Problem 35, S o lut ion 3
Starting with the second inequality of (1) and the resulting estimate (2)
( ...;;;b v'2 ) , we transform the given expression as follows:

ab
a+b+2

.;;;b .

(
..rai(

..rab

=
(noting that for a = 0 or b
GM Inequality implies

a +...;
b +;;I; 2
2 -1
+ -b- + ...;;;b ...;;;I; ...;;;I;
+ fF_ + 2 - 1

V "b

...;;;I;

(5)

0 the claim holds trivially) . Now, the AM

/a + {F_ = a + b > 2

V "b

)
)

...;;;I; - ,

and so v'2.

(6)

Transformation (5) hence leads to the estimate

1
+1

v'2

V2 -

1,

just as required, equality holding (in (6) , hence in the claimed inequality)
if and only if a = b = h.
P roblem 35, S o lut ion 4
The case where a = 0 or b 0 is trivial. So we may assume ab > 0 and
invert the proposed inequality:
=

1
a + b + 2 > -ab - J2 - 1 - '

---

equivalently,

2
-1 + -1 + a b ab -> h + l.
The given condition a 2 + b 2 = 4 calls for setting
a = 2 sin x, b 2 cos x, x E (0, 7r/2).
=

The inequality ( 7) we are about to prove becomes

1 + -1 +
2
> v'2 + 1'
-2 sin x 2 cos x 4 sin x cos x -

(7)

Algebra

79

1
1
2
+ -- + -- - 2v'2 + 2.
sin x cos x sin 2x

equivalently,

>

(8)

cos x
sin x 4 cos 2x
+ -- -cos
sin2 x
2 x sin2 2x

Denote the left side by f (x) and examine the derivative:


f'(x)

-=---

This is positive when sin x > cos x and negative when sin x < cos x; con
sequently, f (x) decreases in (0, 11) 4] and increases in [1r I 4, 1r 12), attaining
the minimum value

()

7r
1
1
2
+
+
f - =
4
sin(7rl4) cos(7rl4) sin(7rl2)

= vIn
2 + vIn2 +

2.

Inequality (8) is proved, and the condition for equality is x = 1r I 4, which


corresponds to a b v'2.
=

Problem

36

The real numbers a i , b i , q , di are such that 0 c i a i


a i + bi ci + d i for i = 1, 2, . . . , n. Prove the inequality

bi

di and

i=l

i=l

i=l

i=l

II a i + II bi II Ci + II di.

Problem

36,

S olution 1

We proceed by induction. For n = 1 we have a1 + b 1 = c1 + d b accord


ing to assumption. Assume the claim holds for a certain n . Consider
n + 1. Suppose a i , b i , c i , d i (for i = 1, . . . , n + 1) are numbers with
0 Ci a i bi di and ai + bi = ci + di. Set
n

By the inductive hypothesis, A + B

C + D ; rewrite this as

A - C D - B.

(1)

The following inequality is the content of the inductive claim:


(2)

Solution s

80

In view of the conditions imposed on the numbers a i , b i , c i , di , we have

a n + l - Cn + l
a n +l
C

<
<

d n + l - bn + l ,
b n+ l
D.

(3)
(4)

(5)

Note that the differences that occur in inequalities ( 1) and (3) are non
negative numbers. Multiplying (1) by (4) and (3) by (5) we obtain the
inequalities

(A C ) a n+ l
a
( n + l - Cn + l ) C
-

<
<

(D B ) bn+ b
( dn+ l - b n + l ) D.
-

Adding them, we get

and this is exactly the inductive claim (2) . The assertion results by
induction.
Problem

36,

Solut ion 2

By the given conditions,

ri ai - Ci = di - bi ;:::: 0 for i = 1, . . . , n .
=

Look at the product

II di

i=l

(bl

rt ) ( b 2 + r 2 ) ( b n + rn )

Multiplying out, we obtain the term b 1 b 2 bn plus several summands


of the form
(0 :5 k :5

n-

1),

(6)

with distinct indices i 1 . . . . , i k from the set { 1 , . , n }, complemented by


. . , in- k to the whole { 1 , . . , n } . Similarly, the product

h,

II Ci = ( a l - r 1 ) ( a 2 - r 2 ) ( a n - rn )
i=l
is equal to a 1 a 2 a n plus the sum of terms
a 1 a '" ( -r31 ) ( -rJn-/o ) (0 :5 k :5 n 1).

(7)

Algebra

81

Since 0 :::; a i :::; bi for i = 1, . . . , n, we see that each term (7) is dominated,
in absolute value, by the corresponding term (6) . So the joint sum of all
the numbers (6) and (7) is non-negative.
Now, the sum of all numbers (6) equals II d i - II bi ; the sum of all
numbers (7) equals II Ci II a i. Consequently,
-

=1

=1

=1

=1

di - II bi + II Ci - II ai 0,
II
i
i
i
i
as claimed.
Problem 37
Prove the following inequality for all integers n > 1:

( 1 + (n + 1) n+1 ) n-1 > ( 1 + nn ) n


n+2

n+1

37, Solution 1
We show that the number n n(n-1 ) can be ( smartly enough ) put in be
tween the two expressions we are about to compare:
P roblem

( 1 + (n + 1) n+1 ) n- 1 > n n (n- 1 ) > ( 1 + nn ) n

(1)
n+2
n+1
Taking roots of order n - 1 and n, respectively, we recast the left in
equality and the right inequality of (1) into their equivalent forms:
(n + 1) n+l + 1 > n n and n n - 1 > n n + 1 .
(2)
n+2
n+1
___

---

The second inequality is immediate:

(n + 1)n n- 1 = n n + n n - 1 > n n + 1 for n > l.


For the first one, apply the Binomial Theorem to obtain ( for n > 1)

(n n+ l + (n + 1)n n + + 1) + 1
> n n+1 + (n + 1)n n > n n+l + 2n n
n n (n + 2).
Thus, both inequalities of (2), hence of (1), are proved.
(n + 1) n+1 + 1

82

Solutions

2
Denote the left-side expression and the right-side expression by L and R ,
respectively. The sequence
tends increasingly to There
fore we have for
Problem 37, S o lut ion

((n + 1)/n) n

n 2

e.

( n + 1 ) n+ l
4
n+2
(n n + 1)/nn 1 + n -n ::; 5/4 for n 2, and so
n n + 1 - -45 n n for n 2.

Moreover,

>

and

>

<

>
_

We now estimate the ratio L j R from below:

((n + 1)n+1 + 1 ) n-1 ( nn + 1 ) -n


R
n+1
n+2
( (n + 1 ) n+l ) n- 1 ( nn + 1 ) -n ;
n+2
n+1
and we continue the estimate, using inequalities (4) and (3):
en + 1)n+1 ) n- 1 ( . ) -n
R
n+2
4. n + 1
1
(n + 1)n2 +n - n n ( ) n
(n + 2) n 1
( n : 1 ) n2 ( ) n ( : :) n 1
= ( ) n (( n : 1 ) n ) n ( :: n+ 1 nn ++ 21 2
-45 n -94 n 31 nn ++ 21 2
,
9 n 1
-5 . 3 ;
and this number exceeds 1 for n 2. Thus
R , as asserted.
L

=
>

>

>

>

(}

'

(3)

'

'

(}

>

Problem 3 7 , Solut ion 3

Taking logarithms on both sides, rewrite the claimed inequality as

(n - 1 ) ln 1 + (nn ++ 21)n+ l

>

n ln

1 + nn
n+1

(4)

Algebra

83

or, which is the same,

ln 1 + (n + 1)n+ 1
n
n+2

>

So the problem reduces to showing that f (n + 1)


f (x)

_1_ ln x"' + 1
x-1
x+1

_1_ ln 1 + n n
n-1
n+1
>

f (n) for n

2, where

ln(x"' + 1) - ln(x + 1)
.
x-1

This is done in a more or less routine way by calculus. Since


(x"') ' = (e"' 1" "' ) 1 = e"' 1" "' (ln x + 1) = x"' (ln x + 1),
we get
f'(x)

1-)
( x"' (lnx x+ +1 1) - x - 1) - (ln(x"' + 1) - ln(x + 1) )
x+1 C
"'

(x - 1) 2
_1_ x"' (ln x + 1) _ _1 _ _ ln(x"' + 1) - ln(x + 1) .
(x - 1) 2 .
X+1
X-1
Xx + 1

Since (x"' + 1) / (x + 1) < xx -1 for x


played fraction fulfills the estimate

>

1, the numerator of the last dis

x"' + 1
ln(x"' + 1) - ln(x + 1) = In -- < ln x x -1 = (x - 1) ln x.
x+1
Therefore
-1
ln x
x"' (ln x + 1) -1
- -!'(x) > -X+1
X-1
X "' + 1
X-1
(xx+ 1 - 1) - (x + 1) ln x
(x - 1) (x "' + 1 ) (x + 1)

In view of the well-known inequality In x < x - 1 we obtain


(xx+ l - 1) - (x + 1) ln x > (x"' + 1 - 1) - (x + 1)(x - 1)
=
x2 (x"' -1 - 1) > 0 for x > 1,
and consequently !' ( x) > 0 for x > 1. This mel:j.ns that f ( x) is strictly
increasing in (1, ) and hence f(n + 1) > f (n) for n 2.
Problem 38
Let n 9 be an integer. Which one of the numbers ( v'n ) vn+ 1 and
( Vn+1 ) ,;n is greater?
oo

84

Solutions

Problem 38, S olut ion 1

The answer is easy to guess, the first number is greater:

(1)
To prove this guess, take logarithms on both sides: inequality (1) is
equivalent to
v'n + 1 ln yn > yn ln v'n + 1,
i.e., to
l n ..;n l n v'n+l
9

10r n .
>
r=
n
v
n
+1
y
This holds because the function f (x) = (ln x)jx is strictly decreasing for
x v'9 = 3 (the derivative f'(x) = (1 - ln x)jx2 is negative for all x > e;
and since. 3 > e, we are done) .
Problem 3 8 , Solution 2
We will use the well-known relation ( 1 + r < e, which holds for all
positive integers n. Inequality (1) is equivalent (via squaring) to

Division by n ..;n transforms this into

( 1 + -n1 ) n+../n(n+l)

and raising both sides to the power + ..;n brings this to the form

Qn

>

for

(2)

Qn ::; ( 1 .jii- ) n < 2.06n.

the last inequality is also equivalent to (1).


The exponent = n + Jn (n + 1) is estimated as follows:

Hence

n (n + 1) ::; 19 n 2

( 1 + ;;1 ) qn < ( ( 1 + ;;1 ) n) 2 .0

proving (2) .

=}

=}

<

e2 06 < 7.846 < 8 <

::;

n,

Algebra

85

Remark

J1i R

The use of a calculator can be avoided if we resort to another well-known


inequality (1 + f + 1 > e, holding for all n 1 ; in particular, we have
(18/ 1 7) 1 8 > e. Since
< 1 + l8 , we get q n :$ ( 2 + l8 n
for n 9. Knowing just that e < 2.8, we obtain e 2 < 7.84, and so

(
1 n 2+1/1 8
1
+
( ) ( ;;: ) )
1
1 + ;;:

<

<

e2 + 1/1 8

e 2 e l/ 1 8 < 7.84 .

18
< 9 :$ n .
17

39
Prove the inequality

Prob lem

c:)

ffn < 4n for

n = 1, 2, 3, . . . .

39, Solut ion 1


The following stronger inequality will be proved by induction:

Problem

(1)
Obviously, the proposed inequality is immediately implied by (1).
For n 1 , inequality (1) holds. Assume (1) holds for a certain integer
n 1 ; we must show that
=

(2)
This is done as follows:

(2nn ++ 12 ) . v'3n + 4

=
<

(2nn ) . (2n(n + 1)1)(n(2n++1)2) , v'3n + 4


+
2(2n + 1) . (2n ) . v'3n + 1 . J 3n + 4
3n + 1
n
n+1
2(2n + 1) . 4n . J 3n + 4 .
3n + 1

n+1

The claim (2) will be proved if we show that

2n + 1
n+1

. J 3n3n ++ 41 - 2_
<

(3)

86

Solutions

By squaring, relation (3) is equivalent to


1 )-'- 2 -'-( 3 n + 4--'-) < .
-'--(2_n_+---4
___
( n + 1) 2,---( 3n + 1) - '

(4)

and this is recast into


12n3 + 28n2 + 19n + 4 ::; 12n3 + 28n2 + 20n + 4,
holding trivially. Induction is complete.
Remark

This is a very graceful example of an induction proof which requires a


strengthening of the claim in order to carry out the induction step. ( In
the inductive procedure, there is a moment in which "assertion becomes
assumption" . ) An attempt to prove the inequality in its original form by
induction, in a straightforward way, would fail!
Problem 39, S o lut ion 2

We transform the asserted inequality into successively equivalent forms:


(2n) ! 5n
1 2 3 (2n - 1) (2n) 5n
1 3 5 (2n - 1) 5n
(1 3 5 (2n - 1)) 2 (3n)

and finally

::; (2 n n!)2 ,
< (2 4 6 (2n)) 2 ,
::; 2 4 6 (2n),
< (2 4 6 (2n)) 2 ,

( 23 45 (2n(2n -- 2)1) ) 2 . 2n
1

Denote the left side of (5) by a n ; thus

::; 3

(5)

2n - 1 . --2n - 1
a n = 21 . 23 . -43 . -45 . -65 . -67 . . . --2n - 2
2n .
To create a n+l out of a n , two further factors have to be attached at
-

the end of : this product. Appending only one of them we obtain an


expression, which we denote here by b n :
2n - 1 2n - 1 2n + 1
b n = -21 . -23 . -43 . -45 . 65 . 67 . . . --2n - 2 . --2n . --2n .
-

Accordingly,

2n + 1 >
b n = a n --an .
2n

(6)

Algebra

87

On the other hand,

- - ---

4n 2 1
< bn - 1
4n 2
Thus b 1 , b 2 , ba , . . is a decreasing sequence. And since
1 3 3 5 5 7 7 9 9 11
2
bs
2 2 4 4 6 6 8 8 10 10 = 0 66618 < 3 '
all the subsequent bn s are smaller than 2/3. Hence by inequality ( 6 ) ,
a n < 2 / 3 for n 5 , 6 , 7, . . . . Also the initial terms a1, a 2 , aa, a 4 are
smaller than 2/3 , as can be verified directly. Thus estimate (5) is proved,
and we are done.
b

2n 1 2n + 1
n 1 2n
2n

---

= bn - 1

Remark

At calculus courses it is taught that the sequences ( a n ) and ( bn ) tend to


a common limit, whose exact value is 2/7r ( the Wallis formula).
P roblem 40
Prove that the inequality

holds for any real numbers at, a 2 , . . . , a r . Find conditions for equality.
Problem 40, S olution 1
Denote the given expression by Fr (a l , . . . , a r ):

Fr (a1, . . . , a r ) =

t
t
(
:m:: )
n=l m=l

(1)

Clearly, Fr ( O, . . , 0) 0. Now, for r 1 we have F1(a1) = a V 2 ;::: 0,


with equality only for a 1 = 0. It is thus natural to conjecture, for each
r, that the inequality
(2)
should hold for every r-tuple of real numbers ( a1, . . . , a r ) =/= (0, . . . , 0) .
We will prove this guess by induction. The start ( r 1) has been done
already. Fix r ;::: 2 and assume inductively
=

Fr - 1(a1 , . . . , a r -d > 0 whenever (a1, . . . , a r-d =/= (0, . . . , 0) . (3)


Consider r real numbers a1, . . . , a r -1, a r , not all zero. In the expression
( 1 ) , isolate the terms corresponding to n = r or m

= I: ( I: amann
n=l m=l

m +

r:

aran

r+

88

Solutions

mam+arr + a2r .
(m=1
)

(4)

If a 1 = = a r 1 = 0, then automatically a r =f 0, and therefore we have


Fr (O, . . . , 0, a r ) = a/ (2r) > 0, as needed.
Thus assume that at least one of the numbers a1 , . . . , a r-1 is different

from zero. Consider the expression on the right side of (4) as a function
of the variable ar, which we now denote by x:
T (x) = Fr (at . . . . , U r- b x) = A x 2 + Bx + C ;
according to (4) , the coefficients A , B , C are expressed by the formulas
1
A= -,

B-

2r

r- 1

r-1

r-1

ak
a m 2 -U n + L -L
L -r+k '
m +r
r +n

n=1

m=1
k =1
UmUn
C=
m
n=1 m=1 + n
Let us calculate the discriminant D = B 2 - 4A C of the quadratic trino
mial T (x) :

hence

i.e.,
(5)

where

C mn

(m + r) (r + n ) 2r(m + n )
2r(m + n) - (m + r)(r + n)
(m + r)(r + n ) 2r(m + n )
(r - m) (r - n )
=
2r(m + r) (r + n ) (m + n )

A lgebra

Writing

89

r k
b k = -for k = 1 , . . . , r - 1
r+k
-

we thus have

1 bmb n
2r m + n
Inserting these expressions into formula (5) we obtain

C mn =

D
=
4

---

_ ]_2r I: ( bmmbn+amnan )

n=1 m=1
Fr-1 (a 1 b 1 , a 2 b 2 , . . . , a r-1 b r-1 )

Fr-1 (u l t u 2 , . . . , U r-1 )

2r

(6)
2r
where U k = a k b k for k = 1 , . . . , r - 1 . Since the b k s are positive, there
is a non-zero number among the numbers u 1 . . . . , U r-1 Thus, by the
inductive hypothesis (3) ( applied to the numbers u 1 , . . . , U r - 1 in place
of a 1 . . . . , a r - 1 ), we have Fr - 1 (u l t u 2 , . . . , u r - 1 ) > 0. This, in view of
the equality ( 6 ) , implies D < 0. A quadratic trinomial with a negative
discriminant and a positive leading coefficient (A = 1 / (2r) > 0) assumes
positive values only.
We have thus shown that the inequality in (2) holds for every real num
bers a 1 , . . . , a r - 1 , a r , not all equal to zero. This concludes the inductive
step.
Problem 4 0 , S olution 2

Consider the polynomial P(x) =

(P (x)) 2

L anxn .

n=l

( an xn) (t, amX m)


an (t, amxm) x n
ll am anxm+n) .

Now introduce the polynomial

Q(x) =

By squaring,

a m a n x m+n ) .
t
t
(
m
n=l m=l
+n

(7)

Solutions

90

The assertion of the problem is that


Q (1)

2:

0.

(8)

Claim (8) will be proved by examining the derivative of Q (x ), which


equals

Comparing equation (7) , we see that

Q'(x) = .!.X (P( x)) 2 2: 0 for x > 0.


(9)
Notice that Q(O) = 0, by the definition of Q(x). The function Q (x) is
continuoHs in [0, 1] and non-decreasing, in view of the inequality in (9) .
Hence Q(1) 2: Q(O) = 0; claim (8) is settled.
Equality holds in (8) if and only if Q(x) is constant, i.e. (see (9)), when
P ( x) is the constant null function. And this is the case if and only if
= ar = 0.
a1 =

41
For a fixed integer n

P r oblem

2:

1 find the least value of the sum

given that x1 , . . . , X n are positive numbers satisfying


1
1
1
+++= n.

Xl

X2

Xn

41, S olution 1
Denote the sum under consideration by S ( = S(x1 . . . . , x n )). The follow
ing inequality is the key to the solution:
Problem

x k + 1 2: 1 + k1 for x > 0,
-;;

k=

1 , 2, 3, . . . .

(1)

Four proofs of (1) are presented below! Now, taking inequality (1) for
granted, we just match each term of S with the corresponding term of
the constraint condition, to obtain

A lgebra

91

with equality for x1 = = X n = 1. The "n" cancels and we get

1+-+-++ 2 3
n
as the minimum value of S. It remains to prove inequality (1).
First proof of (1).
For x, k > 0 fixed (x real, k an integer) ,

k
kx xk + -X1 - 1 - -k1
-

(2)

= x (x k - 1) + k(1 - x)
=

k-1

( (k xi ) - k)

x(x - 1) L x i + k(1 - x)
i=O

= (x - 1)

k
= (x - 1) L (xi - 1)

j=l

= L (x - 1)(x i - 1)

j=l

:2:

because the factors in each term of the last sum agree in sign.
Second proof of {1).
For fixed x, k consider the arithmetic mean and the geometric mean of
the k + 1 numbers, one of them being xk and the others equal to 1/ x:
1/(k+ l ) = 1;
_1_ xk + .!_ + . . . + :2: xk . .!_

k+1(

.!. ) (

X
X

.!. )

X
X
--.,....._...,

hence xk + (kjx) ;:::: k + 1, which is just a restatement of {1).


Third proof of {1).
The Bernoulli Inequality {1 + a) k . ;:::: 1 + ka holds for all a :2: - 1 and
every integer k ;:::: 1. Set a = x - 1, thus obtaining: xk ;:::: 1 + k (x - 1).
Hence

x k 1 1 + k (x - 1) + -1 = x + -1 - 1 + -1 > 1 + -1 .
- + -X >
X
X
k

92

Solutions

Fourth proof of (1) .


For a fixed integer k 1 consider the left side of ( 1) as a function f ( x ) ,
defined on positive reals. Since

!' ( x ) x k - 1
=

-2

<
>

0 for x E (0, 1) ,
0 for x E (1 , oo ) ,

we see that f (x) takes at x = 1 its (global) minimum value 1 + (1/k).


Proble m 4 1 , S o lution 2
The constraint imposed on the numbers x1 , . . . , X n is that their harmonic
mean is equal to 1. So their geometric mean is at least 1; i.e. , we have
(3)
Take the sum (2) to the common denominator

1
1 m1 + m 2 + + m n m
1+-+..+-=
=,
(4)
,
2
n
n.
n!
where m k . = (n!)/k for k = 1, 2, . . . , n, and consider the m positive num
bers:
. . . , x: , . . . ' x: .

------

Their arithmetic mean compares with the geometric mean:


(5)

Since km k = n! for k = 1, . . .
the product in the parentheses equals
( x 1x 2 x n ) n ! , and we get by inequalities (5) and (3)
, n,

Division by n ! now yields (see (4))


Xl
X
X
m
1
1
> - = 1 + - + . . + - ,
-+-+..+- (6)
1
2
n
n!
2
n
showing that the sum (2) is the least value that the expression in question
can have.

Remark

The argument of the last solution shows that the inequality in ( 6 ) is a


consequence of the condition (3) , actually weaker than that given in the
problem statement (the geometric mean instead of the harmonic mean) .

Algebra

93

P r oblem 42

On a given segment AD, find points B and C so as to maximize the


product of the lengths of the six segments AB, AC, AD, BC, BD, CD.
Problem 4 2 , Solut ion

Without loss of generality, assume that B lies between A and C. Take


the length of AD for a unit (AD = 1) and set

X = BC, y = AB - CD;

x E (0, 1 ] , y E (- 1, 1].

Then AB + CD = 1 - x, and hence

AB = ! (1 - x + y) ,
AC = ! (l + x + y) ,

CD = !(1 - x - y),
BD = !(1 + x - y) .

The product under consideration equals

p(x, y)
l6 x(1 - X + y) (1 - X - y)(1 + X + y) (1 + X - y)
= l x ((1 - x) 2 - y 2 ) ( (1 + x) 2 - y 2 ) .

Hence,

(1)

where

f(x) = x ( 1 - x 2 ) 2 = x 5 - 2x 3 + x;
equality holds i n (1) i f and only i f y = 0 .

(2)

The derivative
is positive for x E (0, ! .J5 ) and negative for x E ( ! .J5, 1) . Thus the
maximum value of f(x) is attained at x = ! .J5. Consequently, the prod
uct p( x, y) is maximized at x = ! .J5, y = 0. This corresponds to placing
points B and C symmetrically with respect to the midpoint of AD, at
the mutual distance BC = ! .J5.
P roblem 42, Solution 2

above, the problem is reduced to the maximization of the polyno


mial (2). This can be done without calculus. In the weighted AM-GM
Inequality
As

apbq :5 pa + qb for a, b 0, p, q 0, p + q = 1,

94

Solutions

set a = 4x 2 , b = 1 - x 2 , p = 1/5, q = 4/5, to obtain


i.e. ,

4 1 / 5 f (x) 2/ 5 t '
equality holding only when 4x 2 = 1 - x 2 ; that means, for x = i v'5 . Con

clusion as before.

Problem 42, S o lut ion 3

The means inequality can be used in a yet smarter manner. Assuming


that B lies between A and C, set

AB = u,
so

AC = u + x,

BC = x,

BD = X + v,

CD = v ;
AD = u + X + v = 1 .

Denoting the product under investigation again by p , we have

(uxv (u + x) (x + v) ) 2
(u(x + v)) (u(u + x)) (v(u + x)) (v(x + v)) (x 2 ) .

(3)

The geometric mean of the five numbers

u(x + v) , u(u + x), v(u + x) , v(x + v), x 2

(4)

does not exceed their arithmetic mean. Thus the product on the right
side of (3) does not exceed

( u(x + v) + u(u + x) + v5(u + x) + v(x + v) + x 2 ) 5

The numerator of the fraction in parentheses equals

ux + uv + u 2 + ux + vu + vx + vx + v 2 + x 2 = ( u + v + x ) 2 = 1 .
Expression ( 3 ) for p now yields p ( i ) 5 1 2 .

The means inequality becomes an equality only if the averaged quantities


are equal. Since the arithmetic mean of the numbers listed in (4) is 1/5,
the condition for equality takes the form

u(x + v) = u(u + x) = v(u + x) = v(x + v) = x 2 = .


The last system of equations ( together with u + v + x = 1) is fulfilled
only for x = k v'5 and u = v = ! ( 1 - x). Conclusion as in Solution 1 .

95

Algebra

43
Find all functions f: JR. JR. satisfying the equation

Problem

x 2 j (x) + f (1 - x) = 2x - x 4 for x E R
43, Solution
In the given equation

Problem

x 2 f(x) + /(1 - x) = 2x - x 4
set 1 - x in place of x:
(1 - x) 2 !(1 - x) + f (x) = 2(1 - x) - (1 - x) 4 .
Multiply equation (1) by (1 - x) 2 :
x 2 ( 1 - x) 2 f(x) + (1 - x) 2 /(1 - x) = (1 - x) 2 (2x - x 4 ).

(1)
(2)
(3)

Subtract equation (3) from (2):


Rewrite this as

K (x)f(x) = (1 - x)M (x),

(4)

K ( x) and M ( x) denoting the polynomials in square brackets:


1 - x 2 (1 - x) 2
K (x)
(1 - x + x 2 )(1 + x - x 2 ),
M (x)
2 - (1 - x) ( (1 - x) 2 + 2x - x 4 )
=
2 - (1 - x) (1 + x 2 - x 4 )
1 + x - x2 + x3 + x4 - x 5
( 1 + x 3 )(1 + x - x 2 )
=
= ( 1 + x)(1 - x + x 2 ) ( 1 + x - x 2 )
=
(1 + x)K(x) .
Equation (4) takes the form

K(x)f(x) = (1 - x 2 )K(x),
implying f(x) = 1 - x 2 , unless K (x) = 0 . In this latter case . we get
x 2 (1 - x) 2 = 1 (by the definition of K (x)), and this is equivalent to say
ing that

96

Solutions

The first equation of


roots:

(5)

has no real roots and the second one has two

a = ! (1 + '5 ) and f3 ! (1 - '5 ) ;


(6)
these roots of x 2 x + 1 satisfy
a + /3 = 1, a /3 = - 1;
(7)
a 2 = a + 1, {3 2 = f3 + 1.
In conclusion, f (x) = 1 - x 2 for all x =f a, {3. And for these two ex
ceptional values of x equation (1) yields
a 2 f (a) + / (/3 ) = 2 a - a 4 and {3 2 f(/3 ) + f(a) = 2{3 - {3 4 . {8)
Using formulas (7) we calculate: a 4 (a 2 ) 2 (a + 1) 2 = a 2 + 2a + 1,
and likewise {34 = {3 2 + 2{3 + 1. Equations (8) become
a 2 f (a) + f ( /3 ) = -a 2 - 1 and {3 2 f(f3) + / (a) = -{3 2 - 1. (9)
Multiplying the second equation of {9) by a 2 we obtain, by identities {7),
f(/3 ) + a 2 f(a) -a 2 {3 2 - a 2 = - 1 - a 2 ,
=

which is j ust the first equation of (9) . Thus the two equations {9) do not
yield a unique evaluation of f(a), f(f3) . In fact, f{a) can be any real
number c, and then

f(f3) = - (a 2 + 1) - a 2 c = - (a + 2) - (a + 1)c.
Thus the general solution of equation (1) is
for x =f ! (1 '5 ) ,
1 - x2
f(x)
c (arbitrary real number) for x = ! { 1 + '5 ) ,
- ! {5 + '5 ) - ! (3 + v'5 )c for x = ! (1 - '5 ) .

(1 0 )

Problem 43 , Solution 2

A functional equation like {1), with polynomial coefficients and a polyno


mial on its right side, is likely to have a polynomial solution. Evidently,
if a polynomial f (x) satisfies equation (1) , it must be a polynomial of
second degree. Postulating f(x) Ax 2 + Bx + C we get from (1)
=

Ax 4 + Bx 3 + Cx 2 + A(1 - 2x + x 2 ) + B (1 x) + C = 2x - x 4 ,
whence A = - 1, B = 0 , C = 1 ; i.e. , f(x) = 1 - x 2
Thus we have found the solution f(x) = 1 - x 2 by simple trial. It
-

is
unique in the class of polynomials; one is tempted to conjecture that it
is unique in general. In an attempt to prove this guess, let us set

f(x) = 1 - x 2 + g(x) ;

{11)

A lgebra

97

{1)

that takes equation

equivalent to
Replacing

to the form

x 2g(x) + g(1 - x) = 0.

{12)

(1 - x) 2 g(1 - x) + g(x) = 0.

{13)

x by 1 - x,

Viewing {12) and (13) as a homogenous system of two linear equations


with unknowns g(x) and g(1 - x), compute its determinant:

1
x2
2
2
1 (1 - x) 2 = x (1 - x ) - 1.

{14)

If the determinant is different from zero, the system has only the triv
ial solution g(x) = g(1 - x) = 0. If the determinant is zero, then the
two equations {12) and {13) are linearly dependent ; g(x) may be gien
any value, and then g(1 - x) can be computed from any one of these
equations. (We see that the uniqueness conjecture fails.)
Now, the determinant {14) vanishes if and only if x satisfies one of the
equations {5) from Solution 1; equivalently, if x is one of the numbers
a, (3 given by {6). Equation {12) with x = a and with x = (3 becomes,
respectively,

Since (a f3) 2 = 1, the two equations (just obtained) coincide. Denote g(a)
by d; then g( f3) = -a 2 d. Hence by definition {11) (and formulas {7))

f(a) = 1 - a 2 + d = d - a, f(f3) = 1 - (3 2 - a 2 d = -(3 - (a + 1)d.


Setting, further, d - a = c, we get f(a) = c and
f(f3) = -f3 - {a + 1) {a + c) = -(3 - a 2 - a - (a + 1)c = -{a + 2) - (a + 1)c.
So we have arrived at the same formula
Solution 1.

(1 0 )

which we had found in

Problem 44

Let A and B be real numbers different from zero. Prove that the function
f(x) = A sin x + B sin ( ../2 x) is not periodic.

98

Solutions

Problem 44, S olut ion

Assume f is periodic with period T

>

0,

f (x + T) = f(x) = f (x - T) for all x E R


Then of course
f (x + T) - f (x - T)

0 for x E lR,

which in view of the definition of f (x) leads to the equality


A sin(x + T) - A sin(x - T) +
+B sin (v'2 x + v'2 T) - B sin (v'2 x - v'2 T)
0,

2A cos x sin T + 2B cos ( v'2 x) sin (v'2 T) = O for x E R

(1)

Setting x = 0 we obtain
(2)

2A sin T + 2B sin (v'2 T) = 0 ;


and setting in (1) x = 1rj2 we get
2B cos (1r ; v'2 ) sin ( v'2 T) = 0.

(3)

Since B =I= 0, equality (3) shows that sin ( v'2 T) = 0. And since also
A =I= 0, equality (2) now shows that sin T = 0.
It follows that each one of the two numbers T and v'2 T has to be an
integer multiple of 1r. This is however impossible, v'2 being irrational.
Contradiction ends the proof.
Problem 44, Solution 2
The function f (x) has derivatives of order one and two:

f'(x) = A cos x + B v'2cos (v'2 x) ,

f"(x) = -A sin x - 2B sin (v'2 x) .

We thus have the equations


f (x) + f"(x) = -B sin (v'2 x) ,

2f(x) + f"(x) = A sin x.

(4)

Assuming that f is periodic with period T > 0, we conclude that T is


also the period of f' and f", hence also of f' + !" and 2!' + !". In view
of the equations (4) and the condition A =I= 0, B =I= 0, we infer that T is
a period of both sin x and sin ( v'2 x) .

99

Algebra

Since each period of sin x is a multiple of 21r , and similarly that each
period of sin ( V2 x ) is a multiple of V2 21r, we are again led to a con
tradiction with the irrationality of 2.
Problem

45

Find all monotonic functions

f : lR

lR satisfying the equation

J(4x) - f(3x) = 2 x
P roblem

45,

for

x E JR.

Solut ion 1

If a monotonic function

f satisfies the given equation, then

f(4x) - f(3x)

> 0

< 0

for
for

x > 0,
x < 0,

showing that f is strictly increasing in each one of the two intervals


(-oo , O) and (O, oo). Hence, f is increasing on JR. Replacing 4x by x,
rewrite the equation as

f(x) = !x + f (x)
For any real number x =f
tion of formula (1) gives

f(x)
=

for

x E JR.

(1)

and any natural number n , repeated applica

! x + f (x)
!x + ! x + f ( ( ) 2x)
! x + ! x + ! ( i)2x + f ( ( ) 3 x)
!x + ! x + ! ( ) 2 x + + ! ( ) n - l x + f ( ( ) n x)
1 (3)n J(( n ,
!x -\
+ ) x)
4

i.e. ,
(2)
Now, keep x fixed and let n vary. The sequence ( ( i) n x ):= is decreasing
l
if x > 0 , and increasing if x < 0 ; the same is the behaviour of the sequence
(f ( ( t x ) ):=1; the value f (O) is its lower (upper) bound, not necessarily
sharp. Thus there exists a finite limit (maybe, depending on x):

g(x)

n -+oo
lim

f ( ( t x) .

100

Solutions

Assume that
an f with

g(u) < g(v) for some positive numbers u and v, and choose
0

<

<

! (g(v) - g(u)) .

(3)

By the definition of a limit, we have

f ( ( r u)
and

<

g ( u) + f

! ( (rv) > g(v) -

for n large enough

(4)

for n large enough.

(5)

Pick an integer k so large that inequality ( 4) holds for n = k . Next, find


an integer m so large that estimate (5) holds for n = m and, moreover,
( ) mv < () k u. Since f is strictly increasing, and in view of condition
(3), we hence obtain

0 < f ( ( ) k u) - f ( () m v)

(g(u) + ) (g(v) - )
= g(u) - g ( v) + 2e < 0
- obviously a contradiction. This means that g ( u) = g ( v) for any posi
tive numbers u and v. In other words, there exists a common limit
lim f ((rx) =
for every X > 0.
n->oo
<

::

Analogously, there exists a common limit


lim

n->oo

! ( ( rx) = a

for every

x < 0.

(2) , thus obtaining


f (x) = 2x + c for x > O.

So we can pass with n to infinity in formula

f(x) = 2x + a for x < O and


S ince f has to be increasing on lll, we arrive at the final result:
2x + a for x < 0,
(6)
f (x) = b
for x = 0,
2x + c for x > 0,
where a, b, c are arbitrary constants such that a b
(Clearly, every

c.

such function satisfies the given equation. )


P ro blem

4 5 , S o lut ion 2
The reasoning becomes shorter if we resort to the well-known fact from
calculus that every function, monotonic in some real-line interval, has
one-sided finite limits (equal or not ) , at each point of that interval. Ac
cordingly, if f is a function satisfying the conditions of the problem, then
there exist the finite one-sided limits

a = xlim
f(x),
--+0-

c = X--+0+
lim f (x) .

Algebra

101

As in Solution 1, we observe that .f must be increasing on


f ( O ) by b we have, as before, a ::::; b ::::; c. Define

Denoting

h (x) = f(x) - 2x.


Then also

x-+lim0+ h(x) = c.

lim h(x) = a,
:J:-+0-

The given functional equation, recast in terms of the function


the form

h,

takes

(h(4x) + 2(4x)) - ( h(3x) + 2(3x)) 2x,


i.e. , h(4x) h(3x) ; equivalently,
h(x) = h ( ix) for all x E R
So we have, for every x E lR and every n E N,
=

When n tends to infinity, the sequence (h ( ( i t x) ):=l tends to a, c or


b, according as x is negative, positive or zero. So we obtain, in limit,

h(x) =

a
b
c

for x < 0,
for x = O,
for x > 0,

which is nothing else than the formula ( 6 ) , worked out in the first solu
tion.
Problem 46

A sequence ao, a1, a 2 , . . . of real numbers different from zero is generated


according to the rule: a n +l = (a; - 1)/ (2a n ) Show that it contains
infinitely many positive terms and infinitely many negative terms.
Problem 46 , Solution

1
If we change the signs all terms of a certain sequence that obeys the
given rule, we obtain another such sequence. Therefore it will be enough
to show that any such sequence has infinitely many negative terms.
Assume the contrary; that is, assume a n > 0 for n no. Then

a n for n no,
2
implying a no +k < 2 -k a n0 for k = 1, 2, 3, . . . . Thus, sooner or later, there
must appear a term a m E (0, 1). The next term a m+l . equal to
(a !, - 1)/(2a m ) ,
a n+ l

a; - 1
2a n

---

<

102

Solutions

is negative. The assumption that "almost all" terms are positive has
driven us to a contradiction.
P roblem 46, S olut ion

Let

d n = an+l - an .

By the recursion, 2anan+l = a -

1, and so

for all n. Now suppose we have a block a N , aN+l . . . , aN+r of consecutive


positive ans. Thus, if N ::::; n < N + r, then

(1)
which combined with the inequality d > 1 implies: dn < - 1 . S o we
have
(2 )
aN > 1 + aN+l > 2 + aN+2 > > r + aN+r > r,

yielding an upper bound on the length of the block ( r < aN ) . Conse


quently, a block of consecutive positive terms cannot be infinitely long;
a negative term must eventually occur. A similar argument shows that
any block of negative terms necessarily has a finite length; j ust the in
equalities in (1) and ( 2 ) have to be reversed.
Problem 46 , S olution

The first two solutions do not differ in any essential way. The third one is
different. If not so simple as the foregoing ones, this solution gives more
insight into the nature of the problem. The sequence is determined by
its initial term ao, so the information about the positions of positive and
negative terms must be somehow encoded in that single number, and the
present proof shows how to decode it.
The clue observation is that the recursion formula imitates the well
known trigonometric identity
cot 2 (} - 1
cot 20 = ---2 cot (}
Now, let t E (0, 1) be the unique number such that a0 = cot ?rt. Then,
according to the above, a 1 = cot 27l't, a2 = cot 41l't, and by induction,
n
an = cot 2 1l't

for

0, 1, 2 , . . .

(3)

These values of the cotangent function are well defined. ( Indeed: assum
ing n is the least index such that cot 2 n 1l't makes no sense, i.e. , 2 n t is
n l
an integer, we would get that 2 - t is an "integer and a half" , implying
l
n
an - l = cot 2 - 7l't = 0, contrary to the condition that the sequence has

Algebra

103

non-zero terms. ) In other words,


representation

t = (O.c1c 2 c3 . . . ) 2

with

is not a dyadic fraction; its binary

ci E {0, 1}

for

i = 1, 2, 3,

has infinitely many zeros and infinitely many ones.


Notice that
> 0 if L2xJ is even,
co t 1rx
< 0 if L2x J is odd;

therefore ( see (3)) a n is positive or negative according as L2 n + 1 tJ is


even or odd. And since L2 n+l tJ = (c1 . . . C n + 1) 2 , we see that a n > 0
when C n +1 = 0 and a n < 0 when Cn +l = 1 . Thus the distribution of
positive and negative terms in the sequence corresponds exactly to the
distribution of zeros and ones in the binary expansion of the number
t = (cot- 1 ao)/7r.
P roblem 47
Four sequences of real numbers

satisfy the simultaneous recursions

for n =

0, 1, 2, . . . . Suppose there exist integers k, r

Prove that

;:::

1 such that

a1 = b1 = c 1 = d1 = 0.

Problem

47, S o lution 1
Define s n = a n + bn + C n + d n . Conditions imposed on the given se
quences imply that S k+ r = S k and s n +1 = 2s n for n = 0, 1 , 2, .
The
last equality entails ( by induction ) s n = 2 n so for n = 0, 1, 2, . . . . Thus
2 k+r so = 2 k so, and hence so = 0. This yields s n = 0 for all n ;::: 0. Con
sequently, we get for n ;::: 1:
.

a n + Cn (a n - 1 + b n - 1) + (cn - 1 + d n - 1) = S n - 1 = 0.
=

Wn+l

(an + bn ) 2 + (b n + Cn ) 2 + (en + dn ) 2 + (d n + a n ) 2
2(a; + b; + c; + d;) + 2(a n bn + bn Cn + Cn d n + dn a n )
2w n + 2(a n + Cn ) (bn + d n )

(1)

104

Solutions

For n ;::: 1 we have by (1): Wn + l = 2w n . Induction yields Wn = 2 n - l w 1


In particular (setting n = k + r and n = k ) , we obtain
for n = 1, 2, 3,
2 k+r - 1 w 1 = 2 k - l w 1 , whence w1 = 0. And this is j ust enough to conclude
a 1 = b1 = c1 = d1 = 0 .
.

P roblem

4 7 , Solution 2
Introduce the polynomials

Pn (x) = a n x 3 + bn x 2 + CnX + dn

= 0, 1 , 2, . . . .
The conditions of the problem imply that Pk+ r (x) = Pk (x) and
Pn+l (x)
(a n + b n )x 3 + (bn + cn )x 2 + (en + dn )x + (dn + a n )
= a n (x 3 + 1 ) + b n (x 3 + x 2 ) + cn (x 2 + x) + dn (x + 1)
= (x + l) (a n (x 2 - x + l) + b n x 2 + cn x + dn )
---' (x + 1) ( -a n (x 3 - x 2 + x - 1 ) + a n x 3 + b n x 2 + C n X + dn )
= (x + l) ( Pn (x) - an (x - l ) (x 2 + 1)) .
for

x = 1, x = i, and x = - i (the imaginary units) ,


Pn+ l (l) = 2Pn (l), Pn+l (i) = (l+i)Pn (i), Pn+l (-i) = (1-i)Pn ( - i),

Hence, setting

and by induction:

= 0, 1, 2, . . The polynomials Pk and Pk+r coincide; so we get


2 k+r Po(l) = 2 k Po(l) and (li) k+r Po(i) = (li) k Po(i).
Since r ;::: 1, we have 2 r =/= 1, (l+it =/= 1, (l - it =/= 1, and thus

for

Po(l) = 0, Po(i) = 0, Po(-i) = 0 .


B y the definition o f Po(x), this means that ao + bo + co + do = 0 and
(2)
i(co - ao) = bo - do = i(ao - co) .
The "real-imaginary" equations (2) force ao = co and bo = do, and con
sequently the four numbers a 1 = ao + bo, b 1 = bo + co, c 1 = co + do,
d 1 = do + ao are equal, their common value being

1 05

Algebra

P roblem

48

The sequences

Show that

xo, x, x 2 , . . .

and

yo, Y l . Y 2 , .

. are defined by:

xo = YO = 1 ,
Xn + 2
Xn + 1
y; + 2 for n = 0 , 1 , 2 , . . .
Yn+l
2yn
Yn = X 2 " - 1 for every integer n ;:::: 0 .

P roblem

48, S olution 1
Define sequences ao, at , a 2 , . . . and

bo, b , b 2 , . .

bn - Yn - ...J2 for
Yn + ...J2
Denote the common value of ao and bo by .>. :
a n = X n +- .../2 '
X n .../2

.>.

v'2
1 + v'2
1 -

---

by

n = 0, 1 , 2 , . . .

= ao = bo.

The recursion formulas that define the sequences


analogous formulas for ( a n ) and ( bn ) :

(xn )

X n+l - ...J2
Xn+l + ...J2
x,.+2 ...j2
x,.+l
=
x,.+
x,.+l2 + ...j2
1
= X n .../2 . 1 - ...J2
Xn + .../2 + .../2
= >. a n ,
b n+l = Yn+l - ...J2
Yn+l + .../2
Yy,.+2 ...j2
2
y+2 + v'2
2y,.
(Yn - .../2 ) 2
(Yn + .../2) 2

a n+l

and

( Yn ) yield the

Solutions

106

Hence by obvious induction

b n = .>. 2"

for

n = 0, 1 , 2, . . .

2 n - 1 in the first equality we obtain


a 2 " -1 = .>, ( 2 " -1 ) +1 = .>. 2" = b n .
By the definition of an and b n , this is equivalent to
X 2 "- 1 ,.j2 = Yn - ,.j2
X 2 " -1 + ,.j2 Yn + ,.j2 '

Replacing n by

i.e. , to

2V2 = 1 - 2 V2 .
X 2" -1 + ,.j2
Yn + ,.j2
X 2 " -1 = Yn follows !

1The claimed equality

48, S o lution 2
The inductive definition of the XnS can be rewritten as

Problem

where

f(x) =

X n +1 = f(x n ) for n = 0 , 1 , 2, . . .
(x + 2)/ (x + 1 ) . Hence
/(1),
J
X 2n = ...._
___,_....
"
2
0

the circle denoting composition.


For n = 1 and n = 2 we are dealing with the functions

g(x ) = f o f(x)
x+2 +2
-= xX ++ 21
x+l +1
--

-3 x + 2
= 2 3 ,
x+2
f o f o f o f(x) = g o g(x)
=

(I)

107

Algebra

-1217 x + 2
=
x + -1217
Compare these expressions with the initial Yn S:
) 2 + 2 17
(
=
=
1,
Yo
Y2 2 . -12 .

(2)

It is natural to guess that

+2
f o o f(x) = YnX
X + Yn .

(3)

______.,
2"

Once guessed, this is without much trouble proved by induction; accord


ing to equations (1) and
equality (3) holds for n = 1 and n =
Assume i t holds for a certain n. Then

(2),

(x)

2.

o (x)

+2 +2
YnX
Yn
x
+
= YnX + 2Yn
x + y n + yn
(y;, + 2)x + 4yn
2yn x + (y; + 2)
y; + 2 x + 2
-= 2 yny;_ +' 2
x + 2 yn
+2
= Yn+lX
X + Yn+l '
showing that (3) holds with n replaced by n + 1 . By induction, claim (3)
is true for every integer n ;:::: 1. Now, setting in (3) x 1 we get in view
of representation (1)
X 2 n Yn1 ++Yn2
The number on the left side equals f (x 2 n - l ); that on the right side
equals f( Yn ) And since f(x) is strictly decreasing, we conclude that
X2n-l = Yn
---

--

= -- .

108

Solutions

Problem

48,

Set

Solution

Un = 1 + X n

n = 0, 1, 2, . . . .

1
1
Xn + 2
U n+ I = 1 + X n + I = 1 + --Un .
X n + 1 = 2 + --Xn + 1 = 2 + Consider the sequence vo, VI , v 2 , . . . defined by
vo = 1, Vn+ I = UnVn for n = 0, 1, 2, . . .
So

uo = 2,

for

(4)

and

( 1 ) Vn+ I = 2vn+I + -Vn + I ,


Vn +2 = U n + I Vn+ I = 2 + Un
Un

(5)

We obtain

i.e. ,

Vn +2 = 2vn + I + Vn

for

n = 0, 1, 2, . . . .

(6)

This is a homogeneous linear recursive equation of the second order. The


sequel is routine (see Problem 10, Solution 3, for instance) : the general
solution of equation (6) has the form

Vn = Aa n + B /3n for n = 0, 1, 2, . . . ,
where a = 1 + V2 and f3 = 1 - J2 are the roots of the characteristic
polynomial t 2 - 2t - 1. The constants A, B have to be determined from
the initial data vo = 1, VI = 2, thus creating the specific solution of
equation (6) we are looking for. The result is:

Vn = J2(a n +1 - 13n+1 )

for

n = 0, 1, 2, . . .

(simple calculations are omitted ) . Revisiting formulas


find

(4) and (5) , we

Xn = U n 1 = VnVn+ I 1 = Vn+IVn- Vn
0 n + l (a _ 1) _ 13n + I ( /3 1) v'2 . 0 n + I + /3n + l
=
=
0 n + I 13n +I .'
0 n + I _ 13n + I
_

this can be further rewritten as


(7)
Denote the number X 2 " - I j ust by
Wn = Yn for all n. Since wo = xo

w n . We have to prove the equality


= 1 = yo, it will be enough to show

Algebra

109

that the wns obey the same recursion that defines the
we have to show that

Wn+ l
According to formula

w n2 + 2
2 Wn

for

(7) , we have for n

n =

;::::

0,

12
,

, .

Yn S

That means,

(8)

where 'Y n stands for (a/ (3 f" ; the last equality is valid also for n
verification) .

'Yn + l 7;. Consequently,


1
w n2 + 2
Wn + _
2 Wn
2w n
J2 'Yn + 1
1 -'Yn - 1
=
+

2 'Yn - 1 J2 'Yn + 1
J2 . ('Yn + 1) 2 + bn - 1) 2
=
2
('Yn - 1) ('Yn + 1)
J2 . 'Y; + 1
'Y; - 1
In 'Y n +l + 1
v
'Yn + l - 1 - Wn+l for

Notice that

__

--

Equality (8) is settled, and the proof is complete.


Problem 49

Two sequences of integers

and
are defined uniquely by the equality

Compute

n =

0,

1, 2, . . .

0 (easy

Solutions

1 10

Problem 49 , S o lution 1

Expanding (2 + va t and ( 2 - va t binomially, we obtain in both ex


pressions the same coefficients of terms involving the even powers of va,
whereas those involving the odd powers of va differ in sign. Therefore
the equality ( 2 + va t = U n + b n va implies

(2 - VJ ) n = a n - bn v'3 j
this last sequence converges to zero because 2 - va is a number between

0 and 1 .

bn 2': 1 for all n 2': 1 . Therefore


a n = bn v'3 + (2 - J3 t = y'3 + (2 - J3 ) n
bn
bn
bn
which is v'3 in limit.
Note that

Problem 4 9 , S olut ion 2

The implicit definition of the ans and


sive formulas. Since

(2 + J3 t +l
(and since

bn s can be easily made into recur

(2 + v'3 t (2 + VJ ) = (a n + b n VJ ) (2 + VJ )
= (2a n + 3b n ) + (a n + 2bn ) v'3

va is irrational) , we infer
(ao = 1 , bo 0) ;

in the matrix form,

( :: ) = ( i ) ( : )

(I)

I t i s well-known that a pair o f sequences satisfying such a recurrence can


be postulated to have the form

eigenvalues of the matrix i


i.e., the roots
of the characteristic polynomial (2 - >.) 2 - 3:
p = 2 + v'3,
q = 2 - v'3.
The constants A , B, C, D are evaluated from the initial conditions
ao = 1 , bo = 0 and a 1 = 2, b 1 = 1 ,
where p and

are the

( ),

A lgebra

111

which yield the system of four linear equations

A + B = 1,

C + D = 0,

Ap + Bq = 2,

Cp + Dq = 1,

with the solution

C = -D = i J3.

A = B = .
Thus

A + B (qjp) n
C + D (q jp) n
Since 0 < qfp < 1, this ratio tends to AJC = J3 as n --+
Apn + Bqn
Cp n + Dq n

an
bn

oo .

P roblem 4 9 , Solut ion 3

We again use the recursive formulas


Formulas (1) imply

(1). Denote the ratio

a n fbn

n + 3 (x )
x n+ l - 2aa n ++2b3b n - 2x
n Xn + 2 - f n '
n
_

where
f (x)

by

Xn .
(2)

+3
1 .
= 2x
x + 2 = 2 x +_2 '
_

note that f is an increasing function in the interval (0, oo ) .


The initial terms of the sequence (x n ) are x 1 = 2, x 2 = 7/4. In view
of (2) and the strict monotonicity of f , the inequality x 1 > x 2 implies
by induction x n > Xn+ l for all n. So (x n ) is a decreasing sequence of
non-negative numbers, hence convergent to a limit l ;:::: 0. Passing to the
limit in the equality (2) we obtain the equation

l = 2ll ++23 ,
with the unique non-negative root l = J3.
-

P roblem 5 0

The sequence

(x n )

is defined by

Xl = 21 '
Prove the inequality

2n - 3 X 1
Xn =
n-

for

n=

2, 3, 4, .
.

Solutions

112

Problem 50, S o lut ion 1

Consider the auxiliary sequence Yn = (2n - 1)x n The recursion formula


that defines the x n s yields the analogous formula for the Yn S:

2n - 3
(2n - 1) (2n - 3) . Yn - 1 ,
Yn - (2 n _ 1) 2n . X n- 1 _2n
2(n - 1) - 1
i.e. ,

2n - 1
Yn =
Yn -1 for n = 2, 3, 4, . . . .
(1)
This formula i s valid also for n = 1 i f we set yo = 1. Now,
2n
Yn - 1 - Yn = 2n 1 Yn - Yn = 2nYn_ 1 = X n for n = 1, 2, 3, . . . , (2)
_

and therefore

X 1 + X2 +

+ X n = ( Yo - Y 1 ) + ( Y1 - Y2 ) + + (Yn - 1 - Yn )
= YO - Yn = 1 - Yn < 1,
(3)

as needed.
P roblem 50, S olut ion 2

x n s are
X2 = X 1 l = ! ,
The initial

X3 = X 2 = ! X4 = X 3 = ! ,
and in general ( by induction)
2k - 3 for k = 1 , 2, 3, . . . .
X k = 21 41 63 85 7 2k

10

Multiply the numerator and the denominator of this fraction by the


product of even integers from 2 to 2k - 2:
(1 3 . 5 - 7 . . . (2k - 3)) (2 . 4 . 6 . . . (2k - 2))

Xk =

(2 . 4 . 6 (2k - 2)) 2 (2k)


(2k - 2)!

= 4 k-1 1 2kk -- 12 . 2k1 .

(2kk -- 12) . ( - 2k2k- 1 )


(2kk -- 12) - _4k-1_1 (2kk -- 12) (2k -4k1)2 (2k)
(2kk -- 12) - 41k (2kk ) k = 1, 2, 3, . . . .

Continue the transformation as follows:

_
1
X k = 4 k- 1
= _k1_
4 -1
= 4 k-1 1

for

(4)

A lgebra

113

Fix an integer n 2: 1 and set in (4) k = 1 , 2, . . . , n; adding the equalities


that result we obtain, by telescoping,

X l + X 2 + + X n = .!_
4

(00)

(n )

_!_n 2n

()

_!_n 2n .
4 n

(5)

This number is smaller than 1 . Done.


Remark

Solution 2 does not differ from Solution 1 in any essential way ; the Yn S of
Solution 1 are expressed by the explicit formula Yn = 4 - n ( 2: ) ; equalities
(2) and (3) closely correspond to (4) and (5). ( In fact, Solution 2 indicates
how the idea of introducing the sequence ( Yn } in Solution 1 might have
arisen. )
Problem 5 0 , Solution

It is obvious that all the


recurrence as

XkS

are positive numbers. Rewrite the given

(6)
2kx k = (2k - 3)x k- 1 for k = 2, 3, 4, . . . .
Fix n 2: 2 and set in (6) k = 2, 3, . . . , n, n+ 1; if we add all the resulting

equalities and cancel the summands that occur on both sides, we get

X 2 + X 3 + + Xn + (2n + 2)x n + l = x 1 .
Hence

X l + X 2 + X 3 + + X n = 2x l - (2n + 2)x n+ l = 1 (2n + 2)x n + l


-

Since X n + l is a positive number, the value of this sum is smaller than 1 ;


the proof i s complete.
Problem 50, S olution

4
Assume the converse inequality to the asserted one:

X l + X 2 + X 3 + + Xn 2: 1
( for a certain n 2: 2); equivalently: x2 + x 3 + + Xn 2:
X 2 + X-=3 + + Xn -> 1 - 1 = 1
Xl
Xl
( as x 1 = !). We claim that then
X k+ l + . + X n > 2k 1
Xk

--

----

(7)

X t.

i.e. ,
(8)

(9)

114

Solutions

for k = 1, 2, . . . , n- 1 . We are going to show this by induction. For k = 1


the inequality (9) coincides with (8) . Assume that the estimate (9) is
true for some k (1 k n - 2) ; multiply (9) by X k /X k+ l and subtract
1 from both sides of the resulting inequality:
X k+2 +

X k+ 1

Xn

1= k+
(2k - 1)
X k+ 1 - 2 1 j
X k+l /x k = (2k - 1)/ (2k + 2)

2::

(10)

the last equality follows from


( the recur
sion formula from the problem statement ) . The induction step ( from (9)
to (10)) is done; thus inequality (9) holds for all k = 1, 2, . . . , n-1.
Now set i n ( 9 ) k = n - 1:

(11)
>- 2n - 3.
Xn 1
The fraction o n the left equals (2n - 3)/ (2n), according t o the definition
of the sequence. So the estimate (11) yields 1/ (2n) ;:::: 1 - obviously a
-

contradiction. Hypothesis
the assertion is true.

(7) must have been wrong, which means that

Problem

51
A sequence of real numbers ao, a 1 , a 2 , . . . satisfies the recurrence
a n - 1 + an +1 for n = 1, 2, 3, . . . . Show that a n+9 = a n for all n.

ia n I =

P roblem

5 1 , Solution 1
The sequence contains infinitely many non-negative terms. Choose one
of them. By the given recurrence, it is equal to t he sum of the two
neighbouring terms, one of which must be non-negative. So we have two
non-negative terms in succession: a m ;:::: 0, a m +l ;;::: 0. Then

One of these two differences is non-negative. This gives us a third non


negative term adj acent to the two already found.
Now, we have three successive non-negative terms, the middle one equal
to the sum of the other two. Denote them by a, a + b, b (a, b 2:: 0).
Assume a b. The recurrence determines the sequence fo rwa rd and
backward, and we are able to control the signs of the four terms that
follow the block a, a + b, b, as well as the signs of the four terms that
precede that block. Hence, this is a piece of our sequence:

. . . , b, 2b - a, b - a, -b, a, a + b, b, -a, a - b, b, 2b - a,
I n the case where a ;;::: b, the corresponding piece i s

1 15

Algebra

In either case, the two leftmost listed terms coincide with the two right
most ones, the two pairs being separated by a block of length 7. This
yields the desired periodicity.
Problem 5 1 , S olut ion 2

Define a transformation of the coordinate plane IR2 into itself by the


formula f (x, y) = (y , I Y I - x) ; its relevance to the problem is apparent
in view of

f(a n - 1 , a n ) = (a n , a n + 1 )
Fix a positive number r and consider the closed polygonal line

(1)

ABCDEFGHJA
with vertices

A = (O, -r), B = (- r, O), C = (-r, r ), D = (O, r), E = (r, 2r) ,


F = (r, r), G = (2r, r), H = (r, O ) , J = ( r, -r) ;
denote this line by Cr . Pick a point P from the segment AB ; t hus
P = (x, y ) , with y = -r - x, -r ::; x ::; 0, and hence its image
Q = J(P) = ( - r - x , 1 - r - x l - x) = ( - r - x , r)
lies on the segment CD and partitions it in the same ratio as P partitions
AB: CQ/QD = AP/PB . In an analogous fashion we show that CD is
mapped by f onto the segment E F, and so on: every side of the polygon
Cr is mapped onto the next-to-neighbouring side ( in the clockwise sense) :

AB ---+ CD ---+ EF ---+ GH ---+ JA ---+ BC ---+ DE ---+ FG ---+ H J ---+ AB.
Restricted t o each particular side, the mapping f i s linear; that i s t o say,
if a point divides a side in some proportion, then its image divides the
corresponding (oriented ) side in the same proportion.

It follows that the nine-fold application of f maps each point P E Cr


onto itself: j 9 (P) = P. And since the union of all polygons Cr ( as the
parameter r varies ) covers the whole plane ( except the origin, which is
obviously a fixed-point of f), we conclude that the ninth iterate ! 9 is the
identity map. This in view of equation ( 1 ) proves that a n + 9 = a n .
Problem 5 1 , Solut ion

The forward recurrence a n + l = l a n l - a n-1 yields the backward recur


rence a n - 1 = l a n l - a n + l Fix an index m 0; consider the terms a m
and a m + 9 , and write for brevity a m+4 = x , a m + 5 = y . Starting from
the pair x , y and applying four times the recurrence in its forward form

Solutions

1 16

l l l i Y I - x i - y i - I Y I + x i - I IY I - x i + y,
l l l i x l - Y l - x l - l x l + y l - l l x l - Y l + x.

and in its backward form, we express


follows:

a m+9
am

a m and a m+9 through x and y as


(2)

(3)

If we denote the expression on the right side of equation (2) by g(x, y ) ,


we get that the right side of equation (3) i s j ust g(y, x ) . S o the problem
reduces to showing that g(x, y) g(y, x ). The verification of this identity
is a more or less automatic task, rather tedious. (Without going too much
into details, let us j ust observe that in view of the symmetry between the
roles of x and y , one only needs consider three main cases: 0$x $y;
x $ 0 $ y; x $ y $ 0 ; but then they split into subcases . . . )
=

Solutions: Geometry
Problem

52
Construct a right triangle ABC with a given hypotenuse
of its medians are perpendicular.

c such that two

Problem

52, S olution 1
Assume the right angle is at C and the two perpendicular medians are
issued from the vertices A and C. Choose the coordinate system with
origin at B and with A on the x-axis. Let R be the midpoint of AB and
P the midpoint of BC. Suppose C has coordinates (u, v); thus

A = (c, O), B = (0, 0) , C = (u, v),


The orthogonality condition A P ..lC R

= (u j2, vj2) , R = (c/2, 0).

i s restated in terms o f the inner

product of vectors:

Since

C lies on t he circle with centre R and radius c/2,

Substitute this into the former expression to get u = c. So the point


is the foot of the altitude from C.
The method o f construction follows: draw t he semicircle with diameter
AB of the given length c; partition this segment in the ratio

D = (c, O)

AD : DB = 1 : 2.
Draw the perpendicular to AB through D ; it will intersect the semicircle
at C, the third vertex of the triangle sought.
Problem 52, S olut io n 2
Assume the triangle ABC, right-angled at C, has its medians AP and
CR perpendicular. They intersect at S, the centroid of ABC. Let Q be
the midpoint of AC and let T be the projection of S on BC. Since S
partitions A P in the ratio AS : SP = 2 : 1, the segment C P is partitioned
by T in the same ratio. If C S P has to be a right angle, S must lie on
the circle with diameter C P .
This yields the following method o f construction: draw a n arbitrary
segment BC, find its midpoint P and the point T on C P such that

1 18

Solutions

CT : T P 2 : 1. Draw the semicircle on C P as diameter; then draw the


perpendicular to BC through T ; it cuts the semicircle at S, the centroid
of the triangle. Find A as the point of intersection of P S and the line
perpendicular to C B at C. Triangle ABC has the desired shape, but
not the desired size. Transform everything using a suitable similarity to
obtain AB = c .
=

Problem 5 2 , Solut ion 3

Let P be the midpoint of side BC and S be the centroid of the triangle


we wish to construct (right-angled at C, with perpendicular me
dians from A and C). Let R and W be the midpoints of AB and BR,
respectively.

ABC

Construction: Draw segment AB of length c and erect semicircles k, k 1 .


k 2 and k 3 over AB, BR, AR and AW a s diameters (in one o f the two
half-planes determined by AB). Since ACB and ASR are assumed to be
right angles, points C and S have to lie on k and k 2 , respectively. Circle
k1 is the image of k in the homothety with centre B and coefficient 1/2,
and so P (the midpoint of BC) lies on k 1 . The centroid S divides A P
s o that A P
AS. Therefore P lies on the circle k 3 , which i s the image
of k2 in the .homothety with centre A and coefficient 3/2.
=

Hence, P is obtained as the point of intersection of k1 and k3 . The vertex


C is the point of intersection of line B P and circle k .
Problem 5 3

Let ABC be a triangle, AC =I= BC. Assume that the internal bisector of
angle AC B bisects also the angle formed by the altitude and the median
emanating from vertex C. Show that ABC is a right triangle.
Problem 5 3 , Solut ion 1

Denote by E the midpoint of AB and by H c the foot of the altitude


dropped from C. The perpendicular bisector of AB intersects the cir
cumcircle in two points, one of which lies on the other side of line AB
than vertex C; denote this point by D .

Geometry

1 19

D
Let 0 be the circumcentre of triangle ABC. S ince AD = BD, the arcs
AD and BD are equal, and so they subtend equal angles ACD and
BCD; thus ray CD is the bisector of angle C. According to assumption,
it bisects angle H eC E; in other words, angles H eC D and DC E are
equal.
Lines CHe and
fore

DE are parallel ( both are perpendicular to AB) . There

LEDC = LHeCD = LDCE,


which means that CDE i s an isosceles triangle: CE = DE . Note that
also triangle CDO is isosceles: CO = D O . Since 0 lies on line DE,
EO = (DO - DE) = (CO - CE)

(1)

( plus sign if E lies on segment DO, minus sign otherwise) . Hence, points

E and

0 coincide; otherwise CEO would be a non-degenerate triangle


and equality (1) could not hold. The circumcentre coincides with the
midpoint of side AB only if AB is the diameter of the circumcircle.
Thus LAC B = 90 .

Pro blem 5 3 , Solut ion

Let E and He have the same meaning as in Soh1tion 1 ; let a, b, c be the


lengths of sides BC, CA, AB and let a, {3, 'Y be the sizes of angles A, B ,
C , respectively. By assumption, angles AC B and He C E have a common
bisector, and this means that angles ACHe and BCE are equal:

LBCE = LACHe = 90 - L CAHe = 90 - a ;

120

Solutions

hence

LACE = LACE - LBCE = 7 - ( goo - a: ) = a + 7 - goo = goo - {3.


Apply the Law of Sines to triangles

ACE and BC E:

sin LACE

AE
CE

sin LCAE
=

BE
CE

sin ( goo - {3 )
sin a
cos {3
sin a:

(2)

sin LBCE
sin LCBE
=

sin ( goo - a: )
sin {3
cos o:
sin {3

(3)

Since E is the midpoint of AB, the left-side terms of ( 2 ) and (3) are
equal. Equating the right-side terms we obtain sin a: cos a: = sin {3 cos {3,
i.e. ,
(4)
sin 2a: = sin 2{3.
Sides AC and BC are not equal. Hence a # {3 , and so equation
2a: + 2{3 = 180 , which means that 7 = goo .
Problem

( 4) yields

54

If ABCDEF is a convex hexagon with

AB = BC,

CD = DE, EF = FA ,

prove that the altitudes ( produced ) of triangles


anating from vertices C , E , A , concur.

BCD, DEF, FAB, em

Problem

54, Solution 1
Consider three circles W I . w 2 , w3 , centred at D , F, B , respectively; W I
passing through C and E ; w 2 passing through E and A; and w 3 passing
through A and C . Let w 2 and w3 intersect at A and A'. Similarly, let w3
and W I intersect at C and C '. Finally, let W I and w2 intersect at E and

E'.

Points A and A' are symmetric across the line connecting the centres F
and B of circles w2 and w3 ; thus AA' ..l F B . This means that the altitude
of triangle F A B , dropped from A, is contained in line AA'. Analogously,
the other two altitudes considered in the problem are contained in lines
CC' and EE'.

121

Geometry

Now, line AA' i s the power axis o f circles W 2 and W 3 j lines c c ' and EE'
are the power axes of the pairs w 3 , w 1 and w 1 1 w 2 . For a triple of pairwise
intersecting circles whose centres are not collinear, it is a well-known fact
that the three power lines determined by pairs of these three circles are
concurrent. This proves the claim.
Problem

S olut ion 2

54,

The altitudes ( produced ) of triangles F AB and BCD, issued from ver


tices A and C, intersect at some point P ; so PAl. B F and PC l. B D .
I t will b e enough t o show that also P E l. D F. I n terms of vectors ( and
their inner products ) , we have to prove that the equalities

FA. . ifF = o

imply P"E F'D


.

and

PC . i5B = o

= o.

Points B , D , F lie ( respectively ) o n the perpendicular bisectors o f seg


ments AC, CE, EA, concurrent at 0 , the circumcentre of triangle ACE .
Thus o:8 AC = 0, on . cE = 0, oF EA = 0, and hence
.

oB AC + on GE + DF EA
= o:B (oc 01 ) + on . (DE - oc ) + oF (01 - DE )
= 01 . (oF - o:B ) + oc (DB - on ) + DE (on - "OF )
= 01 - ifF + OC DB + DE FD
(FA. - PO ) . IfF + (PC - PO ) J5B + (P"E - PO ) FD
= PA BF + PC DB + PE FD + PO (lfD + DF + F"B ) .
.

In the sum obtained, the first two summands vanish, according to as


sumption; and the vector sum in the parentheses is the zero vector.
Therefore PE FD = 0, and this is j ust what we wished to prove.
Problem

55

ABCDEF be a regular hexagon with M and N points on diagonals


CA and CE ( respectively ) such that AM = CN . If M , N and B are
collinear, prove that AM = AB.
Let

Problem

55,

Solution

Triangles ABM and CDN are congruent, as AB = CD = a ( the length


of the side of the hexagon ) , AM = CN ( by assumption ) , and

LM AB = LNCD = 30.
Therefore

LDNC = LBMA = LNMC,

122

Solutions

and consequently

LDNB

LDNC + LCNB
= LNM C + LCNM
= 180 - L M CN
180 - 60
120 .

Also LDOB = 1 20 , where 0 denotes the centre of the hexagon. Since


0 lies on the circle with centre C and radius a, it follows that also N lies
on this circle. Hence AM = CN = CO = a = AB.
Prob lem

55,

Solution 2

Let AC = CE = 1 ; then AB = ! v'J. Denote the common length of AM


and C N by x ; then C M = EN = 1 - x , and we have the vector equalities

eM = ( 1 - x) GA,

Since

B lies i n line with M and N , there exists a real number t such that
cB = ( 1 - t) CM + t eN

O n the other hand, EB

= ( 1 - t) ( 1 - x) cA + tx GE.

= ( EC + EA ) , whence

cB = GE + EB = C'E + (- C'E + ( CA: - C'E ) ) =

GA - ! C'E.

The representation o f a vector as a linear combination of cA and cE is


unique. Thus, equating the coefficients in the formulas above we obtain
( 1 - t) ( 1 - x)

= .

tx

= - 1.

These equations, combined, yield t + x = 0. Hence t -x and the sec


ond equation becomes x 2 = l Thus AM = x = l v'J = A B .
=

Problem

55,

Solut ion 3

Let points A , B , C , D , E, M , N be represented by the complex numbers


a, b, c , d, e, m , n, chosen as follows:

e=
Writing

CN : CE = AM : AC
m

= ( 1 - .X)a,

.X ,

we have

3-

iv'3
2

CM : CA = 1 - .X , so that

= .Xe (.X real positive) .

123

G eometry

The collinearity of B, M, N requires that the following quotient q be a


real number:
m-b
(1 - >.)a - b
(1 - >.)a - (a - 1)
1 - a>.
=
=
q =
>.e - b
>.e - b
e>. - b
-b
Since a = e and b = d ( bar denoting complex conjugation ) , the required
equality q = 7j takes the form
1 - e>.
1 - a>.
= -e>. - b a>. - d
Substitution of the numerical values of a, b, d, e simplifies this to:
3 >. 2 = 1. Hence AM = >. AC = >. i a l =
+ i = 1 = AB .

--

/l Vl

56
Let ABC be an acute triangle with altitudes B D and CE. Points F and
G are the feet of perpendiculars B F and C G to line DE. Prove that
EF = D G .
Problem 5 6 , S o lution 1
Since LBDC and LBEC are right angles, BCDE is a cyclic quadrilat
eral, and hence
Problem

LBCD = 180 - LBED = LBEF.

(1)

Therefore BEF and BCD are similar triangles, and we get


EF CD
=
(2)
B E BC
Analogously, considering the similar triangles CD G and C BE, we have
CD
DG (3)
=
B E CB
The asserted equality EF = DG follows immediately from relations (2)
and (3) .
Problem 56, S o lut ion 2
A slight variation of the previous solution: by equation ( 1 ) ,
EF = BE cos ( LBEF ) = B E cos C = BC cos B cos C,

(4)

where of course B and C are the angles of triangle ABC. The roles of
the point systems B , E, F and C, D, G are symmetric, and therefore we
may replace each character from the first system by the corresponding
one from the second. Formula ( 4) thus yields
DG = CB cos C cos B .

(5)

1 24

Solutions

Since the right sides of equations ( 4 ) and (5) are equal, so are the left
sides.
Problem 56, Solut ion 3
Let H and H1 be the midpoints of BC and FG. Quadrilateral B CGF
is a trapezoid; thus H H 1 is parallel to BF and CG, hence perpendicular
to DE. As in Solutions 1 and 2, notice that D and E lie on the circle
with diameter B C. Line H H 1 passing through its centre H and perpen
dicular to chord DE, must be the perpendicular bisector of that chord:
Consequently H1 is the common midpoint of DE and FG, and therefore
EF = DG.
P roblem 56, S olut ion 4
The repeated use of the Pythagorean Theorem will also do the job:
b.BFE :
b.CGD :
b.BEC :
b.BDC :
b.CGE :
b.BFD :

EF 2 + BF 2
BE 2 ;
CD 2 = CG 2 + DG 2 ;
BE 2 + CE 2 = BC 2 ;
BD 2 + CD 2 ;
BC 2
2
2
CE 2 ;
cc + EG
2
BF 2 + DF 2 .
BD
=

If we add these six equalities (and cancel the terms BF 2 , B E 2 , CD 2 ,


CG2 , CE 2 , BC 2 , BD 2 that appear on both sides) , we are left with
Since EG = DE + DG, DF = DE + EF, this is equivalent to
EF 2 + DE2 + DG 2 + 2 DE DG = DG 2 + DE 2 + EF 2 + 2 DE EF.
Hence DE DG = DE EF, and consequently DG EF.
Problem 57
Consider the right triangle ABC with LC = 90 . Let At and Bt be two
points on line AB (produced beyond A and B ) such that
=

AAt = AB = BBt
and let N be the foot of the perpendicular from A 1 to line BtC. Show
that the rectangle with sides B 1 C and C N has area twice as large as the
square with side AB.
Problem 57, Solution 1
Let CC1 be the altitude in triangle ABC and let N P be the altitude in
triangle A 1 B1N .

1 25

Geometry

Write

We have to show that EtC CN = 2 AB 2 ; that is, x y 2c2


The right triangles BtCtC and BtN A t are similar ( LABtN being their
common angle), and therefore BtCt : EtC = BtN : BtA t ; equivalently,
(c + p) x (x + y) : (3c), or
=

xy

3c2 + 3cp x 2

(2)

The altitude of the right triangle ABC satisfies the well-known equality
h 2 p(c - p); hence, by the Pythagorean Theorem for triangle BtCt C ,
=

Inserting this into equation ( 2 ) we obtain


xy =

(3c2 + 3cp) - (c2 + 3cp) 2c2 ,


=

as wished.
2
Segments AB and AtBt are diameters of two concentric circles whose
common centre is M , the midpoint of AB . Since LACE and LAtNBt
are right angles, points C and N lie on those circles. Line BtN cuts
the smaller circle in two points (which can coincide, in the limit case) .
Denote them by X and Y , with X lying closer to B t and Y closer to N ;
point C coincides with either X o r Y .
Problem 5 7, S olution

Solutions

1 26

Let S be the foot of the perpendicular from M to line B1N. Chords


XY and B1N of the two circles are perpendicular to line MS. Since M
is the common centre of those circles, M S is the common perpendicular
bisector of X Y and B1N. Therefore SB1 = SN and SX = SY ; denote
the common length of SX and SY by d. We get
(3)
B1X NY ( = B1S - d ) and B1Y = N X ( = B1S + d ) .
Our task is to prove the equality
=

(4)
According as C = X or C Y , the product B1C NC is either equal to
B1X N X or to B1Y NY . By virtue of (3) , we have
=

B1C NC = B1X B1Y


in each case. Considering segments intercepted by circle ( ABC ) on rays
B1N and B1A 1 . we have by polarity
B1X B1Y = B1A B1B.

And since B1A = 2 AB and B1B = A B , claim (4) results.


Problem 57, S o lut ion 3
This is a variation of Solution 1, from which we preserve notation (1).
Moreover, let AC1 = q . The altitude CC1 = h of the right triangle ABC
satisfies: h 2 = pq.
The Pythagorean Theorem now implies:
for triangle AtN Bt :
A t B = A1N 2 + NB ? ,
(5)
A1C 2
for triangle A1NC :
A1N 2 + N C 2 ,
(6)
2
2
2
A 1 C = A 1C 1 + C1C ,
for triangle A 1C1C :
(7)
B1C 2 = B1c r + C1c 2 .
for triangle B1C1C :
(8)

Geometry

127

Elimination of A 1N 2 and A1 C2 from equations (5), (6) , and (7) gives


N B - NC2 = A tB - AtC2 = AtB - (At C + CtC2) ,
i.e. ,
(9)
Equation (8) says that x 2 = (c + p) 2 + h 2 . Substituting this into equa
tion (9) (whose left side reduces to 2xy + x2) we obtain
2xy + (c + p ) 2 + h2

9c2 - (c + q) 2 - h2 .

Hence, in view of h 2 = pq and p + q


2xy

c,

9c2 - ((c + p)2 + (c + q)2) - 2h2


9c2 - (2c2 + 2c(p + q) + (p2 + q2)) - 2pq
= 9c 2 - 2c2 - 2c(p + q) - (p + q )2 = 4 c 2,

showing that xy = 2 c2
Problem 58
Let ABC DE be a convex pentagon inscribed in a circle. The distances
from A to lines BC, CD, DE, and B E are b, c , and d, respectively.
Express d in terms of b, c .
Problem 58, S o lut ion 1
Denote the feet of the perpendiculars from A to lines BC, CD, DE
and B E by H, N , P and K, respectively; so
AH , b = AN, c = AP,
d = AK . Assume, for definiteness, that N lies on segment CD, K lies
on segment BE, H lies on line C B produced beyond B , and P lies on
line DE produced beyond E. Obviously, other configurations are also
possible; the reasoning then requires but minor changes. The reader is
invited to find out what cases can occur and to draw suitable diagrams.
Now, in the case at hand: quadrilaterals AH B K , AH C N and AND P
are cyclic, each of them having two right angles at opposite vertices ( at
H , K , N , P). So
a,

a,

a =

LH AK = 180 - LH BK = L CBE,
LHAN = 180 - LHCN = 180 - LBCD,
LN AP = 180 - LN DP.

(1)
(2)

(3)

Since also BC DE is a cyclic quadrilateral ( inscribed in the given circle ) ,


LCBE = 180 - LCDE = 180 - LNDP,
LBED
180 - LBCD.

(4)
(5)

Solutions

1 28

Comparing equations (2) and (5) , we see that angle HAN equals BED,
hence also BAD (inscribed angle sub tended by the same arc B D ) . There
fore
L H AB = LH AN - L BAN = LBAD - LBAN = LN AD.

= LN AP - LN AD = LDAP.

(6)

Comparing equations (3) and (4) , we see that LN AP = LCBE, and in


view of (1) we get L H AK = LN AP; thus by ( 6 ) :
L B A K = L H AK - LH AB

(7)

On account of relations ( 6 ) and (7) , we have the following pairs of similar


right triangles:
i::J. H AB "' b.N A D

b.BAK "' b.DAP.

Consequently AH B K and AN DP are similar quadrilaterals, which im


plies that H AK and NAP are similar triangles. Thus
AK AP
=
AH AN
In other words, d/a = cfb, and we obtain the desired result:
ac
d= -.
b

58, S olution 2
Preserving notation of Solution 1, consider the angles:

Problem

= LABE = LADE,

f =

LAEB = LACB

( is the size of any angle subtended by arc EA, and f is the size of any
angle subtended by arc AB) ; and let
0!

= LDEA = 180 - LACD,

the last equality following from the fact that quadrilateral AC DE is


inscribed in the given circle.
Assume for the while that the projection points H , N , P, K are situated
as in Solution 1. Considering the right triangles AK B , AP D, AH C ,
A K E and A N C we see that
AK
= sin LABK
AB
= sin
sin LADP

AP
AD '
AH
sin LACH
AC
= sin e
sin LAEK
AK
AE '
AN
sin LACN
AC
sin LACD
sin(180 - a )
sin a .

(8)

(9)

( 10)

In the general case, each one of LABK and LADP might be equal either
to > or to 180 - >. This however does not affect the validity of formulas
(8) , as sin(180 - >) sin >; the same observation applies to the for
mulas in line (9) , while in line (10) angle AC N can be either equaL or
complementary to AC D, without affecting the formula. Thus, equalities
(8) , (9) , (10) are true in any case.
From (8) and (9) we have
=

AB
AD

AK
AP

-d and AE
c
AC

AK
AH

d
a

Multiplying these equalities,


d2
ac

AB AE
AC .AD

(11)

(a nice formula in itself) . Now, applying the Law of Sines to triangle


AD E and using equations (8) and (10) we obtain
AE
AD

sin >
sin a

--

AK : AB
AN : AC

AK AC
AN AB

d AC
-b AB

Hence

AB AE -d
( 12)
AC AD b
Equations (11) and (12) result in d 2 f (ac) dfb, and so, finally, d = acfb.
=

3
Denote the radius of the given circle by R. It is the circumradius of
each triangle determined by any three points out of A, B , C , D , E. To
Pro blem 5 8 , Solut ion

1 30

Solutions

express the area of any one of these triangles, we may apply either the
formula: {product of sides) / {4R) or: {base times altitude)/2. And thus:
area ABC =

AB BC A C B C AH
=
4R
2

=>

a=

area ACD =

AC AD CD
4R

CD AN
2

=>

AD
b = AN = AC 2R '

area ADE

AD AE DE DE AP
=
2
4R

=>

c=

area ABE =

AB A E BE
4R

Therefore
ac =

B E AK
2

AC
AH = AB 2R '

AP = AD

AE
,
2R

AE
d = AK = AB - .
2R

AB AC AD AE
= bd,
4R 2

implying d = ac fb .
Remark

The last solution is shortest , easiest to comprehend (though not to invent,


perhaps) , and it does not depend on any picture or assumption about
the particular configuration (unlike Solution 1 and, to some extent, also
the second one) . Moreover, it shows that A, B , C, D, E might be any
five distinct points on a circle, not necessarily the consecutive vertices of
a pentagon.
Problem 5 9

Let ABC be an isosceles triangle with base A B . Let U be its circumcen


tre and M be the centre of the excircle tangent to side AB and to sides
CA and CB produced. Show that 2 C U < CM < 4 CU .
Problem 5 9 , Solution

Let D be the intersection point of the circumcircle of triangle ABC and


line CM . Denote the incentre of triangle ABC by I. Rays AI and AM
are the internal and the external bisectors of angle A, hence they are
perpendicular and I AM is a right triangle.
Thus LIMA = LIAB = o/2 (where of course o = LCAB ) .
The orthogonality relations AC .l.AD and AI .l.AM also yield the equality
LM AD = LIAC = o/2. It follows that L M AD = LAM D, i.e. , DAM
is an isosceles triangle and we have D M = D A < CD; the last inequal
ity holds because CD is the diameter and AD is another chord of the
circumcircle of ABC.

131

Geometry

CD < CM = CD + DM < 2 CD.


And since CD = 2 C U , the claim results.
P roblem 59, Solut ion 2

Let he be the altitude from C and let Pe be the exradius from M to the
midpoint of AB . With the usual notation

BC = a, CA = b (= a), AB = c, a + b + c = 2s,
R = UA = UB = UC, F = area ( ABC )
we restate the claim as

2R < he + Pe < 4R.

(1)

Using the well-known formulas

abc = a2 c
R=4F 4F '

2F ,
he = c

F ,
Pe = -c
8 -

we recast inequalities (1) into the form

gp 2 4 F 2
2
2a 2 c < -c + -s - c < 4a c.

(2)

The area F is expressed by Heron's Formula

sc 2 (s - c) ; (3)
F 2 = s(s - a)(s - b) (s - c) = s (s - a) 2 (s - c) = 4
we have used the fact that the triangle is isosceles (a = b), so that
2a +c
c
s - a = a + 2b + c - a = 2 -a= 2.

132

Solutions

In view of formula (3) , claim (2) becomes just

2a 2 < s (2s - c) < 4a 2 ;


and since 2s - c = a + b = 2a, division by 2a reduces this inequality to

a < s < 2a.


The left part holds trivially, and the right part follows, for instance, from:
2a - s = (a + b) - s = ( 2 s - c) - s s - c > 0. The claimed inequality
(2) is thus proved.
=

Problem 5 9 , S olut ion 3

Let H be the midpoint of side AC. Suppose the excircle in question


touches side AB at T1 and the lines AC and BC at T2 and Ta, respec
tively.

The segments AT1 and AT2 are. equal, as they are the tangents from A
to the excircle. Thus AT1 = AT2 = c/2, and consequently CT2 = a + c/2
(with a and c standing for the lengths of BC and A B) . The right triangles
CHU and CT2 M are similar, and hence

CT2 = a + c/2 = 2 + !:_


a
CH
a/2
The proposed inequality says that the ratio C M : CU should be com
prised between 2 and 4, and so we are left with showing that
c
0 < - < 2.
a
CM

CU

The lower estimate is evident, and the right one is so too, due to the
triangle inequality c < a + b = 2a.

Geometry

1 33

60
The diagonals AC and BD of a convex quadrilateral ABCD intersect in
E. Let F1 , F2 and F be the areas of trians ABE, CDE and quadri
lateral ABC D, respectively. Show that y F1 + ..jF; :::; ../F. When does
equality hold?
Problem

60, Solut ion 1


Denoting the areas of triangles BC E and DAE by F3 and F4 , we have
to show that
...jF; + M :::; VFl + F2 + F3 + F4 .
D

Problem

A
By squaring, this is equivalent to

(1)
Let K and L be the feet of perpendiculars dropped to line AC from D
and B , respectively. (They can lie on or outside segment AC . ) Write
BL = b, DK d, AE = m, C E = n. Then
=

The inequality (1) we are about to prove becomes

Vmb nd :::; ! (nb + md) ;


and this is just the inequality between the arithmetic mean and the
geometric mean of the two products nb and md.
To achieve equality, we need equality between the averaged quantities nb
and md; and this is equivalent to

b : d = m : n.

(2)

1 34

Solutions

Lines BL and DK are parallel. So b : d = BL : DK = BE : DE, by the


Intercept Theorem, and we can restate (2) as
BE AE
(3)
DE CE
By the (inverse) Intercept Theorem, equation (3) holds if and only if
lines AB and CD are parallel, i.e., ABCD is a trapezoid with AB IICD.
This is the condition for equality in (1).
Problem 6 0 , S olut ion 2

Reduce the problem to inequality ( 1), as in Solution 1. Everything


goes even faster if we write BE = p, DE = q (preserving the notation
AE = m, CE = n) and express the areas Fi as
Ft = mp sin a,

r 2 = 2nq sm a ,
D

Fa = np sin ,B,

F4 = mq sin ,B,

where

,8 = LBEC = LDEA = 180 - a.

a = LAEB = LCED,

Hence sin a = sin ,B; denote this common value by 8 . Since a is a convex
angle, 8 is a positive number. Inserting the trigonometric expressions for
the Fi s into (1) we obtain the inequality
Jm p 8 nq8 :::; ! np8 + mq8
(to prove) . Factor 8 cancels and we are left with
Jmp nq :::; (np + mq) ,

the AM-GM Inequality for np and mq.


Equality requires that np = mq, i.e. , pfq = m/n; and this is nothing else
than equality (3) from Solution 1. Conclusion as before.
Problem 6 1

Let P1P2 be a fixed chord (not a diameter) of a circle k . The tangents to


k at Pt and P2 intersect at Ao. Let P be a variable point on the minor
arc P1P2 . The tangent to k at P intersects lines AoPt and AoP2 at A t
and A 2 , respectively. Determine the position of P for which the area of
triangle AoA 1A 2 is a maximum.
Problem 6 1 , Solut ion 1

Let M and r be the centre and the radius of k . Consider k as the


excircle of triangle AoA1A 2 escribed at side AtA 2 . Denoting by 8 the
semiperimeter of AoA 1A 2 , and by F its area, we have the formula

( 1)

Geometry

1 35

(Readers who have not encountered that formula are invited to provide a
proof, which is not at all difficult - using, e.g., the more familiar F = ps ,
with p the inradius, plus a similarity argument.)
The factor {s - A 1 A 2 ) in (1) is the distance from Ao to the point of
contact of the incircle with side AoA 1 . To make it a maximum, the
incentre should be chosen on ray AoM as far from Ao as possible; and
this is the case (given the conditions of the problem) when P is the
midpoint of arc P1 P2 .
Problem 6 1 , Solut ion 2

Choose M, the centre of k, to be the origin of a coordinate system, with


the radius of k as unit (r = 1). Thus the equation of k is x 2 + y 2 = 1.
Choose P1 P2 parallel t o y-axis; i n coordinates, let

P1 = (u, v),
P2 = (u, -v) ,
and let P (p, q). Assume u, v > 0, without loss of generality; then
u < p :5 1. The lines t 1 , t 2 and t 3 , tangent to k at P1 . P2 and P, are
=

described by the equations

v-q p-u ) A ( v+q u P )


A 1 -- ( pv-qu
' pv-qu ' 2 -- pv+qu ' pv+qu
(Ao = t 1 n t 2 , A 1 = t1 n tg, A 2 = t 2 n tg) .
The area F o f triangle AoA1A 2 is expressed by the determinant formula
tg: px + qy = 1.

t1: ux + vy = 1;

( . o),

They intersect pairwise at the points

Ao =

--

1 [1 ( p - u
p-u )
2 u pv - qu + pv + qu +
q . u-p + v +q u-p ]
+ pvv -- qu
pv + qu pv + qu pv - qu
1 ((p - u) f u) 2pv + 2v(u - p)
2
(pv - qu)(pv + qu)
v (p - u) 2
:;; p 2 v 2 q 2 u 2 i

x i and Yi standing for the coordinates of A i Substituting these coordi


nates,

- -

--"-

Solutions

136

here u, v are constants and p, q are variables satisfying p 2 + q 2


u2 + v2.
Therefore p 2 v 2 - q 2 u 2 p 2 v 2 (1 - F 2 )u 2 p 2 - u 2 , and hence

v (p - u) 2 v p - u v
2u
1
p2 - u2 p + u = - P + u .
Recalling that 0 < u < p :::; 1 , we see that F is a maximum when
2u
p+u
is a minimum, i.e . , when p is a maximum, i.e. , when p 1. This corre
sponds to P lying on the x-axis, hence coinciding with the midpoint of
arc P1P2 .
F

Problem 6 1 , S olut ion 3

Again, consider k to have centre M and radius

r =

1.

t2
Let the angles Ao, A 1 , A 2 of triangle AoA 1A 2 have sizes a , {3 , 'Y Now,
A1M is the bisector of LP1 A 1A 2 ; therefore L P1A1M 90 - (/3/2) , so
that (in view of P1M
1)
=

= r =

and similarly,
Knowing AoP1 AoP2
cot (a/2) cot(a/2) , we can calculate the
area F of triangle AoA 1A 2 from the trigonometric formula
=

= r

.!. AoA 1 AoA 2 sin a

Geometry

=
=

) (cot 2 - tan 2 )
sin a
(cot2 2 - cot 2 (tan !!._2 + tan 2 ) + tan !!._2 tan I).
2
2
(

sin a
cot - tan !!._
2
2
2

1 37

I
I

Since

a
f3
f3 1
1
a
tan - tan - +tan - tan - +tan - tan - = 1
2
2
2
2
2
2
'
we can express the product of the numbers tan(/3 /2) and tan (I /2) by
their sum:
f3
a
{3 + tan 21 .
tan 2 tan 2I = 1 tan 2 tan 2

Hence

sin a
a
a
a
F = -- cot 2 2 + 1 - tan 2 + cot 2
2

) (tan 2{3 + tan 12 ) ) .

The angle a is constant. Consequently the area F is maximized when


the sum tan(/3 /2) + tan(l /2) is minimized.
Now, one can set 1 = 180 - a - {3 and examine this sum by calculus,
as a function of the single variable {3 ; one can also use the convexity of
tan x to deduce that this sum is a minimum when {3 = I But we prefer
to use a more elementary argument:

. {3 + I
sm
2 2
=
{3
2
cos - cos -I
2
2
2 sin

{3 + tan -I
tan -

( + )
cos ( + ) + cos( - )
a

2 cos 2
=
/3 - 1
. a
sm
2 + cos -2For a fixed a , this is a minimum when cos((/3 - 1)/2) = 1, i.e., when
{3 1, and we arrive at the same conclusion as in the two previous solu
tions.
=

Problem 62

Let P be a point inside a parallelepiped whose edges have lengths a,


and c. Show that there is a vertex whose distance from P does not
exceed Ja 2 + b 2 + c 2 .
b

Solutions

138

62, S o lut ion 1


Consider the six planes containing the faces of the parallelepiped. Let
be the plane whose distance from P is a minimum, let ABC D be the
face contained in and let N be the foot of the perpendicular dropped
from P to
Then N lies within ABCD; otherwise the segment PN
would intersect another face, less distant from P than contrary to the
choice of Assume without loss of generality that the edges not parallel
to have length c. Then P N $ c/2.
Now, N is a point inside parallelogram ABC D , with sides of lengths
a and b. Repeating the previous reasoning ( one dimension lower) , we
find a side of ABC D whose distance from N is a minimum. Assume
( relabeling if necessary ) that this is side AB, with AB = a. Denote by
K the foot of the perpendicular from N to line A B ; then K is a point of
the segment AB . Note that N K $ b/2. We may also assume AK $ B K .
Thus AK $ a/2.
The three segments AK, K N , N P are the edges of a rectangular box
and P A is its space diagonal. Hence, finally,
Problem

1r

1r

1r .

1r ,

1r .

1r

PA
<

VAK 2 + KN 2 + NP2
J(a/2) 2 + (b/2)2 + (c/2) 2
! v'a2 + b2 + c 2
.

62, S o lut ion 2


Let 0 be the centre of the parallelepiped and let u, v, w be vectors of
lengths a/2, b/2, cj2, parallel to the respective edges. The vertices can
be labeled so that

P roblem

----+

OAt = u + v + w, OA 2 = -u - v - w,
OAa = -u + v + w, OA4 = u - v + w, OAs = u + v - w,
OAs = u - v - w, OA7 = -u + v - w, OAs = -u - v + w.
--+

--+

--,-.+

( In fact, vectors u, v, w provide a basis of a non-orthogonal coordinate


system in the space. ) The vector OP determined by the given point P
has representation
--+

OP = xu + yv + zw with

- 1 $ x, y, z $ 1.

Consider the eight non-negative numbers - (1 + ix ) ( 1 + jy) (1 + kz) with


i, j, k taking independently values +1 and - 1. Denote them by P l , . , ps
according to the rule:
.

---t

if OA m = iu + jv + kw then Pm =

(1 + ix) (1 + jy) (1 + kz )
8

{1)

139

Geometry

Compute their sum:


8

L Pm = S1

m =l

(1 + i x + i y + k z + ij xy + ik xz +jk yz + ijk xyz ) .

i , j, k E { +l,- 1 }

When ( i, j, k) range over the set of the eight triples of plus-minus ones,
then each one of the expressions i, j, k, ij , ik, jk, ijk takes values +1
and - 1 equally often. Therefore the sums L; i, L; j , L; k , L; ij , L; ik,
L; jk, L; ijk are zero, and hence

t Pm (8 + X ?: i +
=

m=l

,J ,k

+ xyz ?: ijk = 1.
,J,k

(2)

Choose an index m E { 1, 2, 3, 4, 5, 6, 7, 8}; it corresponds to a certain


configuration of plus ones and minus ones, in agreement with (1) . For
those values of i, j, k:
( 0A m - OP ) 2
[( i - x)u + (j - y)v + ( k - z)w] 2
= [i (1 - i x ) u + j (1 - j y)v + k (1 - kz)w] 2

PA

(3)
where

(4)
Um = (1 - ix) 2 u 2 + 2jk (1 - jy) (1 - kz ) ( v w) ,
Vm is obtained from Um by the cyclic shift i -+ j -+ k -+ i and the si
multaneous shift x -+ y -+ z -+ x, and Wm arises from Vm in the same
manner. By (1) and (4) ,

(1 - x 2 ) (1 - ix) (1 + jy) (1 + kz) u 2 +


8
(j k + ijkx ) ( 1 - y 2 ) (1 - z 2 ) V W .
+
( )
4
Summing over m = 1, . . . , 8 ( that is, over all possible configurations of
signs i, j, k ) we obtain
8

L Pm Um

m=l

[ ?: (1 - ix ) ( 1 + jy) (1 + kz)] (1 - x 2 )u2 +


+ [ ?: (jk + ij kx) ] (1 - y 2 ) ( 1 - z 2 ) (v
'I.,J, k

,,,k

w) .

Solutions

140

The first sum in square brackets equals 1, and the second one equals 0;
see the argument preceding definition (2) . Thus
8

L PmUm = (1 - x 2 ) u2 .

m= l
Analogously, by cyclicity,

L PmWm = (1 - z 2 ) w 2 .

L PmVm = (1 - y 2 ) v 2 ,

m=l
m= l
Equalities (3) , which hold for m = 1, , 8, now imply
.

L P m PA = (1 - x 2 ) u 2 + (1 - y 2 ) v 2 + (1 - z 2 ) w 2

m =l

u2 + v 2 + w 2
a 2 + b2 + c2
=
4
In view of (2) , this sum is a weighted mean of the eight numbers
<

PA, . . . , PA

(with weights P I , , P 8 ) At least one of those numbers does not exceed


the mean. Consequently, there exists an m such that
a 2 + b 2 + c2
PA 2m <
'
4
and this is exactly what had to be proved.
Problem 6 3
Do there exist two cubes such that each face of one of them meets each
face of the other one (possibly at an edge or a corner)?
Problem 6 3 , Solution 1
Suppose a cube C has vertices (1, 1, 1) and let be a plane not
passing through the origin 0 = (0, 0, 0) and having points in common
with all the six faces of C. The equation of can be written in the general
form ax + by + cz = k , with k ::J 0, a 2 + b2 + c 2 > 0. The distance
from 0 to equals d l k l f v'a2 + b2 + c2 In view of the standard
symmetries of C, there is no loss of generality in assuming c b a 0.
By assumption, meets (in particular) the two faces of C, perpendicular
to the z-axis. So there exist points P = (p, q, - 1) and U ( u, v , 1), both
lying on with coordinates p, q, u , v E [- 1 , 1] . Consequently,
.

1r

1r

1r

1r

1r ,

k = ap + bq - c :::; a + b - c :::; a and k = a u + bv + c -a - b + c -a;

141

Geometry

these two inequalities jointly imply a J k J . So a


d

>

0, and hence

a
a
Jk l
<
<
=
.ja 2 + b 2 + c 2 - .ja 2 + b 2 + c 2 - .ja 2 + a 2 + a 2

__!.._

v'3

<

1.

Thus if a plane meets all faces of a cube, its distance from 0 , the cube
centre, is shorter than the distance of any face of that cube from 0 .
Assuming that two opposite faces o f another cube C ' meet all faces o f C ,
we are led t o the conclusion that C ' has strictly smaller size than C. And
since the roles of the two cubes in the problem statement are symmetric,
the negative answer results.

Problem 63, Solution

Let C, C' be the two cubes. Assume C has edge length 1 and C' has edge
length 1. Choose two opposite faces of C'; visualize them horizontally
and call them B and T (base and top) . Denote by 1{. the half-space
consisting of all those points that lie below or on the plane of B. Suppose
it contains at least two non-adjacent vertices A, B of C. Let M be the
midpoint of AB. Clearly, M belongs to 1{. .
I f AB i s a space diagonal o f C then M i s the centre o f C, and consequently
every point of C lies within distance ! v'a from M . Since the distance
between B and T is at least 1, the top face T is disjoint from C.
If AB is a face diagonal of C then M is the centre of the corresponding
face, whose all points lie therefore within distance v'2 from M . Since
also this number is smaller than 1 , the face in question (of C) cannot
reach T.
Now assume there are no two non-adj acent vertices of C in 71.. This
means that 1{. contains either no vertex or exactly one vertex of C, or
exactly two vertices of C, linked by an edge. Among the remaining (8 or
7 or 6) vertices of C one can find four points that span a face of C. As
they are situated strictly above the plane of B, that face has no point in
common with B.
Thus, in any case, C has a face that does not meet either B or T. A pair
of cubes with the proposed property does not exist.

Remark
An analogous problem might be considered in the four-dimensional space:
do there exist two 4-cubes in R.\ each 3-face of one cube meeting each
3-face of the other one? The answer, rather unexpectedly, is yes .
Example: let C be the 4-cube whose vertices are the 16 points
(1, 1, 1, 1).
Pick those points that have an even number (four, two o r none) o f co
ordinates equal to 1 - there are eight of them - and adjoin to that

Solutions

142

set another eight points, each having one coordinate equal to 2 or -2,
and the remaining three coordinates 0. These sixteen points also span a
4-cube C' with the property as needed: if you choose arbitrarily a 3-face
of C and a 3-face of C', those two "faces" (3-cubes) will have at least one
common vertex! (To verify this, without having to deal with too many
cases, can be a nice challenging exercise in itself. )
The olympiad problem , discussed above, has been motivated by this
four-dimensional example.

Problem

64

Let A 1 , A 2 , A a , A4 be points on the sphere circumscribed about the


regular tetrahedron with edge 1 such that AiAj < 1 for i -:f. j. Prove
that these four points lie on one side of a certain great circle of the
sphere.

Problem

64,

Solution

To say that the points lie on a certain hemisphere is as much as to claim


that the tetrahedron spanned by these points does not contain the centre
of the sphere in its interior. Thus assume that 0 , the centre of the sphere,
lies inside tetrahedron A 1 A2AaA4, which is therefore the union of four
pyramids (numbered 1 through 4) , with a common vertex at 0 , the ith
pyramid having for its base the face of A 1 A2AaA4 opposite to Ai. Let Vi
be the volume of the ith pyramid and let di be its altitude issued from
vertex 0 ; without loss of generality assume d 1 d 2 ;:::: da ;:::: d 4 .
Further, denote by hi (i = 1 , 2, 3, 4) the altitude of the "large" tetrahe
dron A 1 A 2AaA4 dropped from vertex Ai to the opposite face and let V
be the volume of A 1A2AaA4. Thus Vi/ V = d i/ hi (the ratio of volumes of
those two pyramids equals the ratio of their altitudes, dropped to their
common base) .
The foot of altitude d4 of pyramid O A 1 A2Aa coincides with O', the
circumcentre of t riangle A 1 A 2 A a . This triangle cannot be obtuse-angled;
for if, say, L A a were obtuse, then the points O' and Aa would lie on
distinct sides of line A 1 A 2 (within plane A 1A2Aa) , so 0 and O' would
lie on distinct sides of plane A 1A2A4, and the distance from 0 to that
plane would be smaller than 0 0 1 , in contradiction to da d4 .
Let Vm = min( V1 , V2 , Va , V4) . S ince V = V1 + V2 + Va + V4 , the volume
Vm does not exceed V . Note that h m d m + R, where R denotes the
radius of the sphere. Thus
< h m < dm + R
d m = Vmhm 4 4
V

whence
(1)

Geometry

143

Now, visualize a regular tetrahedron inscribed into the given sphere so


that one of its faces ( call it ) is parallel to plane A 1 A A 3 . By hypoth
2
esis, all edges of have length 1 . Point 0 , which is simultaneously the
circumcentre, orthocentre and centroid of the regular tetrahedron, par
titions each of its altitudes in ratio 3 : 1 . Hence, the distance from 0 to
is exactly ! R . Comparing this with inequalities ( 1 ) we see that the
plane A 1 A A 3 is less ( or equally ) distant from 0 than ( as ) the plane of
2
- Consequently the circumradius of triangle A 1 A A 3 , denote it by r,
2
is not smaller than the circumradius of :
(2)
In triangle A 1A A3, let LAk be the greatest angle and let i, j be the
2
two remaining indices in { 1 , 2, 3 } . The triangle is not obtuse-angled,
and so the size of LAk is comprised between 1r / 3 and 1r /2. As O ' is
the circumcentre of A 1 A A3, the angle LAiO' Aj = 2 L Ak is comprised
2
between 27r / 3 and 1r . Therefore cos ( LAiO'Aj ) ::::; - 1 / 2 and we obtain,
by the Law of Cosines and by inequality (2) ,
(AiAj ) 2

(0' Ai)2 + (0' Aj ) 2 - 2 01 A i O' Aj cos ( L AiO' Aj )


2r2 (1 - cos ( L A i01Aj ) ) 3r2 1 .

This is however impossible, according t o the condition o f the problem.


Contradiction ends the proof.

Problem

64,

Solution

As in Solution 1 , denote by 0 and R the centre and the radius of the


given sphere. Let T1T T3T4 be any regular tetrahedron inscribed in that
2
sphere. The conditions of the problem require that
(3)
Consider the following vectors:

Relations (3) can be translated into the language of inner products:


=

....,....--.? ....,.-,-lo
AiAj AiAj
(OAj - OAi) (OAj - OAi)

(OAj ) 2 - 2 0Ai OAj + (OAi ) 2


2(R2 - U i Uj )

and similarly

so that inequalities (3) become


(4)
Denote by w the sum

(5)
The analogous sum of vectors Vk is the zero vector

(6)
this is j ust a restatement , in terms of vectors, of the fact that point 0 is
the gravicentre of the system of equal point masses placed in the vertices
of the regular tetrahedron.
We now multiply, in the sense of inner product , both sides of equation
(5) by vector Ut :
u1 w

u1 ( ut

Similarly, multiplying equation

(6)

Vt (vt

Subtracting equation (8) from


U t U t = Vt Vt = R 2 ) we get

+ u2 + ua + u4) .

(7)

by v1 we obtain

+ v2 + va + v4) .

(8)

(7) (and making use of the fact that

In view of estimates (4) , the three numbers in parentheses are positive.


Therefore u1 w > 0. The same argument may be repeated with any
one of the vectors Uk in place of Ut . Thus
Uk w

>

0 for k = 1 , 2 , 3 , 4 .

(9)

The inner product of two vectors is positive if and only if they form an
acute angle. Inequality (9) thus says that all the four vectors OA k are
inclined to the vector w under acute angles. If we now draw through
0 the plane orthogonal to w , we get the four points A t , A 2 , Aa , A4
collected on one side of it - and this is exactly what we need.

Problem

64,

Outline Solution

Assume, contrary to assertion, that 0 is an interior point of tetrahedron


A 1 A 2 A3A4 . The four solid angles OAiA;Ak dissect the sphere into four
non-overlapping spherical triangles, of joint area 47r R 2 Hence, at least
one of them has area greater than or equal to 1r R 2 Since the distance
between any two points A i , A; is less than 1, their "angular distance"

Geometry

145

( size of angle AiOAj ) is smaller than the angular distance between any
two vertices of a regular tetrahedron ( of edge 1 ) inscribed in the sphere.

The analogous construction involving solid angles, performed with use


of a regular tetrahedron, produces a partition of the sphere into four
congruent tr-iangular quarters ( spherical triangles ) , of area 1r R 2 each.
We have shown that the largest of the spherical triangles AiAjAk has
area at least 1r R 2 , whereas its "sides" ( circular arcs ) are strictly shorter
than those of a triangular quarter.
These two inequalities contradict each other. Intuitively, this is easy to
believe; a rigorous proof is less easy!

M arcin

Kuczma

graduated with a PhD


in

Pure Mathematics

from the University of

Warsaw. He is now a
Senior

l nstru ctor

in

Pure Mathematics

in

the University's l nstit


ute

of M athemati cs.

His research specialty is real analysis.


He

has

much

experience

as

problem

composer and jury member of the Polish,


and

Austri an/Polish

l n tern ational

M athematical Olympia d s and is an author


of

book

on

the

Austrian -Polish

mathematical Olympiads. He has composed


four

problems

Mathematical

in

the

Olympia d

ln tern ati onal

and

had

many

m ore shortlisted.
Dr Kuczma is probl em contest editor in the
journal Delta, is a frequent contributor to
problem columns in several journals and in
1 99 2

was

Hilbert

awarded

Award

contribution

to

the WFNMC David

for

his

si gn i fi ca n t

the

enrich m ent

of

mathematics learning internationally.

Erich Windischbacher
has worked as a high
school

tea cher

in

Graz, Austria for more


than 30 years. He has

(p

also taught at the Karl


Franzens University in
Graz

and

at

Peda gogical

the

lnstitute

for tea chers.


Since 1 9 69 he has been very much engaged
with mathematics
Austri an

competitions and

M athem atical

Olympia d .

the
Prof

Windischbacher co-authored several books


e . g.

Os terreichisch e

0/ympiaden
Mathematik
A U STR A l l A N M ATH E M ATl C S T R U ST

E N R l C H M E N T

S E R l E S

1 9 70- 1 989

Mathema tik
and

Wege zur

Anregungen

und

Vertiefungen.

l SBN 1 8 7 6 4 2 0 0 2 2

Você também pode gostar